All Recalls of MD

You might also like

Download as pdf or txt
Download as pdf or txt
You are on page 1of 517

‫بسم هللا الرحمن الرحيم‬

RECALL MD EXAM 8.2021

DR.YOSRA HAMAD ABD ELRAHIM

BEST OF 4:

1. Pt with aortic stenosis , then developed heart failure and his


EF% decreased, What is the sign will be seen ?

Decreased in second heart sound

Increase in murmur

2. Pt with VSD present with tonsillitis. His murmur gets Harsher


from before many years, what your action?

Cardiac catheterization

Refer for surgical intervention

Penicillin prophlyactic

Observation

3. Pt with chest pain,ECG show inferior MI with 3mm ST elevation,


after Thrombolysis ST elevation became 0.5 mm on asprin,
statin and heparin,there is 2 episode of VT 6 to 10 seen in
monitor Vitals stable and pt free of pain, What is treatment ?

PCI

Beta blocker

Amiodarone

Pacemaker

4. Which disease of following associated with blunt y descend?

Constrictive pericarditis

Restrictive cardiomyopathy

Cardiac tamponade

Tricuspid regurgitation

5. Pt with anti phospholipid syndrome presented with SOB


suddenly ECG show ST depression in v1-v3 and mildly elevated s.
Troponin , what is appropriate step ?

CTPA

PCI

ECHO

6. Pt with heart failure after delivery received diuretic ,beta


blocker, spironolactone, and still symptomatic ,EF 25% what is the
best step?

CRT
ICD

ACE i

digoxin

7. Pt develop narrow complex tachycardia with absent p wave


,has h.of asthma , has normal vitals and has no chest pain ,what is
the treatment?

Adenosine

betablocker

verapamil

amiodarone

8. PT presented with acute chest pain,his vital BP 90/60 and


tachycardia and tachypenic.. ECG show rt V.strain pattern,Best
treatment :

Thrombolysis

Anticoagulant

Embolectomy

9. Pt recently was in farm present with cough, fever, jaundice


and flushing. CXR: feature of pneumonia deranged RFT, Deranged
LFT diagnosis:

Q fever

Leptospirosis

Legionella
Mycoplasma

10. Pt with history of asthma and drop foot with mild renal
impairment what is the diagnosis?

Wegener granulomatosis

Churg strauss syndrome

PAN

11. Lady with DVT on warfarin discovered to has pregnancy test


positive what is the best treatment?

Continue warfarin

Stop warfarin and give heparin in first TM ,then warfarin in

second TM and heparin in last TM

Give heparin in all trimesters

Delivery with cesarean section

12. Post MI pericarditis within 2 days what is the best


Management?

Steroid

Aspirin

13. What Most common arrhythmia that occurs 2 hours post MI ?

SVT

VT

AF
VF

14. Which One of the following drugs NOT used in IHD?

Labetalol

Atenolol

Bisoprolol

carvidelol

15. Young pt present with syncopal attack , murmur increase with


valsalva . Has abnormal BP during exercise, v. Thickness 2.5 cm

From following what is NOT ass with poor prognosis?

Family history

Early age at onset

V. thickness of 2.5 cm

Low BP during exercise

16. Young female pt with H of dizziness and repeated fell down


when playing and dark room ,on examination she has red eyes ,
no tremor or nystagmus also has mother has similar symptoms
when she washes her face what is the diagnosis?

ataxia telangectasia

Spinocerebllar degeneration

MS

17. In acute ischemic stroke what is the best investigation ?


MRI

CT

DWI

18. Pt with weakness and cortical blindness what is the


management ?

Heparin

Aspirin

Warfarin

19. Pt with dizziness and unsteadiness the symptoms aggravated


when turn on bed ,no nystagmus what is the diagnosis ?

Cerebellar ataxia

BPPV

Labrynthnitis

20. Pt with unilateral hearing impairment and loss of corneal


reflex at the same side where is the lesion ?

Cerebellopontine angle

Brain stem

21. Pt c/o hot sensation in throat ,tonsil and edge of mandible


how to manage ?

Amitryptalin

Gabapentin
Carbamazepine

22. Pt presented with hemiplegia on examination and


investigation he has AF what is the management ?

Thrombolysis

Aspirin

Warfarin

Heparin

23. pt with facial sensory loss and contralateral hemisensory loss


what is the diagnosis?

Lateral medullary syndrome

24. Pt with RT hearing loss and on examination there is nystagmus


when pt follow finger moving to the RT side of examiner, where is
the lesion?

Lt cerebellar lesion

Cerebellar vermis lesion

Rt vestibule

25. female known epileptic on Na valporate was founded to be


pregnant what is your action ?

Continue valporate

Switch smoothly to lamotrigen

Delivery with C/S


26. Pt with long history of asymmetrical weakness with
fasciculation with mild wasting no ataxia , sensory intact ,NCS
show nerve conduction block what is the diagnosis ?

GBS

CIDP

MMN

MND

27. All correct about carpal tunnel syndrome except:

Atrophy of thinner muscle

Loss of sensation in the ulner aspect

28. What is the type of ischemic stroke having high tendency to


transform into haemorrhagic?

Small vessels

Large vessels

Cardio Embolic

Thrombotic

29. : Drug newly discovered to be having role in stroke recovery:

Amantadine

Piracetam

baclofen
30.pt with severe headache awake him from sleep ,with nasal
congestion and eye redness repeated for 3 times ,comes every
year (seasonal) what is the management?

Codeine

Gabapentin

Indomethazine

Aspirin

31. pt with history of forgetfulness, urinary incontinence and


ataxic gait what is the diagnosis ?

Normal pressure hydrocephalus

CJD

MSA

LBD

32. What is the disease ass with ptosis without ophthalmoplegia ?

Myotonia dystrophia

Myasthenia gravis.

33. Which following malignancy metastasis to brain lead to


haemorrhagic stroke:

Lung CA

RCC

Non melaninotic skin cancer


Colonic cancer

34.From the followingwhich is affected in of pneumonia and ILD ?

Alveolar exchange

Vascular involvement

35. Pt presented with SOB ,PFT show normal FIV1/FVC and


decreased TLCO and KCO what is the diagnosis?

Asthma

Polycythemia

HF

Anemia

36. pt with fever and SOB investigation show eosinophilia, feature


of ILD and restrictive cardiomyopathy what is the diagnosis?

hyperesonophilic syndrome

Chronic eosiophilic pneumonia

Churge strauss syndrome

37. pt with Liver impairment and SOB when sit from lying flat ,
CXR showed increased vascularity in lower lobes what is the
diagnosis?

Hepatopulmonary syndrome

A1AT D

AVM
38.pt with SOB ,PFT revealed increased FIV 1/FVC and decreased
TLCO and normal KCO what is the diagnosis ?

Asthma

Lung fibrosis

Pneumoctomy

39.HIV pt with SOB and haemoptysis investigation showed +ve


galactomannan test , most likely diagnosis is?

Aspergilloma

ABPA

Invasive aspergillosis

40. Pt presented with increasing SOB for long time CXR:decreased


lung volume biopsy revealed feature of usual interstitial
pneumonia with neutrophil predominant what is best ttt?

Predisolone

Perfinedone.

Lung fibrosis

Asthma

41.Pt with COPD treated with oxygen before ABG what is


appropriate oxygen saturation level?

98-98

92-100
88-92

95-100

42.Pt received 5 units of FFP then developed severe SOB many


hours after transfusion, O/E widespread Rhonchi and CXR with
bilateral pulm. Infilterate , PCWP is normal , what is the diagnosis?

Pulmonary edema

TRALI

ARDS

Severe pneumonia

43. pt with low ph ,high co2 , low o2 , low HCO3 from the
following what is the correct ?

Fully compensated respiratory acidosis

Compensated metabolic acidosis

Mixed respiratory and metabolic acidosis

Mixed respiratory and metabolic alkalosis

44. Pt with RA for commencement of methotrexate , has contact


with TB Mantoux test –ve ,what is investigation of choice?

CXR

IGRA essay

Sputum for AFB


45.Pt come from travel C/O dry cough and then become ill and
has SOB ,CXR: bilateral infiltrate has deranged LFT the best
investigation is ?

Mycoplasma Serology

HRCT

Urinary legionella antigen

46. HIV pt with dysphagia and CD4 150 the most likely diagnosis
is?

CMV

Candidasis

Esophageal cancer

47.pt with high grade esophageal metaplasia what is the best ttt?

Esohagectomy

Annual endoscopy

Resection and radiofrequency ablation

48. pt with feature of malabsorption ,high MCV , anti endomyseal


–ve , xylose +ve what is the diagnosis?

Celiac

SBBO

Tropical sprue

Whipple disease
49. you called to see a pt in OBS ward delivered recently ,
developed haematemesis ,has h.of using OCP ,O/E: obese , ascitis
and splenomegaly what is the diagnosis?

Liver cirrhosis with portal HTN

portal vein thrombosis

Fatty liver of pregnancy

50. Pt with lymphoid tissue associated tumor best ttt is?

Chemotherapy

H.pylori eradication

Surgical resection

Radiotherapy

51. 50 yrs old pt with epigastric pain and iron deficiency anemia,
O/E: he is slim ,underwent endoscopy and colonoscopy ended
without taking biopsy due to pt discomfort ,what to do?

Repeat endoscopy and colonoscopy

Red cell isotope scan

White cell isotope scan

Anti tissue Trans glutamase

52.Pt with chronic watery diarrhoea ,it continue even during


fasting ,inv: hypokalemia what is the ttt?

Octreotide
cholestyramine

53. Pt with ulcerative colitis presented with mild to moderate


bloody diarrhoea what is the best ttt?

Local steroid

local MSA

IV steroid

Oral MSA

54.Pt presented with abdominal pain and ascitis , ascetic fluid


send for analysis and start antibiotic but no improvement ,Ascetic
fluid revealed high neutrophil and waiting for culture, what to do?

Change antibiotic

Stop antibiotic

Continue antibiotic till culture result

Do abd. CT

55. Pt with gout on allopurinol discover to have h.pylori positive


what is the best management?

Omeprazole clarythromycin amoxicillin

Omeprazole,clarythromycin,mitronidazol

Omeprazole ,tetracycline,bisthmus,mitronidazole

Omeprazole, Levofloxacin, clarythromycin


56. Pt with dementia and diarrhoea ,on examination has scaly
erythmatous ,dark skin lesion what is the diagnosis?

Zinc deficiency

Iron deficiency

B 12 vit deficiency

Niacin deficiency

57. Pt with autoimmune diseases ,from the following which one


suggest autoimmune hepatitis?

ASMA

ANA

ds DNA AB

AMA

58. Pt known alcoholic present with abd distension,on exam:


ascitis, spenomegaly hepatomegaly ,Inv : high ( Hb . Twbs .plt ) -
AST more than ALT , HBV + ve ,what is the diagnosis?

Alcoholic liver disease

Liver disease due to Hepatitis B

Budd chiari syndrome

59. Pt with recurrent peptic ulcer ,his father has history of


parathyroidectomy to recurrent stones , pt s.Ca normal , PTH
increased what is the cause of his symptoms?

Pheochromocytoma
VIPOMA

Gastrinoma

glucagonoma

60. Asymptomatic pt discovered to have diverticulosis

What is the next step ?

antibiotic

surgury

High fiber diet advice

Follow up

61. Pt presented with diarrhoea diagnosed as gardiasis recurrent


3 times inspite full ttt what investigation to do?

HIV

Immunoglobulins level

Celiac antibodies

62. Pt with feature of malabsorption , low ca and iron ,


antiendomeseal AB –ve what is the diagnosis?

B12 deficiency

Celiac

SBBO

Tropical sprue
63. Most serious complication of malaria is:

ARDS

Cerebral malaria

Black water fever

64. lymphocyte proliferation test used in :

malaria

leishmania

trypansomiasis

falirasis

65. Rabies is diagnosed before death by :

CSF culture

Saliva analysis

corneal ampression test

66. best ttt for lepra reaction type 2 is :

High dose steroid

thalidomide

stop leprosy medication

67. From the following which virus of haemorrhagic fever has


human to human spread:

Hanta virus
Crimean-Congo virus

Marburg virus

Dengue virus

68. Pt diagnosed with brucellosis has suspicion of TB also what is


the best ttt for brucellosis?

Doxacyclin ,rifampicin

Streptomycin, doxacyclin

Streptomycin ,rifampicin

69. Which helminth not need human for spread :???

Taenia

Echinococcus

70. Best tt for multidrug resistant TB?

Moxifloxacine,quinolone ,ethamputol,pyrazinamide?

Pyrazinamide,ethmputol,rifampicin,isoniazide

Pt recurrent malaria what is ttt is added after completion of


standard malaria ttt?

Quinine

Artemether

Primaquine

Mefloquine
71. Sowda disease best diagnosed by?

Diurnal blood film

Skin snip

72. From the following which drug associated with malaria


recrudescence?

Artemether

Mefloquine

Primaquine

73. traveler decide to visit schistosoma endemic area of Sudan


what is best drug to give?

Praziquantel

Artemether

Oxaminquine

Metrifonate

74. Ivermectin one dose is treatment of choice for :

Loa loa

Onchoserciasis

Ascaris

75. Which lead to increase urine PH?

hypoproteinemia
DKA

Klebsiella

76.Pt with liver disease ,how to assess his kidney function?

Urea

creatinine

cystatin C protein

chromogranin B

77.pt with diabetes has proteinuria on lisinopril 20 mg come for


follow up, he is asymptomatic, his BP140/90 , eGFR 42 , urine
with 3000 protien what is the next step?

Add ARBs

Increase the dose of lisinopril

Continue same dose

Stop lisinopril and give amlodepine

78 .which suggest tubular rather than pr renal AKI?

Urine osmolarity >350, urine Na <40

Urine osmolarity <350, urine Na >40

Urine osmolarity >350, urine Na >40

Urine osmolarity <350, urine Na <40

79. Pt presented with pulmonary edema , has decresed GFR , U/S


showed bulky kidneys what is the cause?
HF

Uremia

Amyloidosis

80. Familial hyperoxaluria associated with?

Stone formation

RTA

81. Pt known BPH on ttt , hospitalized for pneumonia then


developed increased urea and creatinine and palpable bladder ,
what is your action ?

CT abdomen

Catheter insertion

Renal biopsy

Abd. U/S

82.HIV pt allergic to sulpha , presented with SOB received ttt


during admission pt developed increased K and cratinine, low
HCO3, high CL ,what is the cause of his hyperkalemia?

Lactic acidosis

Rhabdomyolysis

Drug induced

83. Acid base status in Rhabdomyolysis ?

Hyperkalemia and metabolic alkalosis


Mixed respiratory and metabolic acidosis

Hyperkalemia and metabolic acidosis

Hypokalemia and respiratory acidosis

84. DM pt with sickle cell disease what is the test used for follow
up ?

HBA1c

Fructosamine

85. Female with amenorrhea for ,has prolactin 3000 , TSH high ,
T4 low , what is the diagnosis ?

Macroprolactinemia

Primary hypothyroidism

Pituitary microadenoma

86. Pt with pheochromocytoma first drug to control HTN ive crisis


is ?

Labetalol

Phenoxypenzamine

Doxazocine

87. Diabetic pt with many autoimmune diseases what is the best


investigation?

Anti GAD AB

Anti beta islet cell antibody


88. Obese female with hirsutism inv: high LH,FSH ,morning
cortisol slightly elevated and normal at midnight ,

normal urinary B 17 hydroxyprogestrone , What is the


investigation to confirm the diagnosis?

ACTH level

CT abdomen

Abdominal U/S

89. Young pt with HTN ,no abnormalities on examination,inv:


hypokalemic alkalosis , what is the diagnosis?

Cushing syndrome

Primary adrenal hyperplasia

Fibromuscular dysplasia

pheochromocytoma

90. Pt presented with polyuria polydepsia Underwent head


surgury before many weeks , S.Na high , urine osmolarity low ,
S.Ca 10 , glucose normal ,What is the most likely diagnosis ?

Primary hyperparathyroidism

Dm

Transient Cranial DI

91. In dermatomyositis, which of the following associated with


malignancy?

Skin ulcer
Increased CK

Anti Mi 2 AB

ILD

92. In RA which is correct ?

Suppluxation proximal radioulnar joint is common

Ruptured tendon is common

DIP joint more common affected

Heberden‘s nodes more common

93.Pt with SLE what is most specific ?

ANA

Anti smith

Anti centromer

Anto Ro ss

94. HTNive pt on aspirin presented with joint pain X ray revealed


intra articular calcification , what is the diagnosis?

Gout

Pseudogout

haemoarthrosis

95. Pt with DVT start warfarin , on day 2 developed brown


discoloration in buttock , plt 250 ooo , what is the cause?
HIT

Protein C deficiency

Anti thrombin III dwficiency

96. Pt with dark urine at the morning with h. of DVT , what is best
investigation?

Urinary porphyrin

Acid lysis test

(they didn’t mention flowcytometry)

97. Female with menorrhagia and easy bruises , her sister also has
menorrhagia , has high APTT and normal PT , what is the most
likely diagnosis?

Antiphospholipid

Von willebrand disease

Cristmath disease

98. Asymptomatic pt during routine investigation discovered has


TWBCs 35000, lymphocytes 31000, and has splenomegaly, also

CD 19, CD 20 , CD5 what is the diagnosis?

AML

ALL

CLL

Hodgking lymphoma
99. Sickeler on chronic opiates presented with severe abd. Pain
and jaundice , what is the most likely diagnosis?

Vaso-occlusive crisis

Cholecystitis

Opiates seeking

100. Female using OCP on investigation has very low platelet and
has no bleeding what to do ??

Platelet transfusion

Repeat test using heparinized tube


GREY CASES:

1. pt presented with joint pain , has history of recurrent hospital


admission with abd. pain which resolve after treatment, on
examination has knee joint swelling with effusion and tenderness

Aspiration of joint fluid clear (normal) ANA and RF negative

s. ferritin normal

 What is the diagnosis?

Familial Mediterranean fever

 Give two differential diagnosis ?


Rheumatoid arthritis
Adult onset still disease
 Investigation ?(they need one option)
Genetic testing
Metaraminol provocation test
 Management?
Colchicine
2. young pt presented with fever ,headache ,confusion for 2
days ,pt also presented with headache and fever before
1week received antibiotic but not improved ,pt has no
contact with pt of TB ,heavy smoker, on examination pt
slim and pale ,temp 38,ESR 45 has neck rigidity ,bilateral
brisk reflex and extensor planter reflex, other systems
unremarkable . Inv : CSF : high protein and very low glucose
and high total with lymphocyte predominant ,gram stain and
ZN stain both show no organism
MRI : enlarged ventricles with periventricular enhancement
and high intensity lesions more around temporal horn

 What is working diagnosis?


Tuberculous meningitis complicated by communicating
hydrocephalus

 What is the more specific investigation


PCR for mycobacterium tuberculosis

 Outeline of management?
Corticosteroid
Antituberculous
Transfer for neurosurgical center for consideration of
ventricular drainage

 Outline prognosis?
The mortality of tuberculous meningitis remains as high as
15-30 % .
3. pt presented with joint pain and cough with haemoptysis
Investigation : CXR show pulmonary infilterate , PFT show
restrictive pattern with high transfer factor .

 Give 3 differential diagnosis ?

Systemic lupus erythematous


Good pasture syndrome
Wegener granulomatosis

 Investigation?

ANA ,Anti DsDNA antibody


Anti GBM antibody
ANCA

 If pt has renal impairment how to manage?


IV methylprednisolone + cyclophosphamide
4. pt known of autoimmune hypoadrenalism on
corticosteroid and fludrocortison presented complain of
gaining weight and sleepiness on examination obese ,
bradycardic ,…

 What is the diagnosis?


Autoimmune hypothyroidism
 Give 3 investigation?
TFT:T4,T3,TSH
Thyroid antibodies
Thyroid US/ radioisotope scan
 In addition to his chronic illness what is the complete
diagnosis?
Autoimmune polyglandular syndrome type 2
 What is the treatment you have to add?
Levothyroxin
5. young male recently travel to Halfa presented with fever 2
days and bilateral lower limb weakness . On examination LL
hypertonia , hyperreflexia , loss of sensation till umbilicus ?
pt has eosinophilia
 What is the diagnosis?
Transverse myelitis due to schistosomiasis
 Give one differential diagnosis?
Potts disease of the spine
 Investigation?
MRI spine
Investigation for schistosoma (rectal snip)
 Outlines of management?
IV methylprednesolone then oral prednisolone
Praziquantel .
OSCE:

1. Pt with Atrial fibrillation

 What is the diagnosis?


Lingual artery thrombosis due to Atrial fibrillation

2.MRI brain of pt from South Sudan present with convulsion

 What is the diagnosis?


Neurocystocercosis
 What is the management?
praziquantel or albendazole + steroid
3.Describe ?

Extensive juxta-articular periostitis in DIPJs , (pencil-in-cup)


 What is the diagnosis?
Psoriatic arthritis

4.

 What is the diagnosis?


Charcot joint (neuropathic arthritis)
 What is the diagnosis if the pt has meiosis?

Neuro-Syphilis (tabes dorsalis)

5. Blood film of pt with confusion and renal impairment


describe ?

Anisocytosis ,reticulocytosis and fragmented RBCs (schisocytes)

 What is the haematological diagnosis?


Microangiopathic haemolytic anemia due to (TTP)
6. CT abdomen

 What is the finding?


Calcification of spleen and mesenteric lymph nodes
 What is the differential diagnosis?
Histoplasmosis, TB, sarcoidosis

7. Pt presented with fever and murmur ,

 What is the finding?


Erythema marginatum
 What is the diagnosis?
Rheumatic fever
 Mention other signs?
Subcutaneous nodule
Sydenham chorea
Migratory arthritis

8.

 What is the diagnosis?


Rt lung with total homogenous opacity (white out )
 What is the causes?
Total lung collapse
Pleural effusion
Malignancy
9. pt with hepatitis C infection

 What is the diagnosis?


Porphyria cutanea tarda

10. CT chest

mention the signs which can seen in this pt?


Raised JVP
kussmaul sign
pulsus paradoxus
(pericardial calssification _ constrictive pericarditis)
11.

 What is the diagnosis?


Ventricular bigeminy
 What is the cause?
IHD

12. Pt has developed this skin lesion 3 month after febrile


illness

 What is the diagnosis?


PKDL
 Management?
Sodium stibogluconate+ paromomycin
Amphotercine B
13.

 Give anatomical diagnosis?


varicella zoster (shingle) in T10 distribution
 What is the cause ?
HZV infection
14.

 this worms is removed after incision of pt skin


what is the diagnosis?
Myasis
 What is the ttt:
Petroleum cause asphyxiation of maggot facilitates
removal by tweezers

surgury
15.

 What is the diagnosis?


Vitreous haemorrhage
 What are the causes?
Proliferative diabetic neuropathy
Accelerated hypertension
Trauma

16.
 What is the diagnosis?
Diverticulosis
 Mention 2 complication?
Bleeding
Infection
Obstruction

17. Pt with normal TFT

 What is the diagnosis?


Pretibial myxedema
 What is the other signs of the underlying disease?
Graves ophthalmopathy
Graves dermopathy
18.

 What is the finding?


Gynaecnomastia
 Mention 2 neoplastic causes?
Testicular cancer
Estrogen secreting adrenal tumor

19.

(Actually the arrow was more superior near the superior edge of
scapula)
 What is the muscle?
Serratus anterior
 What is the nerve supply?
Long thoracic nerve (C5_C7)

20.

 Pt with epilepsy, What is the finding?


Strawberry haemangioma in distribution of
ophthalmic division of the trigeminal nerve

Gum hyperplasia due to phenytoin


 What is skull x.ray shows?

Cerebral calssification
AVM (carotid artery)
 What is the disease ?
Sturge Weber syndrome

‫مع تمنياتي لكم بدوام التوفيق وال تنسونا من صالح الدعاء‬

‫سبحانك اللهم وبحمدك‬


‫اشهد أن ال اله اال انت‬
‫استغفرك واتوب اليك‬
RECALL BOF 5TH OCTOBER 2020

D. Hyder Ishag Adam


1-COVID 19 with which of the following has higher mortality :

a- Hypertention
b- Diabetes Millites
c- Ischaemic heart disease

2-COVID19, steroid (dexamethasone) has prognostic benefit in those following patients:

a- In ICU ventilated patients


b- Asymptomatic

3-Regarding to prevalence of COVID19:

a- 50% of population have symptoms


b- 5% of population have symptoms

4-which one of the following viral haemorrhagic fever transmitted by arthropode except

a- Dengue fever
b- Ebola
c- Yellow fever
d- Rift valley fever

5- Patient with mantoux test 7mm induration, in which statement we can anti TB:

a- Comorbid with HIV

6-What is the genetic factor that has preventive mechanism against plasmodium vivax

a-sickle cell trait

7-Malaria has causal and suppressive prophylaxis,which one of the the following is causal
prophylaxis

a-doxacycline

b- atovaquone/proguanine(malerone)
c- quinine
e- mefloquine
8-characteristic clinical feature of cavernous sinus thrombosis

a- Early papilloedema
b- Hyperaesthesia on frontal area

9-First line treatment of myoclonus

a- Sodium valporoate
b- Carbamazepine
c- Ethuximide
d- Levetiracetam

10- patient presented with recurrent blackout with immediate recovery and normal
investigations(ECG-CT brain-ELECTOLYTES)

a- Cardiac arythmias
b- Vasovagal attack
c- Addison disease

11-patient with crohn,s disease has neumoturea ,what is the best investigation to reach the
diagnosis :

a- CT abdomen
b- Barium swallow
c- Enterograhy
d- Sigmoidoscopy

12- Female has osteoporosis and received Hormone replacement therapy(tamoxifene) and
developed deep vein thrombosis then stop tamoxifene.which one of the following is
contraindicated :

a- Calcitonin
b- Alendronate
c- Etodronate
d- Raloxifene

13-orbicularis oculi ---lagophthalmos :caused by :

a- onchocerciasis
b- loa loa
c- wuchereria banchorufti
14- patient developed cholestatic jaundice after liver transplantation

a- common duct stricture


b- Ca pancreas

15-Nurse has exposed to HIV and has been given HIV prophylaxis ; follow up will be as
following

a- 6 weeks _12weeks and 6 months


b- 4weeks 6weeks and 8 months

16-Treatment of strongloides stercolaris

a- Albendazole
b- Praziquentil

17- Which one of the following ABGs is of stable COPD

a- PH7.35 pco2 po2

18- patient with eosinophilic esophagitis has complained of

a- Heart burn
b- Dysphagia
c- Wheezy chest
d- Skin itching

19-patient with chronic diarrhea and weight loss 5Kg for 4months and all investigations
including D-xylose are normal what is the diagnosis

a- coeliac disease
b- IBD
c- Whipple disease
d- Tropical sprue

20- patient received triple eradication of H.pylori .what is the most sensitive investigation for
follow up

a- Disappearance of symptoms
b- Urea breath test
c- Negative stool for H.pylori

21-patient with burn .CVP is normal ,deteriorating kidney function due to


a- Acute tubular necrosis
b- Hypovolaemia
c- Sepsis

22- 18years old presented with hypokalaemic hypocholoraemic metabolic alkalosis with
normal blood pressure investigation hypercalciuria and hypocalcaemia . what is the diagnosis

a- Barter syndrome
b- Gittleman syndrome
c- Renal tubular acidosis type 1
d- Renal tubular acidosis type 2

23- A lady is a known to have rheumatoid arthritis ,hyperlipidaemia, in postpartum period


,Rheumatoid arthritis is controlled with methotrexate ,hydroxychloroquinine,aspirin and
statin.Recently added NSAID conseguently RFT deteriorate then NSAID was stopped .Which
of the drug should be stopped :

a- Methotrexate
b- HCQ
c- Aspirin
d- Statin

24- patient with crohn,s disease received infliximab and developed deterioration of vision
and fundoscopy revealed haemorrhage along retinal artery .What is the treatment of this
condition that causing visual disturbance :

a- Ganciclovir
b- Photocoagulation

25- ASD with :

a- Ventricular tachycardia
b- Complete heart block
c- Atrial fibrillation

26- Scenario of typical presentation of multiple myeloma . what is the cause of renal
impairment:

a- Hypercalcaemia
b- Amyloidosis

27- What is the feature of severe aortic stenosis:


a- Ejection systolic murmur radiated to the axilla
b- S4

28- Patient with collapsing pulse and feature of pulmonary hypertention what is the
diagnosis:

a- Patent Ductus Arteriosus


b- Aortic stenosis
c- Mitral stenosis

29- Pt with clinical presentation of brucellosis and has renal impairment .what is the safer
regimen:

a- Doxacycline and streptomycin


b- Cotrimoxazole and doxacycline
c- Doxacycline and rifampicin

30- patient with lymphoadenopathy and blast cells on peripheral blood picture .what is the
best investigation:

a- Lymph node biopsy


b- Immunophenotyping
c- Bone marraw aspiration

31- Patient with the feature of IBD and has dilated biliary system on ultrasound . What is the
best next investigation :

a- MRCP
b- ERCP
c- CT abdomen

32- Patient with HIV and presented with CNS symptoms. CT brain showed homogenous
enhancing and given likely diagnosis .what is the treatment:

a- Radiotherapy
b- Surgery
c- Chemotherapy
d- AntiTB

33- What is the feature of tuberculous pericarditis :

a- Absent x descent
b- Absent v descent
c- Steep y descent

34- Patient with features of multiple sclerosis ;what is investigation

a- Visual evoked potential

35- pt with clinical presentation of multiple sclerosis what is most sensitive investigation

a. MRI brain
b. Lumber puncture
c. Nerve conduction study

36- Feature of lateral medullary syndrome what is the diagnosis

a- Left lateral medullary infarction


b- Right lateral medullary infarction

37- Patient with DM type 2 with the feature of nephritic syndrome and fundoscopy is normal
. what is the cause of nephropathy

a- Diabetic nephropathy
b- Membranoproliferative glomerulonephritis
c- Focal segmental glomerulonephritis
d- Membranous glomerulonephritis

38- Female in 34 weeks gestation presented with headache and normal BP . low HB low
platelet and reticulocytes 6%(MAHA) given likely diagnosis what is the best management

a- Plasma exchang
b- IV immmunoglobulins
c- Prednisolone
d- Magnesium sulphate

39- Pregnant lady presented with severe headache ,impaired liver functions and no
convulsion highBP .what is the treatment

a- MGSO4
b- Termination of the pregnancy

40- Feature of the polycythaemia rubra vera . what is the treatment

a- Venesection with aspirin


b- Venesection with hydroxycarbamide
41- Female teacher HBV positive ,she is worry and wish to know the state of viral replication ;
what is the best investigation

a- HBV DNA
b- HbeAg
c- HbcAb

42- Patient presented with haematemesis with low blood pressure after resuscitation the
patient became stable with normal blood pressure but still there is minimal bleeding .what is
the next best step:

a- OGD with banding


b- Blackmore sengstaken tube
c- Terlipressin

43-treatment of HCV

a- Tenofovir and ribavirin


b- Entacavir
c- Pegillated interferon alfa
d- Sofosbuvir/velpatasvir400mg/100mg

44- Clinical feature of hypothyroidism(which is the most obvious one) and mild raised
prolactin .what is the appropriate management

a- Give thyroxine for hypothyroidism


b- Dopamine agonist

45- 72 years old man presented with stroke .brain scan showed bleeding .

a- Amyloid angiopathy
b- Hypertention

46- Phosphodiestrase inhibitor 5(PD5),non responsive in case of :

a- Angiotensin converting enzyme (ACE)


b- Take with meal

47- which of the following is not induced by drug

a- Bronchiectasis
b- Pulmonary fibrosis
c- Pleural effusion
d- Pulmonary haemorrhage

48-Patient with inferior myocardial infarction developed pleuritic chest pain after 2days
(pericardial rub) . what is treatment

a- High aspirin dose


b- colchicine
c- Steroid

49- Patient with myocardial infarction received streptokinase and ECG received after lysis ST
elevation .5mm .what is the next appropriate step

a- Relysis
b- Percutanous Coronary Intervention
c- Tenectaplase
d- Alteplase

50- Patient with lipodystrophy and haematuria and proteinuria &low C3 .what is the
diagnosis

a- Membranous GN
b- Membranoproliferative GN
c- FSGN

51- Patient with systemic sclerosis and renal crisis given captopril and still RFT is impaired
.what is the next step

a- Stop captopril
b- Switch captopril to amlodipine
c- Continue with captoril and add prednisolone

52- Patient with dry eyes and mouth ,arthritis and erythematous rash

SSA,SSB,RF,ANA are positive Labda/kappa ratio is normal.what is the diagnosis

a- Sjogren,s syndrome
b- Mixed connective tissue disease
c- Rheumatoid arthritis with Sjogren
d- Systemic lupus Erythematosus

53- Patient with asymmetrical polyarthritis and mainly distal interphalangeal joint(DIP).what
is the diagnosis
a- Rheumatoid Arthritis
b- Psoriatic Arthritis

54- Patient with feature of posterior column affection(impaired vibration and


proprioception) and pupil reacted with only with to accommodation what is the diagnosis

a- Tertiary syphilis
b- Subacute combined degeneration of spinal cord

55- Patient presented with right lumbar mass ,hamaturia , abdominal pain and recurrent
urinary tract infection .what is best investigation to reach the diagnosis

a- CT abdomen
b- Abdominal us
c- Renal biopsy

56- Prosthetic valve endocarditis .what is the best regimen

a- Vancomycin and rifampicin


b- Benzylpenicillin for long time

57- Patient with recurrent ear and nasal bridge pain ,saddle nose CXR is normal .what is the
diagnosis

a- Wegener granulomatosis
b- Relapsing polychondritis

58- Patient with DM on metformin and NPH, his FBS 80 mg/dl HbA1c 7.5% and patient
experiences hypoglycaemic attack at the morning. What is the the next appropriate step:

a- Basal insulin 0.2 u/kg


b- Glargine insulin (lantus)
c- Premixed insulin 50 U once aday

59- Patient with goiter and has upper airway obstruction(stridor).what is the best test to
monitor his condition

a- Peak expiratory flow rate


b- Flow volume loop

60- pregnant lady with thyrotoxicosis and had relapse .what is the best treatment

a- Methimazole /carbimazole
b- Propylthiouracil
c- Radioiodine therapy

61- Patient with head trauma and features of hypopituitarism (low testosterone,low
aldosterone,high TSH)received corticosteroid and also has feature of hypothyroidism
(irregular cycle,constipation and weight gain).what is the next step

a- Increase the dose of corticosteroid


b- Give levothroxine

62- patient has atrial fibrillation presented with malena and taking warfarin so he received
vitamin K. what is an another management should be given

a- Fresh frozen plasma


b- Cryoprecipitate

63- patient on Rivaxaban and preparing for colonoscopy.Rivaxaban should be stopped before
procedure by

a- 2days
b- 3days
c- 7days

64- NOAC is preferable because of

a- No need for dose adjustment

65- Adjuvant treatment in severe malaria beside antimalaria

a- Gentamycin
b- Glandamycin
c- Levofloxacin
d- Amikacin

66- Patient presented with carcinoid tumour and CT abdomen showed left adrenal mass
1.6cm .what is the next step

a- CT guided biopsy
b- MIBG

67- Patient received medication including paracetamol and developed cholestatic jaundice
.which one of the following drugs is responsible

a- Paracetamol
b- Coamoxiclav
c- Phenytoin

68- Patient presented with productive cough ,fever,weight,esinophlia and CXR revealed
bilateral hilar shadowing .what is the diagnosis

a- Strongloides stercolaris
b- Allergic bronchopulmonary aspirogillosis

69- Patient with COPD on SAMA&SABA,referred to you complaining of dyspnoea.what is the


next step in management

a- Inhaled corticosteroid
b- LABA
c- LAMA
d- Non invasive ventilation

70- patient presented with acute attack of COPD&received medication&clinically patient


improved even SO2 94% but chest radiograph is worse than before at initial
presentation.what is the next appropriate step

a- CT chest
b- Change antibiotic
c- CXR

71- Patient with brainstem stroke and has dysphagia

a- Ambiguous nucleus

72- 30years old young female presented with behavioral changes and memory problem and
also she has fever.what is the best investigation

a- MRI brain
b- Lumber puncture

73- patient with haemolytic feature in CBC &has recurrent menorrhagia,Hb8g/dl,low


platelet,high LDH,high ferritin,low TIBC,MCV,HB electrophoresis show Hba1,HbF.what is the
diagnosis

a- Sickle cell trait


b- Iron deficiency anaemia
c- Thalassaemia minor
74-Patient with liver cirrhosis and tuberculosis started anti TB then liver
enzymes(transaminases)raised to double what is the plan of action

a- Stop antiTB
b- Stop INH&rifampicin and continue with ethambutol and pyrizanimide
c- Continue antiTB with the follow up after one month
d- Continue only with INH&RH

75- Patient diagnosed with Bell,s palsy and received prednisolone and acyclovir ,after three
months she came with left side of mouth twitching with blinking her eyes .what is the
diagnosis

a- Focal seizure
b- Facial nerve regeneration

76- Patient with autosomal dominant polycystic kidney disease ,worry about inheritance of
disease to his two children.what is the percentage

a- 0%
b- 25%
c- 50%
d- 75%

77- Patient complains of dyspnoea at night and received drug of weight reduction PH Po2
PCO2. What is the diagnosis

a- Obesity
b- Pulmonary hypertention

78- Patient underwent parathyrodectomy due to primary hyperparathyroidism ,then serum


calcium became low 7mg/dl& given calcium and vitamin D but not responding.what is the
diagnosis

a- Hungery bone disease


b- Secondary hypoparathyroidism

79- Risk factor of multidrug resistant TB

a- Comorbid with HIV


GREY CASES 6th OCTOBER 2020 RECALL

D. hyder Ishag Adam

CASE NO (1)

Building constructor complains of fever ,nausea and vomiting for 3


weeks.

O/E:Abdomen,chest and CNS are normal

Investigations:

Deranged liver functions mainly hepatocellular features and renal


functions are also impaired.

Hb 11g/dl TWBC 11.2 Platelet 100

BFFM &ICT for malaria &Widal test for entrica &brucella all are
negative

1_What is most likely diagnosis ?


Leptospirosis

2-Mention one differentional diagnosis ?


Viral haemorrhagic fever

3_Mention one sensitive investigation?


Blood &CSF culture or PCR for leptospira interogans

4_Mention specific treatment ?


Doxacycline or penicillin
CASE NO (2)

Young female from southern Kordofan presented with fever associated


with sweating and no rigors and also complained of palpitation,
exertional shortness of breath, dark urine and weight loss .She has
history of recurrent malaria

O/E:

The pt looks unwell pale not jaundiced

There is soft systolic murmur at apex

Mild splenomegally

Hb 9.5 G/dl TWBC & PLT are normal

ESR70mm/h reticulocyte count 1.5%

1-Mention two differential diagnoses ?


Infective endocarditis

Systemic lupus erythematosus

2- Mention three another clinical signs?


Splinter haemorrhage

Roth,s spot

Janeway lesion

3-Mention three investigations ?


Blood Culture

Echocardiography

Antidouble stranded DNA


CASE NO(3)

Elder man known diabetic on oral hypoglycaemic drugs and on good


control .he presented with unsteadiness,wide base gait,nystagmus and
vertigo. He is heavy smoker

O/E:

Disdiadodochokinesia,proprioception and diminished reflexes .

Planter reflex downgoing and no weakness. BMI 18.5

RBG 135 mg/dl HbA1c 6.5% MRI brain unremarkable

1-What is your full clinical diagnosis?


Paraneoplastic syndrome with cerebellar syndrome

2-What is underlying aetiology ?


Small cell lung cancer

3-Mention two relevant investigations?


Bronchoscopy with biopsy

HRCT chest

4-Mention two further approaches to reach the diagnosis?


Syphilis serology and serum B12 and folate

Anti HU
CASE NO (4)

Elderly man presented to A&E with acute centeral chest pain radiated
to the neck and back after an argument with his wife .

O/E: PR 120 BP 220/160

ECG was done revealed only tachycardia

His chest pain was relieved after given an analgesia but unfortunately
his chest pain recurred again after awhile .ECG was done again which
revealed ST elevation in V1---------V4

There is right hand pallor.BP190/100

1-What is your full clinical diagnosis ?


Aortic Dissection type (A) complicating with anteroseptal myocardial infarction and right hand
ischaemia

2-Mention three investigations ?


CT chest with contrast or MRI

Echocardiography

3-what is the appropriate management?


Control blood with betablocker(eg.labetalol iv)

Surgical correction
CASE NO (5)

Patient with HIV developed PCP and received septrin and developed
hypersentivity then switch to clindamycin and primiguine, his RFT is
impaired s.creatinine 1.8.Hb 10 plt 120

Then added dapsone for prophylaxis of PCP ,patient became dyspnic


So2 85% PH 7.32 HCO3 19 PCo2 33mmHg PO2 normal, cyanosed and
jaundiced .

Hb 7g/dl plt 100 retics count 10%

1-What is the most likely diagnosis?


dapsone induced methaemoglobinaemia ,possibly patient may have G6PD deficiency

2-What is the confirmatory test?


G6PD assay and serum level of methaemoglobulin

3-what is an immediate intervention?


Stop dapsone and primiguine(sulfur containing drugs)

Hyperbaric oxygen

Exchange transfusion

4-What is the treatment for long term?


Avoidance of sulfur containing drugs

Neublised pentamidine as prophylaxis of pcp

HAART continuation
RECALL OSCE 6th October 2020

D.Hyder Ishag Adam

1-
koilonychia and pallor

Ascaris lumbricoids
2-Picture

neurofibromas and café au lait on back

a-what is the diagnosis ?

Neurofibromatosis type 1

b-mention two causes of hypertention ?


-phaeochromatosis - fibromascular renal artery stenosis

3-picture

a-what is the diagnosis ?

cutanous larva migrans

b-mention two drugs ?

ivermectin and albendazole


4-ECG of patient with chronic cough

a-What is an ECG diagnosis ?

b-mention two causes of chronic cough ?

5-CXR :Aortograph

a-discribe :narrowing of the descending aorta(coarctation of the aorta)

b-mention three clinical signs?

a- Hypertention
b- Radiofemoral Delay
c- Rib notching

6-Picture

a-discribe the lesion ?

Two big frontal well demarcated ulcers

b-mention three causes?

a- Syphilis
b- Cutanous leshmaniasis
c- leprosy

7-ECG

a- What is an ECG diagnosis?


Inferior myocardial infarction

b-Patient developed hypotention ---what are causes of deterioration of patient within


48hours?

-Right Ventricular infarction

-Complete heart block


8-CXR

a-What are the findings?

Cardiomegally –upper lobe diversion –left atrial enlargement

b-what is the diagnosis ?

Left ventricular failure

9-CT CHEST
10-Picture shows :

a-What does the picture show?

Atrophic glossitis

b-Mention two causes of this sign?

Coeliac disease

Pernicious anaemia
11-CT ABDOMEN

a-what does it show ?

hydatid cyst with daughter in the left lobe of the liver

b-Mention two modalities of the management ?

-Albendazole for 1-3months

_PAIR procedure(puncture-aspiration-injection-reaspiration)
12-Blood film

a-what does it show ?

spherocytes-target cells –elleptocytes

b-What is the cause of abdominal pain?

Pigment gallstone causing cholecystitis

13-CXR

a-What are the findings?

Bilateral hilar shadowing-reticulonodular shadowing and cavity

b-Mention two causes?

-Tuberculosis

-Sarcoidosis
14-Two pictures

a- what is the diagnosis?


Leperomatous leprosy

b-Mention two drugs ?

-Dapsone - clofizamine
15-Pictures show

conjunctival haemorrhage and jaundice


a-Name the picture (b)-fly?

Aedes egypti

b-Mention three differential diagnoses ?

chikungunya

Yellow fever

Dengue fever

16-picture

a-what are the findings?

shows upper and lower lips pigmentation with blue and red colour involving teeth and
gum

b-what is the diagnosis ?

AML??
17-Picture

a-What does the picture show ?

Right scapular swinging

b-What is the diagnosis ?

long thoracic nerve palsy(serratus anterior weakness)


18-picture

a-What does the picture show?

Deviation of the uvula to the right side

b-what is the diagnosis ?

9th cranial nerve palsy


19-picture

a-what does picture show?

Right foot showing fourth toe gangrene with big toe vasculitic changes

b- Mention two causes ?


Diabetes Mellites-----Thromboangitis obliterans
1. A farmer presented with haematemesis he was brownish 6 units of
blood BP 120/ 80, INR 1-6 , What do you want to do next
a- Platlets
b- Nothing
c- Fresh frozen plasma
d- Cryopcipitate

2. A patient with V.leishmamises treated with sfiboglucanate for 30days


but still febrile what will you give
a- Repeat sfiboglucanate coarse
b- Amphoterien B(liposomal)
c- Amautadiue
d- Pentamedine

3. Patient present with right loin mass and haematuria CXR bilateral apical
scaring, What is the diagnosis
a-Renal TB
b-Renal cell CA
c-Good Posteur
d-PAN

4. A farmer presented with fever for 6 weeks temperature 38, tip of spleen
palpable blood culture –ve, urine has RBC, What is the diagnosis
a-Q fever endocardites
b-Renal TB
c-Renal cell CA
d-glumerulonephritis

5. Patient with HIVpresented with confusion and fever LP protein 0.8,


sugar low, What is the next step
a- Acyclovir
b- Ganciclovir
c- Pyrimebliamin and sulfadeazinc
d- amphotericin
6. An obese lady presented with S.O.B she used appitite supressent to
reduce weight Echo RT ventricular hypertrophy, What is the diagnosis
a- primary pulmonary hypertension
b- secondary pulmonary hypertension
c- pulmonary fibrosis

7. Patient was found to have retroperitoneal sarco ,What is likely to be


found
a- hypocalcemia
b- hypercalcemia
c- hypoglycemia

8. Elderly patient with CKD on HD presented with fracture in both


shoulders, What is the cause
a- Amuloidosis, A
b- B- microglobulin amiploiet
c- Hyperparathyroidism
d- Myeloma

9. Patient with sickle cell disease presented with a vascular necrosis in the
hip and fever, What treatment will u give
a- Ceftriaxone
b- Vancomycn
c- Ceftriaxone + Vancomycn
d- Surgery
e- Anti TB

10. Patient with MGUS how can you differentiate from multiple
myeloma
a- IGG 2gm
b- Bencezonce in urine
c- Plasma cells in bone marrow 7%

11. Patient presented with convulsion and left side weakness CT


showed temporal heamorage MRVshowed lateral sinus thrombosis,
what is the treatment
a- Warfarin
b- Heparine
c- Aspirin
d- Nimodipine
e- Streptokinase

12. Patient Presented with headache and blurring of vision glucose


are normal ESR 73 Which of the following is correct
a- Temporal artery biopsy before any medical intervention
b- Methyl prednisolone administration
c- Cyclophosphamide
d- CT brain

13. Young female has frequent attack of unilateral headache lasting


many hours she used a lot of analgesic without any improvement, What
is management steps
a- Sumatriptan
b- MRI brain
c- Stop analgesia, analgesic rebound headache
d- Propranolol

14. Patient presented with chest pain ECGshowed ST in inferior


leads given streptokinase 6hours later she has severe chest pain, ECG
43% reduction in ST, PR 52, BP 130/ 80, What is the next step
a- I.Vnitrates
b- Repeat thrombolysis
c- Send patient for PCI
d- Give morphine
e- Give B. blockers
15. 60 years male presented with severe headache lasting 3 to 4 hrs
associated with fearing and congestion, What treatment can be given to
prevent attack
a- High flow oxygen
b- Tryptan
c- Phenytoin
d- Verapanicl

16. Patient is known to have Parkinson disease well controlled on


levedopa presented with tremer, What is the treatment
a- Pramipixol
b- Benzhexol
c- Cabergolin
d- Amantadine

17. Patient who is Known smoker presented with headache and


conjuctival congestion examination is normal platlets WBC raised HB
18mg/ dl What is the next step
a- RBG
b- Red cell mass
c- Erythropoietin
d- Bone marrow

18. Elderly patient presented with dysphagia facial numbness


examination, Horner sighn and loss of pain few pain in face and lateral
lumb where is the lesion
a- Superior cerebeller artery occlusion
b- Vertibral artery dissection
c- Subarachnoid hemorage
d- Middle cerebral artery occlusion
e- Carotid artery aneurysim

19. Hypertensive patient presented with joint pain in 1 st metacarpal


joint on examination there is cellulites on 1st metacarpal joint, what is
the next step
a- Revise anti hypertensive medication
b- Give allupurinol
c- Hand Xray
d- Check serum uric acid

20. Patient with family history of gout presented with joint pain in
the left ankle on examination swelling left ankle, what is the next step
a- Check serum uric acid because is invariable high
b- Joint aspiration
c- Allupurinol
d- Urine for urate
e- Advice to reduce red meat

21. Alcoholic man presented with fever and confusion CSF showed
pleocytosis and reduced sugar, raised proteins, What is the diagnosis
a- TB meningitis
b- Nocardiases
c- JCvirus infection
d- Cryptococcal meningitis
e- Lesteria infection

22. Pyridoxin mechanism of action


a- Convert homocysteinurea to cystathionin
b- Utilize phosphate group

23. Leflunamide
a- Same action as sulphasalazine and MTX
b- Short duration
c- Relative contraindication in pregnancy nursing women and liver tissue

24. Apatient presented with chalculous cholecystitis he has history of


operation in his abdomen what is diagnosis
a- Ulcerative colitis
b- Pernicious anemia
c- Crohnis disease

25. Patient presented with pain and swelling in his muscle, What is
the cause
A- Flariases
B- Trichenellosis
C- Hook worm
D- Cystecercocis

26. Patient with bronchial asthma complain of fever shortness of


breath given eythromysene developed nausea and vomiting, she was on
inhaler and saluetrol + thuophyllin, whats the diagnosis
a- Asthma attack
b- Chest infection
c- Theophyllin toxisity
27. Student presented with SOB hemoptysis CXRshowed patchy
shadowing lung function test restrictive with high KCO& DLCO, What is
the diagnosis
a- Asthma
b- Sarcoidosis
c- Emphysema
d- Idiopathic pulmonary hemosidrosis
e- Pulmonary hemorrhage

28. Patient with tricyclic overdose, what is the treatment


a- Oxygen
b- HCO3
c- Dialysis
d- Lavage

29. Patient with shezophrania presented with rigidity and fever, what
is the treatment
a- Dantrolene
b- Diazepam
c- Phenytoin
d- haloperidole

30. Patient with history of dysentery presented with echomosis CBC


platelets 45 fragmented cell, what is the diagnosis
a- HUS
b- TTP
c- Renal failure
d- DIC

31. Patient with COPD what will you likely find


a- PH 7.5, PO2-14, PCO2-3, HCO3-20
b- PH 7.4, PO2-12, PCO2-6, HCO3-25
c- PH 7.3, PO2-10,PCO2-6,HCO3-36

32. Patient known diabetic for 20years he is on insulin and good


control 6weeks ago he developed sudden edema urine showed 24hrs
protein 6gm, how are you going to manage
a- ARB
b- ACED
c- Renal biopsy
d- Abdominal ultra sound

33. Female presented with confusion and jaundice she is known to


have auto immune hepatitis on examination she is febrile confused
deeply jaundice and there is massive ascites, What is the next step in
management
a- Abdominal CT
b- Blood culture
c- Urine analysis
d- TWBC
e- Peritoneal tap

34. HIVpatient presented with blurring of vision on examination


funduscopy there is hemorrhage following retinal artery branches, what
is the diagnosis
a- Herpes zoster infection
b- Toxoplasmosis
c- CMVinfection
d- Cryptococus

35. Obese lady on routine investigations found to have increased liver


enzymes, what is the next appropriate investigation
a- CT abdomen
b- Liver biopsy
c- Abdominal ultrasound
d- Upper GI indoscopy
e- Hepatitis screening

36. Patient with Hodgchen lymphoma presented with SVCO, what is


your management
a- Diuretics
b- Radiotherapy
c- Chemotherapy
d- Surgery

37. Apatient known to have antiphospholipid syndrome presented


with left leg DVT started on worfarin, what is the appropriate next step
a- Worfarin for life with INR 2-3
b- Worfarin for life with INR3-4
c- Worfarin for 3months
d- Worfarin for 6months with INR2-3 then aspirin for life

38. Patient with lateral sinus thrombosis, What is the treatment


a- Worfarin
b- Heparin
c- Aspirin
d- streptokinase

39. Nurse while giving injection to patient with hepatitis B injected


herself, What screening test do you request
a- HBSA+ anticore IGM
b- HB e AG
c- HB s AB
d- HIVtest
40. Diabetic patient is discovered recently to have hypertension
started on ACE, What is the target for BP control
a- 110/ 70
b- 125/ 75 if had microalbumin
c- 130/ 80
d- <140/ 80

41. Apatient who is known to have angina he is on maximal tolerated


doses of nitrates and beta blockers, what drug is likely to benefit his
cardiac prognosis
a- ARBS
b- Noridihidro
c- ACEIS
d- Statins

42. Apatient who is known case of COPDon oral steroid started


5days ago and short long acting presented with worsening shortness of
breath ABGPO2-11, PCO2-6, PH 7.25,What further investigation(he is
on O2 28%)
a- Increase O2
b- Decrease O2
c- IVsteroid
d- Mechanical ventilation
e- NIPPV

43. Young female asymptomatic found to have RBG131ml/ dl,


FBG131ml/ dl, BMI 42, What is appropriate next to do
a- GTT
b- Repeat FBG
c- Life style modification
d- Metformin
e- Glemibride

44. Apatient with normal thyroid function found to have thyroid


nodule, What is the appropriate next step
a- Radioiodine Isotope
b- FNA
c- Surgery
d- Observation

45. Apatient who is started on H.D recently after first H.D section he
becomes confused, what is the most likely explanation
a- Hypercalcemia
b- Osmolar shift
c- Infection

46. Adiabetic and hypertensive patient with known history of


atherosclerosis US renal shape is normal but right kidney 7cm left
kidney 8cm, what most likely explanation
a- Chronic glomerulonephritis
b- Ischemia
c- Pyelonephrites
d- Diabetic nephropathy
e- Hypertensive nephropathy

47. Apatient presented with paraplegia and kyphus1 disease, What


was diagnosed and vertebral compression fracture was confirmed after
2months on treatment the patient power improved grade4 but the
kyphus still, what could explain the mechanism of neuron lesion
a- Compression vertebral fracture
b- Empyema
c- Paravertebral abscess
d- Dscitis
48. Apatient is known IDDM on insulin developed nightmare and his
morning glucose between 200-300, what is the next appropriate step
a- Increase the insulin dose
b- Decrease the insulin dose
c- Off metformin
d- Start glimibride

49. Patient with prephral neuropathy and painful skin rash, What is
diagnosis
a- Lepra type2 reaction

50. Patient with typhoid fever received treatment improbably


presented after 18days, What likely complications
a- ARF
b- Myocarditis
c- ARDS

51. Patient presented with painful proximal myopathy there is rash at


the back involving upper arm and shoulder Na 120ml, What is
underlying aetiology
a- Hypothyrodisn
b- Poly mayalgia rheumatic
c- Cryoglobiniemia
d- Dermatomyositis

52. Patient presented with high blood pressure on examination


hyperpigmented of buccal mucosa his Kis low, HCO3 is high, What is the
diagnosis
a- Addison’ s disease
b- Nelson syndrome
c- Conns disease
d- Basal cell carcinoma
53. Elderly patient with painful fingers on examination has
spleenomegaly, lymphadenopathy, What is the next step
a- Electrophorisis
b- Bone marrow
c- Urine analysis

54. 45 years female has early menoupause started HRT but


discontinued because of intolerance presented with irritability, What is
the next action
a- TSH
b- Repeat HRT
c- Reassurance
d- Antithyroid treatment

55. Apatient represented with SOB on mild exertion on examination


S1 mid diastolic murmur and systolic murmur echo right MS + minor
MR, what is your next step
a- ACEI
b- Valvotomy
c- Mitral valve replacement
d- Nitrates
e- Frusemide
56. Patient not known to have cardiac disease presented with AF,
What is the best for rate control
a- Verapamil
b- Digoxin
c- Atenolol
d- Flecainide
e- Amiodarone

Alcoholic presented with confusion OGD showed oesophageal varices, what


treatment to reduce portal pressure and prevent hematemesis

a- Propranolol
b- Sclerotheropy
c- Band ligation
d- PPI

57. Patient with lupus nephrites stage 4, what is the best treatment?
a- Steroid
b- Cyclophosphanamide
c- Cyclophosphanamide + steroid
d- Hemodialysis

58. Apatient who have HBVinfection viral load 50,000 liver biopsy
bridging fibrosis, what is the treatment
a- Peg INF
b- Peg INF +lamivudine
c- lamivudine
d- Peg INF + ribavirin

59. Patient presented with chest pain radiating to the back on


examination pulse un equal diastolic murmur in the left sternum BP
200/ 95 what is the likely the management
a- ACEI
b- Heparin
c- Streptokinase
d- labetalol

60. young girl with recurrent chest infection and halitosis developed
convultions what is the next step
a- navalproat
b- no treatment
c- carbamazepine
d- phenytoin
e- CT brain
61. Patient presented with chest pain BP 150/ 95 diastolic murmur
and bruit over both carotids and subclavians absent pulse in right arm
what is the diagnosis
a- Aortic dissection
b- Takayasu’ s
c- Wegner’ s
d- PAN

62. An obese young male presented with SOB and sleeping difficulties
at night BP 150/ 80 on examination lungs normal loud S2, What is the
next step
a- ABG
b- Start ACEI
c- Start calcium blocker
d- Sleep studies
e- Echo

63. Young female presented with blurring vision left hemiparysis CT


white matter changes, how can you diagnose MS
a- Location of the lesion on MRI
b- Dissemination in place and time
c- CSF for olesoclonal bands
d- Nerve conductions studies
e- Visual evoked potentials

64. Regarding Ebola virus


a- Presentation depends on fever
b- Invariably fatal 50% only
c- Common cause of death is cerebral hemorrhage
d- Can be treated in the community

65. Young female with history of two spontaneous abortion


presented with left side hemiparesis she has a history of DVT 6months
ago, What would you do expect to find on echo
a- ASD
b- Patent foramin ovale
c- RVH
d- Normal echo

66. Primigravida presented with progressive joundus liver enzymes


are normal platelets 160,000, What is the diagnosis
a- Fatty liver of pregnancy
b- HELLP
c- Preeclampsia
d- Hepatitis
e- Cholestasis of pregnancy

67. Alcoholic patient was found in the street on examination he was


comatosed normal vital signs ALT 60, AST ?, ALP 180, ABGPH 2.1, PO2-
12. PCO2-3, HCO3-11, urine crystals, What is the diagnosis
a- Methanol poisoning
b- Ethanol poisoning
c- Ethalinic glycol poisoning
d- Acetyl salicelate poisoning
e- TCAoverdose

68. Young patient with past history of gastrojejunstomy presented


with carbopedal spasm , ca 6ml , Mg 0.6, What treatment will u give
a- Oral Ca
b- IVCa & Mg
c- Oral Mg

69. Young patient presented with pain in his muscle at the site of
exercise his urine also is black, what is the diagnosis
a- Mcardles
b- Poupe’ s
c- Vongeirch’ s
d- Mytocondrial
70. Young female presented with palpitation and seizure she has
eaten less carbohydrate since last week yesterday she developed
seizure and was started on pentin but seizure continued her serum
sodium was 120ml/ dl, what is the next step to improve her outcome
a- Stop gabapentin
b- Give dextrose
c- Give hypertonic saline

71. Aman presented with left knee swelling no fever no bleeding


under mucosa APTT64 PLT 150,000 factor 8 normal , What is the
diagnosis
a- Hemophilia A
b- Hemophilia B
c- ITP
d- Septicarthritis

72. Patient presented with anemia HB 10 MCV60 hypochromatic,


What is the diagnosis
a- Iron deficiency anemia
b- Thalasemia minor
c- Seckle cell anemia
d- Thalasemia major

73. Apatient presented with anemia and chronic diarrhea vitamin D


was low, What is the next investigation
a- Colonoscopy
b- CT abdomen
c- OGD
d- Abdominal US

74. Young patient presented with cereballar symptoms and signs he


has liver cirrhosis, What is the cause of cereballar syndrome
a- Acquired hepato cereballar degeneration
b- Alcoholic cereballar degeneration

75. Patient with polysythaemia vera, What is the treatment


a- Aspirin

76. Patient with infective endocardites started on benzyl pencilin and


gentamicin developed nausea and fatigability showed SCR 6.5, urea 180,
urine sodium , urine keratenin 3000Ug, What is the cause of renal
impairment
a- Prerenal failure
b- ATN
c- Glomerulonephritis

77. Elderly man presented with collapse CT showed hemorrhage in


parietal and frontal areas, what is the diagnosis
a- Hemorrhagic stroke
b- Atherosclerosis
c- Amyloid angiopathy

78. Patient presented with blurring vision she was hypertensive


funduscopy retinal hemorrhage INR-high immunoglobulin, what is the
diagnosis
a- Central retinal artery occlusion
b- Multiple myeloma
c- Waldenstron’ s macroglobulinemia

79. Patient with HIVpresented with confusion and hemiplegia MRI


periverticular white matter changes what is most likely the causative
organism
a- HIV
b- Toxoplasmosis
c- CMV
d- JCvirus
e- Herpes

80. Patient with asthma relapses he is on short dose steroid, what is


the next step
a- Ipratropium
b- Increase steroid dose
c- Solmetral
d- Oral thuephylline

81. Patient with ischiemic heart disease known to have multiple


goiter but normal thyroid function he under went KABGlater presented
with irritability TSH 0.02, T4 200, what is the diagnosis
a- Stress induced thyrotoxicosis
b- Sick euthyroid disease
c- Contrast induced thyrotoxicosis

82. Patient presented with fever joint pain involving hand and feet
also she has skin rash on examination lymphadenopathy friction rub tip
of spleen in felt, what is the diagnosis
a- Viral arthritis
b- SLE
c- Brucellosis
d- Adult onset stills disease

83. Elderly patient presented with fatigability on examination pale


spleen is 15cm BCM, What is the underlying diagnosis
a- BCR/ ABL mutation
b- 9-22 mutation
c- CLL

84. Young patient who is known cocaine addict presented with


convulsion, what is the treatment
a- No treatment
b- Beta blockers
c- Carbamazepine
d- Na valproat

85. Patient who is known to have atrial fibrillation presented with


bloody diarrhea the patient should undergo colonoscopy, What is the
next step
a- Continue warfarin at same dose
b- Stop warfarin and shift to heparin
c- Stop warfarin shift to heparin 3days before surgery

86. Patient presented with dysphagia Xray showed fluid level behind
the heart, what is the next step
a- OGD
b- Mannometry
c- Oesophageal PH
d- CT chest
87. Hypertensive patient started on ACEI one week later renal
function deteriorates, what is likely the diagnosis
a- Renal artery stenosis due to fibromuscular or dysplasia
b- Renal artery stenosis due to atherosclerosis
???
88. Hypertensive patient presented with albumin urea counter of his
albumin urea reduces incidence of
a- Visual deterioration
b- Renal deterioration
c- Cardiovascular risk

89. Patient with skin rash renal impairment HCV+ve, what is the
diagnosis
a- Cryoglobullnemia
b- ABPA
c- Silicosis most likely associated with increased ACE
d- TB
???

90. Patient with hypercalcemia, what investigations

a-CT chest

91. Patient with HF and arrhythmia what is the treatment


a- CD

Patient presented with elevated JVP no waves

a- CHB
b- SVO

92. Patient with pancytopenia and diarrhea spleenomegaly


a- Vitamin B12 deficiency
?????

93. Patient with primary billiary cirrhosis treatment


a- Deoxycholic acid

94. Patient with PSCbest investigation


a- MRCP
b- ERCP

95. PNH
a- Pancytopenia + thrombosis

96. Patient is prolactinoina started on dopamine agonist she


improved but after few months she deteriorated with high prolactinoina
size with infraintestinal extension, what is the next action
a- High dose of bronociptine
b- Transvicisral superficial recession
c- Radiation

97. Diabetic patient developed repeated hypoglycemia attack on


development of renal impairment he is on metformen, glyburide, how
are you going to manage this patient
a- Stop glyburide
b- Stop metformen
????
98. Patient with a history of GIT bleeding and a machinery murmur at
the back, what is the best underlying cause
a- HHT
b- PDA
99. A50 years patient with iron deficiency anemia low ferritin, which
investigation is the least helpful investigation
a- Urine analysis
b- Upper and lower endoscopy
c- Stool for occult blood
d- Investigation for celiac disease

100. A young male presented with bleeding per rectum upper and
lower endoscopy were normal, what investigation should be requested
a- Meckel’s scan

A patient with CAbronchus(squamous cell) and pleural effusion his lung


function test showed FEV1-1.7 which is regarded as contraindication for
surgical recession

a- His FEV1
b- Evidence of pleural and invasion
c- Presence of pleural effusion
101. Heparin induced thrombocytopenia (type2)
a- Usually develop in less than 7days
b- Usually associated with thrombosis
c- Early invasion 20,000 identical diagnosis
102. female who delivered recently presented with excessive sweating
and palpitation examination revealed a non tender enlarged thyroid
gland TSH <0.001, what is the best investigation to help in her diagnosis
a- Thyroglobulin
b- Thyroid isotope scanning
c- ESR
d- Anti thyroid antibodies
103. A diabetic patient presented with sensory symptoms in the lower
limps plus foot drop then he developed similar features in the upper
limp the cause is
a- Distal diabetic neuropathy
b- Vasculitis
104. In a patient presented with dyspnea lung function test showed
restrictive lung disease with high KCO, what is the most likely
underlying cause
a- Idiopathic pulmonary hemosidrosis
b- Ankyliosing spondylitis
c- Idiopathic lung fibrosis
d- Asthma

105. Most common manifestation of TB in HIVpatient


a- Pleural effusion
b- Apical fibrosis
c- Lymphadenopathy

106. Female presented with tremor and high facial hair the most
likely agent to cause these features
a- Phenytoin
b- Na valproate
c- Carbamezapine
107. Patient with GB, what is the best monitor for prognosis
a- Vital capacity
b- FEV1
108. 28years pregnant female presented with headache blurring of
vision investigation showed low PLT, low HB, what is the most
underlying cause
a- HELLP
b- TTP
109. Apregnant lady with high blood pressure no proteinurea started
on methyldopa but developed depression secondary to drug, what is the
best alternative antihypertensive
a- Lizinopril
b- Labetalol
c- Losartan

110. Afarmer presented with discharging sinuses investigations for TB


were negative what is the most likely underlying cause
a- Actinomyies
b- Staphylococcus aureus

111. Aknown case of sickle cell anemia presented with pneumonia


what is the most likely underlying cause
a- Staphylococcus aureus
b- Haemophilus influeinzae
c- Streptococcus pneumonia

112. Regarding LBBB which statement is true


a- The majority of patient have no underlying structural heart disease
b- It reduces the life expectancy of patients
c- In the presence of acute chest pain it is not an indication for
thrombolysis
113. Apatient 18weeks pregnant with prothoic valve on warfarin 3mg
her last INR is 2.5-3 what is the best action regarding her
anticoagulation
a- Switch warfarin to aspirin
b- Switch warfarin to clopidegrel
c- Continue with the same dose and maintain her INR between 2.5-3
?????

114. With HIVoropharyngeal candidasis what is the best treatment


a- Oral Itraconazole
b- IVganiclovi
c- IVamphetricin
115. What is the common presentation for herpes simplex virus type1
a- Asymptomatic
b- Genital ulcer
c- Stomato gingivitis
116. Patient with colonic carcinoma stage3 with brain metastisis, what
is the common type of convolution presented in such patient
a- Complex partial seizure
b- Secondary generalized seizure
c- Absence seizure
117. Patient presented with hypoacuthetic macule with enlaged nerves
biopsy showed granuloma with a few cells but no leprosy bacilli what is
the most likely underlying cause
a- Tuberculoid leprosy
b- Lepromatous leprosy
118. Common presentation of psoriatic arthropathy
a- Distal arthropathy 10%
b- RAlike presentation 30-40%
c- Axial arthropathy
119. Known case of celiac disease presented with abdominal pain and
loss of weight, what is the investigation
a- CT abdomen
b- Colonoscopy
120. Patient with diarrhea features of malabsorption arthropathy and
lymphadenopathy, what is the treatment your going to give for him
a- Gluteau-free diet
b- Tetracyclin
c- Prednislone
121. Renal transplant patient 6weeks post operative presented with
fever malaise and lymphadenopathy what is the most likely underlying
cause
a- CMV
122. Young male hypertensive on examination fullness of loins, what
investigation you

Want to request for this patient

a- Abdominal US

123. SLE nephritis type4, what type of treatment you want to provide
for this patient
a- Predinsolone
b- Predinsolone and cyclophosphamide

124. Patient with paracitamol poisoning measure monitor her


prognosis
a- Paracitamol level
b- Liver and ceratenin level
c- Alanine aminotransferase level
d- Alcohol level if she injected it with alcohol
??????

125. Female presented with haemptysis she is a nonsmoker chest Xray


revealed a mass, what is the most likely underlying neoplasm
a- Squamous cell carcinoma
b- Sinall cell carcinoma
c- Adeno carcinoma

126. COPDpt developed LVdysfunction , which sign is specific For HF


due to LVdysfunction
a- Basal crepitation
b- Pulsus alternaus
c- S4
d- Pansystolic murmur

127. What is the common disease for PEGtrue except


a- Chron’ s disease
b- Infective ascites
c- Osphegeal stricture ???
d- Previous gastric surgery

128. Patient with osteoporosis on HRT developed DVT which of these


treatments is contraindicated
a- Tamaxifen
b- Vit D, Ca supplement

129. Alcoholic pain killer presented with sensory neuropathy what is


the cause
a- Lead neuropathy
b- Alcoholic neuropathy
c- Hereditary sensomotor PN

130. Patient with HIVCD4 <200 + TB how are you going to treat him
a- Prazinanides ethambutol rifampeicen
1. A farmer presented with haematemesis he was brownish 6 units of
blood BP 120/ 80, INR 1-6 , What do you want to do next
a- Platlets
b- Nothing
c- Fresh frozen plasma
d- Cryopcipitate

2. A patient with V.leishmamises treated with sfiboglucanate for 30days


but still febrile what will you give
a- Repeat sfiboglucanate coarse
b- Amphoterien B(liposomal)
c- Amautadiue
d- Pentamedine

3. Patient present with right loin mass and haematuria CXR bilateral apical
scaring, What is the diagnosis
a-Renal TB
b-Renal cell CA
c-Good Posteur
d-PAN

4. A farmer presented with fever for 6 weeks temperature 38, tip of spleen
palpable blood culture –ve, urine has RBC, What is the diagnosis
a-Q fever endocardites
b-Renal TB
c-Renal cell CA
d-glumerulonephritis

5. Patient with HIVpresented with confusion and fever LP protein 0.8,


sugar low, What is the next step
a- Acyclovir
b- Ganciclovir
c- Pyrimebliamin and sulfadeazinc
d- amphotericin
6. An obese lady presented with S.O.B she used appitite supressent to
reduce weight Echo RT ventricular hypertrophy, What is the diagnosis
a- primary pulmonary hypertension
b- secondary pulmonary hypertension
c- pulmonary fibrosis

7. Patient was found to have retroperitoneal sarco ,What is likely to be


found
a- hypocalcemia
b- hypercalcemia
c- hypoglycemia

8. Elderly patient with CKD on HD presented with fracture in both


shoulders, What is the cause
a- Amuloidosis, A
b- B- microglobulin amiploiet
c- Hyperparathyroidism
d- Myeloma

9. Patient with sickle cell disease presented with a vascular necrosis in the
hip and fever, What treatment will u give
a- Ceftriaxone
b- Vancomycn
c- Ceftriaxone + Vancomycn
d- Surgery
e- Anti TB

10. Patient with MGUS how can you differentiate from multiple
myeloma
a- IGG 2gm
b- Bencezonce in urine
c- Plasma cells in bone marrow 7%

11. Patient presented with convulsion and left side weakness CT


showed temporal heamorage MRVshowed lateral sinus thrombosis,
what is the treatment
a- Warfarin
b- Heparine
c- Aspirin
d- Nimodipine
e- Streptokinase

12. Patient Presented with headache and blurring of vision glucose


are normal ESR 73 Which of the following is correct
a- Temporal artery biopsy before any medical intervention
b- Methyl prednisolone administration
c- Cyclophosphamide
d- CT brain

13. Young female has frequent attack of unilateral headache lasting


many hours she used a lot of analgesic without any improvement, What
is management steps
a- Sumatriptan
b- MRI brain
c- Stop analgesia, analgesic rebound headache
d- Propranolol

14. Patient presented with chest pain ECGshowed ST in inferior


leads given streptokinase 6hours later she has severe chest pain, ECG
43% reduction in ST, PR 52, BP 130/ 80, What is the next step
a- I.Vnitrates
b- Repeat thrombolysis
c- Send patient for PCI
d- Give morphine
e- Give B. blockers
15. 60 years male presented with severe headache lasting 3 to 4 hrs
associated with fearing and congestion, What treatment can be given to
prevent attack
a- High flow oxygen
b- Tryptan
c- Phenytoin
d- Verapanicl

16. Patient is known to have Parkinson disease well controlled on


levedopa presented with tremer, What is the treatment
a- Pramipixol
b- Benzhexol
c- Cabergolin
d- Amantadine

17. Patient who is Known smoker presented with headache and


conjuctival congestion examination is normal platlets WBC raised HB
18mg/ dl What is the next step
a- RBG
b- Red cell mass
c- Erythropoietin
d- Bone marrow

18. Elderly patient presented with dysphagia facial numbness


examination, Horner sighn and loss of pain few pain in face and lateral
lumb where is the lesion
a- Superior cerebeller artery occlusion
b- Vertibral artery dissection
c- Subarachnoid hemorage
d- Middle cerebral artery occlusion
e- Carotid artery aneurysim

19. Hypertensive patient presented with joint pain in 1 st metacarpal


joint on examination there is cellulites on 1st metacarpal joint, what is
the next step
a- Revise anti hypertensive medication
b- Give allupurinol
c- Hand Xray
d- Check serum uric acid

20. Patient with family history of gout presented with joint pain in
the left ankle on examination swelling left ankle, what is the next step
a- Check serum uric acid because is invariable high
b- Joint aspiration
c- Allupurinol
d- Urine for urate
e- Advice to reduce red meat

21. Alcoholic man presented with fever and confusion CSF showed
pleocytosis and reduced sugar, raised proteins, What is the diagnosis
a- TB meningitis
b- Nocardiases
c- JCvirus infection
d- Cryptococcal meningitis
e- Lesteria infection

22. Pyridoxin mechanism of action


a- Convert homocysteinurea to cystathionin
b- Utilize phosphate group

23. Leflunamide
a- Same action as sulphasalazine and MTX
b- Short duration
c- Relative contraindication in pregnancy nursing women and liver tissue

24. Apatient presented with chalculous cholecystitis he has history of


operation in his abdomen what is diagnosis
a- Ulcerative colitis
b- Pernicious anemia
c- Crohnis disease

25. Patient presented with pain and swelling in his muscle, What is
the cause
A- Flariases
B- Trichenellosis
C- Hook worm
D- Cystecercocis

26. Patient with bronchial asthma complain of fever shortness of


breath given eythromysene developed nausea and vomiting, she was on
inhaler and saluetrol + thuophyllin, whats the diagnosis
a- Asthma attack
b- Chest infection
c- Theophyllin toxisity
27. Student presented with SOB hemoptysis CXRshowed patchy
shadowing lung function test restrictive with high KCO& DLCO, What is
the diagnosis
a- Asthma
b- Sarcoidosis
c- Emphysema
d- Idiopathic pulmonary hemosidrosis
e- Pulmonary hemorrhage

28. Patient with tricyclic overdose, what is the treatment


a- Oxygen
b- HCO3
c- Dialysis
d- Lavage

29. Patient with shezophrania presented with rigidity and fever, what
is the treatment
a- Dantrolene
b- Diazepam
c- Phenytoin
d- haloperidole

30. Patient with history of dysentery presented with echomosis CBC


platelets 45 fragmented cell, what is the diagnosis
a- HUS
b- TTP
c- Renal failure
d- DIC

31. Patient with COPD what will you likely find


a- PH 7.5, PO2-14, PCO2-3, HCO3-20
b- PH 7.4, PO2-12, PCO2-6, HCO3-25
c- PH 7.3, PO2-10,PCO2-6,HCO3-36

32. Patient known diabetic for 20years he is on insulin and good


control 6weeks ago he developed sudden edema urine showed 24hrs
protein 6gm, how are you going to manage
a- ARB
b- ACED
c- Renal biopsy
d- Abdominal ultra sound

33. Female presented with confusion and jaundice she is known to


have auto immune hepatitis on examination she is febrile confused
deeply jaundice and there is massive ascites, What is the next step in
management
a- Abdominal CT
b- Blood culture
c- Urine analysis
d- TWBC
e- Peritoneal tap

34. HIVpatient presented with blurring of vision on examination


funduscopy there is hemorrhage following retinal artery branches, what
is the diagnosis
a- Herpes zoster infection
b- Toxoplasmosis
c- CMVinfection
d- Cryptococus

35. Obese lady on routine investigations found to have increased liver


enzymes, what is the next appropriate investigation
a- CT abdomen
b- Liver biopsy
c- Abdominal ultrasound
d- Upper GI indoscopy
e- Hepatitis screening

36. Patient with Hodgchen lymphoma presented with SVCO, what is


your management
a- Diuretics
b- Radiotherapy
c- Chemotherapy
d- Surgery

37. Apatient known to have antiphospholipid syndrome presented


with left leg DVT started on worfarin, what is the appropriate next step
a- Worfarin for life with INR 2-3
b- Worfarin for life with INR3-4
c- Worfarin for 3months
d- Worfarin for 6months with INR2-3 then aspirin for life

38. Patient with lateral sinus thrombosis, What is the treatment


a- Worfarin
b- Heparin
c- Aspirin
d- streptokinase

39. Nurse while giving injection to patient with hepatitis B injected


herself, What screening test do you request
a- HBSA+ anticore IGM
b- HB e AG
c- HB s AB
d- HIVtest
40. Diabetic patient is discovered recently to have hypertension
started on ACE, What is the target for BP control
a- 110/ 70
b- 125/ 75 if had microalbumin
c- 130/ 80
d- <140/ 80

41. Apatient who is known to have angina he is on maximal tolerated


doses of nitrates and beta blockers, what drug is likely to benefit his
cardiac prognosis
a- ARBS
b- Noridihidro
c- ACEIS
d- Statins

42. Apatient who is known case of COPDon oral steroid started


5days ago and short long acting presented with worsening shortness of
breath ABGPO2-11, PCO2-6, PH 7.25,What further investigation(he is
on O2 28%)
a- Increase O2
b- Decrease O2
c- IVsteroid
d- Mechanical ventilation
e- NIPPV

43. Young female asymptomatic found to have RBG131ml/ dl,


FBG131ml/ dl, BMI 42, What is appropriate next to do
a- GTT
b- Repeat FBG
c- Life style modification
d- Metformin
e- Glemibride

44. Apatient with normal thyroid function found to have thyroid


nodule, What is the appropriate next step
a- Radioiodine Isotope
b- FNA
c- Surgery
d- Observation

45. Apatient who is started on H.D recently after first H.D section he
becomes confused, what is the most likely explanation
a- Hypercalcemia
b- Osmolar shift
c- Infection

46. Adiabetic and hypertensive patient with known history of


atherosclerosis US renal shape is normal but right kidney 7cm left
kidney 8cm, what most likely explanation
a- Chronic glomerulonephritis
b- Ischemia
c- Pyelonephrites
d- Diabetic nephropathy
e- Hypertensive nephropathy

47. Apatient presented with paraplegia and kyphus1 disease, What


was diagnosed and vertebral compression fracture was confirmed after
2months on treatment the patient power improved grade4 but the
kyphus still, what could explain the mechanism of neuron lesion
a- Compression vertebral fracture
b- Empyema
c- Paravertebral abscess
d- Dscitis
48. Apatient is known IDDM on insulin developed nightmare and his
morning glucose between 200-300, what is the next appropriate step
a- Increase the insulin dose
b- Decrease the insulin dose
c- Off metformin
d- Start glimibride

49. Patient with prephral neuropathy and painful skin rash, What is
diagnosis
a- Lepra type2 reaction

50. Patient with typhoid fever received treatment improbably


presented after 18days, What likely complications
a- ARF
b- Myocarditis
c- ARDS

51. Patient presented with painful proximal myopathy there is rash at


the back involving upper arm and shoulder Na 120ml, What is
underlying aetiology
a- Hypothyrodisn
b- Poly mayalgia rheumatic
c- Cryoglobiniemia
d- Dermatomyositis

52. Patient presented with high blood pressure on examination


hyperpigmented of buccal mucosa his Kis low, HCO3 is high, What is the
diagnosis
a- Addison’ s disease
b- Nelson syndrome
c- Conns disease
d- Basal cell carcinoma
53. Elderly patient with painful fingers on examination has
spleenomegaly, lymphadenopathy, What is the next step
a- Electrophorisis
b- Bone marrow
c- Urine analysis

54. 45 years female has early menoupause started HRT but


discontinued because of intolerance presented with irritability, What is
the next action
a- TSH
b- Repeat HRT
c- Reassurance
d- Antithyroid treatment

55. Apatient represented with SOB on mild exertion on examination


S1 mid diastolic murmur and systolic murmur echo right MS + minor
MR, what is your next step
a- ACEI
b- Valvotomy
c- Mitral valve replacement
d- Nitrates
e- Frusemide
56. Patient not known to have cardiac disease presented with AF,
What is the best for rate control
a- Verapamil
b- Digoxin
c- Atenolol
d- Flecainide
e- Amiodarone

Alcoholic presented with confusion OGD showed oesophageal varices, what


treatment to reduce portal pressure and prevent hematemesis

a- Propranolol
b- Sclerotheropy
c- Band ligation
d- PPI

57. Patient with lupus nephrites stage 4, what is the best treatment?
a- Steroid
b- Cyclophosphanamide
c- Cyclophosphanamide + steroid
d- Hemodialysis

58. Apatient who have HBVinfection viral load 50,000 liver biopsy
bridging fibrosis, what is the treatment
a- Peg INF
b- Peg INF +lamivudine
c- lamivudine
d- Peg INF + ribavirin

59. Patient presented with chest pain radiating to the back on


examination pulse un equal diastolic murmur in the left sternum BP
200/ 95 what is the likely the management
a- ACEI
b- Heparin
c- Streptokinase
d- labetalol

60. young girl with recurrent chest infection and halitosis developed
convultions what is the next step
a- navalproat
b- no treatment
c- carbamazepine
d- phenytoin
e- CT brain
61. Patient presented with chest pain BP 150/ 95 diastolic murmur
and bruit over both carotids and subclavians absent pulse in right arm
what is the diagnosis
a- Aortic dissection
b- Takayasu’ s
c- Wegner’ s
d- PAN

62. An obese young male presented with SOB and sleeping difficulties
at night BP 150/ 80 on examination lungs normal loud S2, What is the
next step
a- ABG
b- Start ACEI
c- Start calcium blocker
d- Sleep studies
e- Echo

63. Young female presented with blurring vision left hemiparysis CT


white matter changes, how can you diagnose MS
a- Location of the lesion on MRI
b- Dissemination in place and time
c- CSF for olesoclonal bands
d- Nerve conductions studies
e- Visual evoked potentials

64. Regarding Ebola virus


a- Presentation depends on fever
b- Invariably fatal 50% only
c- Common cause of death is cerebral hemorrhage
d- Can be treated in the community

65. Young female with history of two spontaneous abortion


presented with left side hemiparesis she has a history of DVT 6months
ago, What would you do expect to find on echo
a- ASD
b- Patent foramin ovale
c- RVH
d- Normal echo

66. Primigravida presented with progressive joundus liver enzymes


are normal platelets 160,000, What is the diagnosis
a- Fatty liver of pregnancy
b- HELLP
c- Preeclampsia
d- Hepatitis
e- Cholestasis of pregnancy

67. Alcoholic patient was found in the street on examination he was


comatosed normal vital signs ALT 60, AST ?, ALP 180, ABGPH 2.1, PO2-
12. PCO2-3, HCO3-11, urine crystals, What is the diagnosis
a- Methanol poisoning
b- Ethanol poisoning
c- Ethalinic glycol poisoning
d- Acetyl salicelate poisoning
e- TCAoverdose

68. Young patient with past history of gastrojejunstomy presented


with carbopedal spasm , ca 6ml , Mg 0.6, What treatment will u give
a- Oral Ca
b- IVCa & Mg
c- Oral Mg

69. Young patient presented with pain in his muscle at the site of
exercise his urine also is black, what is the diagnosis
a- Mcardles
b- Poupe’ s
c- Vongeirch’ s
d- Mytocondrial
70. Young female presented with palpitation and seizure she has
eaten less carbohydrate since last week yesterday she developed
seizure and was started on pentin but seizure continued her serum
sodium was 120ml/ dl, what is the next step to improve her outcome
a- Stop gabapentin
b- Give dextrose
c- Give hypertonic saline

71. Aman presented with left knee swelling no fever no bleeding


under mucosa APTT64 PLT 150,000 factor 8 normal , What is the
diagnosis
a- Hemophilia A
b- Hemophilia B
c- ITP
d- Septicarthritis

72. Patient presented with anemia HB 10 MCV60 hypochromatic,


What is the diagnosis
a- Iron deficiency anemia
b- Thalasemia minor
c- Seckle cell anemia
d- Thalasemia major

73. Apatient presented with anemia and chronic diarrhea vitamin D


was low, What is the next investigation
a- Colonoscopy
b- CT abdomen
c- OGD
d- Abdominal US

74. Young patient presented with cereballar symptoms and signs he


has liver cirrhosis, What is the cause of cereballar syndrome
a- Acquired hepato cereballar degeneration
b- Alcoholic cereballar degeneration

75. Patient with polysythaemia vera, What is the treatment


a- Aspirin

76. Patient with infective endocardites started on benzyl pencilin and


gentamicin developed nausea and fatigability showed SCR 6.5, urea 180,
urine sodium , urine keratenin 3000Ug, What is the cause of renal
impairment
a- Prerenal failure
b- ATN
c- Glomerulonephritis

77. Elderly man presented with collapse CT showed hemorrhage in


parietal and frontal areas, what is the diagnosis
a- Hemorrhagic stroke
b- Atherosclerosis
c- Amyloid angiopathy

78. Patient presented with blurring vision she was hypertensive


funduscopy retinal hemorrhage INR-high immunoglobulin, what is the
diagnosis
a- Central retinal artery occlusion
b- Multiple myeloma
c- Waldenstron’ s macroglobulinemia

79. Patient with HIVpresented with confusion and hemiplegia MRI


periverticular white matter changes what is most likely the causative
organism
a- HIV
b- Toxoplasmosis
c- CMV
d- JCvirus
e- Herpes

80. Patient with asthma relapses he is on short dose steroid, what is


the next step
a- Ipratropium
b- Increase steroid dose
c- Solmetral
d- Oral thuephylline

81. Patient with ischiemic heart disease known to have multiple


goiter but normal thyroid function he under went KABGlater presented
with irritability TSH 0.02, T4 200, what is the diagnosis
a- Stress induced thyrotoxicosis
b- Sick euthyroid disease
c- Contrast induced thyrotoxicosis

82. Patient presented with fever joint pain involving hand and feet
also she has skin rash on examination lymphadenopathy friction rub tip
of spleen in felt, what is the diagnosis
a- Viral arthritis
b- SLE
c- Brucellosis
d- Adult onset stills disease

83. Elderly patient presented with fatigability on examination pale


spleen is 15cm BCM, What is the underlying diagnosis
a- BCR/ ABL mutation
b- 9-22 mutation
c- CLL

84. Young patient who is known cocaine addict presented with


convulsion, what is the treatment
a- No treatment
b- Beta blockers
c- Carbamazepine
d- Na valproat

85. Patient who is known to have atrial fibrillation presented with


bloody diarrhea the patient should undergo colonoscopy, What is the
next step
a- Continue warfarin at same dose
b- Stop warfarin and shift to heparin
c- Stop warfarin shift to heparin 3days before surgery

86. Patient presented with dysphagia Xray showed fluid level behind
the heart, what is the next step
a- OGD
b- Mannometry
c- Oesophageal PH
d- CT chest
87. Hypertensive patient started on ACEI one week later renal
function deteriorates, what is likely the diagnosis
a- Renal artery stenosis due to fibromuscular or dysplasia
b- Renal artery stenosis due to atherosclerosis
???
88. Hypertensive patient presented with albumin urea counter of his
albumin urea reduces incidence of
a- Visual deterioration
b- Renal deterioration
c- Cardiovascular risk

89. Patient with skin rash renal impairment HCV+ve, what is the
diagnosis
a- Cryoglobullnemia
b- ABPA
c- Silicosis most likely associated with increased ACE
d- TB
???

90. Patient with hypercalcemia, what investigations

a-CT chest

91. Patient with HF and arrhythmia what is the treatment


a- CD

Patient presented with elevated JVP no waves

a- CHB
b- SVO

92. Patient with pancytopenia and diarrhea spleenomegaly


a- Vitamin B12 deficiency
?????

93. Patient with primary billiary cirrhosis treatment


a- Deoxycholic acid

94. Patient with PSCbest investigation


a- MRCP
b- ERCP

95. PNH
a- Pancytopenia + thrombosis

96. Patient is prolactinoina started on dopamine agonist she


improved but after few months she deteriorated with high prolactinoina
size with infraintestinal extension, what is the next action
a- High dose of bronociptine
b- Transvicisral superficial recession
c- Radiation

97. Diabetic patient developed repeated hypoglycemia attack on


development of renal impairment he is on metformen, glyburide, how
are you going to manage this patient
a- Stop glyburide
b- Stop metformen
????
98. Patient with a history of GIT bleeding and a machinery murmur at
the back, what is the best underlying cause
a- HHT
b- PDA
99. A50 years patient with iron deficiency anemia low ferritin, which
investigation is the least helpful investigation
a- Urine analysis
b- Upper and lower endoscopy
c- Stool for occult blood
d- Investigation for celiac disease

100. A young male presented with bleeding per rectum upper and
lower endoscopy were normal, what investigation should be requested
a- Meckel’s scan

A patient with CAbronchus(squamous cell) and pleural effusion his lung


function test showed FEV1-1.7 which is regarded as contraindication for
surgical recession

a- His FEV1
b- Evidence of pleural and invasion
c- Presence of pleural effusion
101. Heparin induced thrombocytopenia (type2)
a- Usually develop in less than 7days
b- Usually associated with thrombosis
c- Early invasion 20,000 identical diagnosis
102. female who delivered recently presented with excessive sweating
and palpitation examination revealed a non tender enlarged thyroid
gland TSH <0.001, what is the best investigation to help in her diagnosis
a- Thyroglobulin
b- Thyroid isotope scanning
c- ESR
d- Anti thyroid antibodies
103. A diabetic patient presented with sensory symptoms in the lower
limps plus foot drop then he developed similar features in the upper
limp the cause is
a- Distal diabetic neuropathy
b- Vasculitis
104. In a patient presented with dyspnea lung function test showed
restrictive lung disease with high KCO, what is the most likely
underlying cause
a- Idiopathic pulmonary hemosidrosis
b- Ankyliosing spondylitis
c- Idiopathic lung fibrosis
d- Asthma

105. Most common manifestation of TB in HIVpatient


a- Pleural effusion
b- Apical fibrosis
c- Lymphadenopathy

106. Female presented with tremor and high facial hair the most
likely agent to cause these features
a- Phenytoin
b- Na valproate
c- Carbamezapine
107. Patient with GB, what is the best monitor for prognosis
a- Vital capacity
b- FEV1
108. 28years pregnant female presented with headache blurring of
vision investigation showed low PLT, low HB, what is the most
underlying cause
a- HELLP
b- TTP
109. Apregnant lady with high blood pressure no proteinurea started
on methyldopa but developed depression secondary to drug, what is the
best alternative antihypertensive
a- Lizinopril
b- Labetalol
c- Losartan

110. Afarmer presented with discharging sinuses investigations for TB


were negative what is the most likely underlying cause
a- Actinomyies
b- Staphylococcus aureus

111. Aknown case of sickle cell anemia presented with pneumonia


what is the most likely underlying cause
a- Staphylococcus aureus
b- Haemophilus influeinzae
c- Streptococcus pneumonia

112. Regarding LBBB which statement is true


a- The majority of patient have no underlying structural heart disease
b- It reduces the life expectancy of patients
c- In the presence of acute chest pain it is not an indication for
thrombolysis
113. Apatient 18weeks pregnant with prothoic valve on warfarin 3mg
her last INR is 2.5-3 what is the best action regarding her
anticoagulation
a- Switch warfarin to aspirin
b- Switch warfarin to clopidegrel
c- Continue with the same dose and maintain her INR between 2.5-3
?????

114. With HIVoropharyngeal candidasis what is the best treatment


a- Oral Itraconazole
b- IVganiclovi
c- IVamphetricin
115. What is the common presentation for herpes simplex virus type1
a- Asymptomatic
b- Genital ulcer
c- Stomato gingivitis
116. Patient with colonic carcinoma stage3 with brain metastisis, what
is the common type of convolution presented in such patient
a- Complex partial seizure
b- Secondary generalized seizure
c- Absence seizure
117. Patient presented with hypoacuthetic macule with enlaged nerves
biopsy showed granuloma with a few cells but no leprosy bacilli what is
the most likely underlying cause
a- Tuberculoid leprosy
b- Lepromatous leprosy
118. Common presentation of psoriatic arthropathy
a- Distal arthropathy 10%
b- RAlike presentation 30-40%
c- Axial arthropathy
119. Known case of celiac disease presented with abdominal pain and
loss of weight, what is the investigation
a- CT abdomen
b- Colonoscopy
120. Patient with diarrhea features of malabsorption arthropathy and
lymphadenopathy, what is the treatment your going to give for him
a- Gluteau-free diet
b- Tetracyclin
c- Prednislone
121. Renal transplant patient 6weeks post operative presented with
fever malaise and lymphadenopathy what is the most likely underlying
cause
a- CMV
122. Young male hypertensive on examination fullness of loins, what
investigation you

Want to request for this patient

a- Abdominal US

123. SLE nephritis type4, what type of treatment you want to provide
for this patient
a- Predinsolone
b- Predinsolone and cyclophosphamide

124. Patient with paracitamol poisoning measure monitor her


prognosis
a- Paracitamol level
b- Liver and ceratenin level
c- Alanine aminotransferase level
d- Alcohol level if she injected it with alcohol
??????

125. Female presented with haemptysis she is a nonsmoker chest Xray


revealed a mass, what is the most likely underlying neoplasm
a- Squamous cell carcinoma
b- Sinall cell carcinoma
c- Adeno carcinoma

126. COPDpt developed LVdysfunction , which sign is specific For HF


due to LVdysfunction
a- Basal crepitation
b- Pulsus alternaus
c- S4
d- Pansystolic murmur

127. What is the common disease for PEGtrue except


a- Chron’ s disease
b- Infective ascites
c- Osphegeal stricture ???
d- Previous gastric surgery

128. Patient with osteoporosis on HRT developed DVT which of these


treatments is contraindicated
a- Tamaxifen
b- Vit D, Ca supplement

129. Alcoholic pain killer presented with sensory neuropathy what is


the cause
a- Lead neuropathy
b- Alcoholic neuropathy
c- Hereditary sensomotor PN

130. Patient with HIVCD4 <200 + TB how are you going to treat him
a- Prazinanides ethambutol rifampeicen
OSCE recall MD July 2017
By: Batol Gurashi

This 15 years old patients presented with abdominal pain and bloody diarrhea

What is the abnormality?

What is the diagnosis?

How to confirm the diagnosis?


2- This are 2 pictures belong to same patient

describe 3 abnormalities in his face?

What is your diagnosis?

2
3- This 2 pictures belong to the same patient

What is the abnormality in the face?

What is the abnormality in the hand?

What is the unifying diagnosis?

3
4- Describe 4 abnormalities in the CXR?

What is the diagnosis?

Mitral valve stenosis

4
5- This patient presented with recurrent bilateral loin pain

ABG done PH 7.32 HCO3 18 CO2 3 mmol

What is the interpretation of his acid base status?

What is the likely diagnosis?

5
6- This is X- ray of an elbow joint of a patient

What is the abnormality?

What is the diagnosis?

What is the finding you expect to find in the examination?

6
7- This is the CXR in patient presented with cough

Mention 2 abnormalities in the CXR?

What is the diagnosis?

7
8- This patient has history of joint pain presented with severe pain in his leg

What is the abnormality?

What is the diagnosis?

Mention 2 associated rash?

8
9- patient who is a known case of alcoholic liver cirrhosis presented with fever and abdominal
pain

His ascitic fluid analysis:

WCC 1000

Albumin 31 mg/dl

Serum protien 40 mg/dl

What is the diagnosis?

Mention 4 other test in ascitic fluid you will request?

9
10- This the hand of a man who is dealing with animal

What is the lesion?

What is the diagnosis?

10
11- This is fundus of a patient

What is the abnormality?

Mention 2 tests you will request to know the underlying cause?

11
12- This is belong to patient with joint pain

Mention 2 abnormalities?

What is the diagnosis?

12
13- This is a family pedigree with an inherited disease?

What is the mood of inheritance?


Vit
Give an example of a disease with this mood of inheritance?
DRecets
Reet
syndrom

13
14- This is MRI of patient who has no headache and no convulsions

What is the radiological diagnosis?

Suggest 2 symptoms may this patient presented with?

14
15- This patient presented with fever and productive cough

Empyema

What is abnormality ?

What may you expect to find in the physical examination?

15
16- This is 2 pictures belong to same patient

IBD
RA
Saricidosis

What are the 2 abnormalities shown?

Mention 2 possible diagnosis?

16
17- This an ECG of a patient with heart failure

What is the ECG abnormality?

What is the likely cause?

17
18- This is a CXR of this patient

18
What is abnormalities in the face and x-ray? Wasted of
small musle
What is the physical finding you expect to find in the upper limbs?
of hand
Weak pulse
19
19- This patient has painful red eyes

What is the abnormality shown?

Mention a likely diagnosis?

20
20- 16 years old girl presented with loss of consciousness and high grade fever

WCC 16000

Hb 7 gm/dl

Urea 11 mmol

Describe 4 abnormalities in the peripheral blood picture? Hypochromic


Spherocyte
What is the haematological diagnosis?
Helemt cell
21 Schisocyte
22
OSCERECALLAUGUST2021

N.B.Answer
smaynotbeaccur
ate

1-Thi
sfemal
ehasat
ri
alf
ibr
il
lat
ion.
.Whati
stheDx?

Rt
.li
ngual
art
eryt
hromboembol
i
sm
2-Namet
hepar
asi
te?Tr
ansmi
ssi
on?

my
iasi
smaggot
..Ski
ncont
act

3-Dx?t
reat
ment
?

Neur
ocy
sti
cer
cosi
s..Pr
azi
quant
iloral
bendazol
4-Thi
spthasepi
lepsy
..2abnor
mal
i
ties?

Skul
lxr
ayf
indi
ng?

Dx?

Por
twi
nest
ain.
.Gum hy
per
trophy
..
.Faci
alpal
sy

Tr
am l
i
necal
cif
icat
ion

St
urgeweber
5-Der
mat
ologi
cDescr
ipt
ion?Der
mat
ome?

Papul
o-Vesi
cul
arr
ash(
Shi
ngl
es)T10

6-Thi
sptpr
esent
edepai
nlesskneeswel
li
ng.
.

Whati
stheDx?

Dxi
fhehasmi
osi
s?

Char
cotknee.
.Neur
osy
phi
l
is
7-ment
iont
heabnor
mal
it
y?2causes?

spl
eencal
cif
icat
ion.
..TB.
.Sar
coi
dosi
s

8-ECG?bi
gemi
ny.
.Mostl
ikel
ycause?

9-Dx?2causes?RtTot
alwhi
tel
ung

Rtl
ungcol
l
apse.
.Pneumonect
omy
10-Thi
sptwasaskedt
oabducthi
sar
m agai
nst

r
esi
stance(
arr
owpoi
ntedt
osupr
aspi
nat
us)
..

Namet
hemuscl
e?Cer
vicali
nner
vat
ion?

supr
aspi
nat
usi
njur
y..C5

11-Thi
spti
sknownHCV.
.whati
stheDx?

por
phy
riacut
aneat
arda

12-Whati
stheabnor
mal
it
y?2mal
ignantcauses?

Gy
necomast
ia.
.Lungadenocar
cinoma.
.HCC
13-Whati
stheabnor
mal
it
y?3cl
ini
calsi
gns?

Thi
ckper
icar
dium (
per
icar
dit
is)

14-Whati
stheDx?2ot
hercl
ini
calsi
gns?

Gr
ave'
sdi
sease(
pret
ibi
almy
xedema)

Thy
roi
dacr
opachy
,opt
hal
mopat
hy
15-Per
ipher
albl
oodpi
ctur
e..
.??
?Hemol
yti
c??
?G6PD

16-Whati
stheder
mat
ologi
calDx?

2ot
hercl
ini
calsi
gns?

Whati
stheDx?

er
ythemamar
ginat
um.
.RHD

17-2abnor
mal
it
ies?Dx?

Psor
iat
icar
thr
it
is
18-Thi
sptpr
esent
edef
ebr
il
eil
lnesssev
eralweaksago.
.

Dx?t
tt?

PostKal
azarder
mal
lei
shmani
asi
s

19-Whati
stheabnor
mal
it
y?2causes?

Subhy
aloi
dhge

20-Thi
sptpr
esent
edel
til
iacf
ossapai
n&f
ever
..

Whati
stheabnor
mal
it
y?2compl
icat
ions?

Di
ver
ti
cul
osi
s/i
ti
s
1. A farmer presented with haematemesis he was brownish 6 units of
blood BP 120/ 80, INR 1-6 , What do you want to do next
a- Platlets
b- Nothing
c- Fresh frozen plasma
d- Cryopcipitate

2. A patient with V.leishmamises treated with sfiboglucanate for 30days


but still febrile what will you give
a- Repeat sfiboglucanate coarse
b- Amphoterien B(liposomal)
c- Amautadiue
d- Pentamedine

3. Patient present with right loin mass and haematuria CXR bilateral apical
scaring, What is the diagnosis
a-Renal TB
b-Renal cell CA
c-Good Posteur
d-PAN

4. A farmer presented with fever for 6 weeks temperature 38, tip of spleen
palpable blood culture –ve, urine has RBC, What is the diagnosis
a-Q fever endocardites
b-Renal TB
c-Renal cell CA
d-glumerulonephritis

5. Patient with HIVpresented with confusion and fever LP protein 0.8,


sugar low, What is the next step
a- Acyclovir
b- Ganciclovir
c- Pyrimebliamin and sulfadeazinc
d- amphotericin
6. An obese lady presented with S.O.B she used appitite supressent to
reduce weight Echo RT ventricular hypertrophy, What is the diagnosis
a- primary pulmonary hypertension
b- secondary pulmonary hypertension
c- pulmonary fibrosis

7. Patient was found to have retroperitoneal sarco ,What is likely to be


found
a- hypocalcemia
b- hypercalcemia
c- hypoglycemia

8. Elderly patient with CKD on HD presented with fracture in both


shoulders, What is the cause
a- Amuloidosis, A
b- B- microglobulin amiploiet
c- Hyperparathyroidism
d- Myeloma

9. Patient with sickle cell disease presented with a vascular necrosis in the
hip and fever, What treatment will u give
a- Ceftriaxone
b- Vancomycn
c- Ceftriaxone + Vancomycn
d- Surgery
e- Anti TB

10. Patient with MGUS how can you differentiate from multiple
myeloma
a- IGG 2gm
b- Bencezonce in urine
c- Plasma cells in bone marrow 7%

11. Patient presented with convulsion and left side weakness CT


showed temporal heamorage MRVshowed lateral sinus thrombosis,
what is the treatment
a- Warfarin
b- Heparine
c- Aspirin
d- Nimodipine
e- Streptokinase

12. Patient Presented with headache and blurring of vision glucose


are normal ESR 73 Which of the following is correct
a- Temporal artery biopsy before any medical intervention
b- Methyl prednisolone administration
c- Cyclophosphamide
d- CT brain

13. Young female has frequent attack of unilateral headache lasting


many hours she used a lot of analgesic without any improvement, What
is management steps
a- Sumatriptan
b- MRI brain
c- Stop analgesia, analgesic rebound headache
d- Propranolol

14. Patient presented with chest pain ECGshowed ST in inferior


leads given streptokinase 6hours later she has severe chest pain, ECG
43% reduction in ST, PR 52, BP 130/ 80, What is the next step
a- I.Vnitrates
b- Repeat thrombolysis
c- Send patient for PCI
d- Give morphine
e- Give B. blockers
15. 60 years male presented with severe headache lasting 3 to 4 hrs
associated with fearing and congestion, What treatment can be given to
prevent attack
a- High flow oxygen
b- Tryptan
c- Phenytoin
d- Verapanicl

16. Patient is known to have Parkinson disease well controlled on


levedopa presented with tremer, What is the treatment
a- Pramipixol
b- Benzhexol
c- Cabergolin
d- Amantadine

17. Patient who is Known smoker presented with headache and


conjuctival congestion examination is normal platlets WBC raised HB
18mg/ dl What is the next step
a- RBG
b- Red cell mass
c- Erythropoietin
d- Bone marrow

18. Elderly patient presented with dysphagia facial numbness


examination, Horner sighn and loss of pain few pain in face and lateral
lumb where is the lesion
a- Superior cerebeller artery occlusion
b- Vertibral artery dissection
c- Subarachnoid hemorage
d- Middle cerebral artery occlusion
e- Carotid artery aneurysim

19. Hypertensive patient presented with joint pain in 1 st metacarpal


joint on examination there is cellulites on 1st metacarpal joint, what is
the next step
a- Revise anti hypertensive medication
b- Give allupurinol
c- Hand Xray
d- Check serum uric acid

20. Patient with family history of gout presented with joint pain in
the left ankle on examination swelling left ankle, what is the next step
a- Check serum uric acid because is invariable high
b- Joint aspiration
c- Allupurinol
d- Urine for urate
e- Advice to reduce red meat

21. Alcoholic man presented with fever and confusion CSF showed
pleocytosis and reduced sugar, raised proteins, What is the diagnosis
a- TB meningitis
b- Nocardiases
c- JCvirus infection
d- Cryptococcal meningitis
e- Lesteria infection

22. Pyridoxin mechanism of action


a- Convert homocysteinurea to cystathionin
b- Utilize phosphate group

23. Leflunamide
a- Same action as sulphasalazine and MTX
b- Short duration
c- Relative contraindication in pregnancy nursing women and liver tissue

24. Apatient presented with chalculous cholecystitis he has history of


operation in his abdomen what is diagnosis
a- Ulcerative colitis
b- Pernicious anemia
c- Crohnis disease

25. Patient presented with pain and swelling in his muscle, What is
the cause
A- Flariases
B- Trichenellosis
C- Hook worm
D- Cystecercocis

26. Patient with bronchial asthma complain of fever shortness of


breath given eythromysene developed nausea and vomiting, she was on
inhaler and saluetrol + thuophyllin, whats the diagnosis
a- Asthma attack
b- Chest infection
c- Theophyllin toxisity
27. Student presented with SOB hemoptysis CXRshowed patchy
shadowing lung function test restrictive with high KCO& DLCO, What is
the diagnosis
a- Asthma
b- Sarcoidosis
c- Emphysema
d- Idiopathic pulmonary hemosidrosis
e- Pulmonary hemorrhage

28. Patient with tricyclic overdose, what is the treatment


a- Oxygen
b- HCO3
c- Dialysis
d- Lavage

29. Patient with shezophrania presented with rigidity and fever, what
is the treatment
a- Dantrolene
b- Diazepam
c- Phenytoin
d- haloperidole

30. Patient with history of dysentery presented with echomosis CBC


platelets 45 fragmented cell, what is the diagnosis
a- HUS
b- TTP
c- Renal failure
d- DIC

31. Patient with COPD what will you likely find


a- PH 7.5, PO2-14, PCO2-3, HCO3-20
b- PH 7.4, PO2-12, PCO2-6, HCO3-25
c- PH 7.3, PO2-10,PCO2-6,HCO3-36

32. Patient known diabetic for 20years he is on insulin and good


control 6weeks ago he developed sudden edema urine showed 24hrs
protein 6gm, how are you going to manage
a- ARB
b- ACED
c- Renal biopsy
d- Abdominal ultra sound

33. Female presented with confusion and jaundice she is known to


have auto immune hepatitis on examination she is febrile confused
deeply jaundice and there is massive ascites, What is the next step in
management
a- Abdominal CT
b- Blood culture
c- Urine analysis
d- TWBC
e- Peritoneal tap

34. HIVpatient presented with blurring of vision on examination


funduscopy there is hemorrhage following retinal artery branches, what
is the diagnosis
a- Herpes zoster infection
b- Toxoplasmosis
c- CMVinfection
d- Cryptococus

35. Obese lady on routine investigations found to have increased liver


enzymes, what is the next appropriate investigation
a- CT abdomen
b- Liver biopsy
c- Abdominal ultrasound
d- Upper GI indoscopy
e- Hepatitis screening

36. Patient with Hodgchen lymphoma presented with SVCO, what is


your management
a- Diuretics
b- Radiotherapy
c- Chemotherapy
d- Surgery

37. Apatient known to have antiphospholipid syndrome presented


with left leg DVT started on worfarin, what is the appropriate next step
a- Worfarin for life with INR 2-3
b- Worfarin for life with INR3-4
c- Worfarin for 3months
d- Worfarin for 6months with INR2-3 then aspirin for life

38. Patient with lateral sinus thrombosis, What is the treatment


a- Worfarin
b- Heparin
c- Aspirin
d- streptokinase

39. Nurse while giving injection to patient with hepatitis B injected


herself, What screening test do you request
a- HBSA+ anticore IGM
b- HB e AG
c- HB s AB
d- HIVtest
40. Diabetic patient is discovered recently to have hypertension
started on ACE, What is the target for BP control
a- 110/ 70
b- 125/ 75 if had microalbumin
c- 130/ 80
d- <140/ 80

41. Apatient who is known to have angina he is on maximal tolerated


doses of nitrates and beta blockers, what drug is likely to benefit his
cardiac prognosis
a- ARBS
b- Noridihidro
c- ACEIS
d- Statins

42. Apatient who is known case of COPDon oral steroid started


5days ago and short long acting presented with worsening shortness of
breath ABGPO2-11, PCO2-6, PH 7.25,What further investigation(he is
on O2 28%)
a- Increase O2
b- Decrease O2
c- IVsteroid
d- Mechanical ventilation
e- NIPPV

43. Young female asymptomatic found to have RBG131ml/ dl,


FBG131ml/ dl, BMI 42, What is appropriate next to do
a- GTT
b- Repeat FBG
c- Life style modification
d- Metformin
e- Glemibride

44. Apatient with normal thyroid function found to have thyroid


nodule, What is the appropriate next step
a- Radioiodine Isotope
b- FNA
c- Surgery
d- Observation

45. Apatient who is started on H.D recently after first H.D section he
becomes confused, what is the most likely explanation
a- Hypercalcemia
b- Osmolar shift
c- Infection

46. Adiabetic and hypertensive patient with known history of


atherosclerosis US renal shape is normal but right kidney 7cm left
kidney 8cm, what most likely explanation
a- Chronic glomerulonephritis
b- Ischemia
c- Pyelonephrites
d- Diabetic nephropathy
e- Hypertensive nephropathy

47. Apatient presented with paraplegia and kyphus1 disease, What


was diagnosed and vertebral compression fracture was confirmed after
2months on treatment the patient power improved grade4 but the
kyphus still, what could explain the mechanism of neuron lesion
a- Compression vertebral fracture
b- Empyema
c- Paravertebral abscess
d- Dscitis
48. Apatient is known IDDM on insulin developed nightmare and his
morning glucose between 200-300, what is the next appropriate step
a- Increase the insulin dose
b- Decrease the insulin dose
c- Off metformin
d- Start glimibride

49. Patient with prephral neuropathy and painful skin rash, What is
diagnosis
a- Lepra type2 reaction

50. Patient with typhoid fever received treatment improbably


presented after 18days, What likely complications
a- ARF
b- Myocarditis
c- ARDS

51. Patient presented with painful proximal myopathy there is rash at


the back involving upper arm and shoulder Na 120ml, What is
underlying aetiology
a- Hypothyrodisn
b- Poly mayalgia rheumatic
c- Cryoglobiniemia
d- Dermatomyositis

52. Patient presented with high blood pressure on examination


hyperpigmented of buccal mucosa his Kis low, HCO3 is high, What is the
diagnosis
a- Addison’ s disease
b- Nelson syndrome
c- Conns disease
d- Basal cell carcinoma
53. Elderly patient with painful fingers on examination has
spleenomegaly, lymphadenopathy, What is the next step
a- Electrophorisis
b- Bone marrow
c- Urine analysis

54. 45 years female has early menoupause started HRT but


discontinued because of intolerance presented with irritability, What is
the next action
a- TSH
b- Repeat HRT
c- Reassurance
d- Antithyroid treatment

55. Apatient represented with SOB on mild exertion on examination


S1 mid diastolic murmur and systolic murmur echo right MS + minor
MR, what is your next step
a- ACEI
b- Valvotomy
c- Mitral valve replacement
d- Nitrates
e- Frusemide
56. Patient not known to have cardiac disease presented with AF,
What is the best for rate control
a- Verapamil
b- Digoxin
c- Atenolol
d- Flecainide
e- Amiodarone

Alcoholic presented with confusion OGD showed oesophageal varices, what


treatment to reduce portal pressure and prevent hematemesis

a- Propranolol
b- Sclerotheropy
c- Band ligation
d- PPI

57. Patient with lupus nephrites stage 4, what is the best treatment?
a- Steroid
b- Cyclophosphanamide
c- Cyclophosphanamide + steroid
d- Hemodialysis

58. Apatient who have HBVinfection viral load 50,000 liver biopsy
bridging fibrosis, what is the treatment
a- Peg INF
b- Peg INF +lamivudine
c- lamivudine
d- Peg INF + ribavirin

59. Patient presented with chest pain radiating to the back on


examination pulse un equal diastolic murmur in the left sternum BP
200/ 95 what is the likely the management
a- ACEI
b- Heparin
c- Streptokinase
d- labetalol

60. young girl with recurrent chest infection and halitosis developed
convultions what is the next step
a- navalproat
b- no treatment
c- carbamazepine
d- phenytoin
e- CT brain
61. Patient presented with chest pain BP 150/ 95 diastolic murmur
and bruit over both carotids and subclavians absent pulse in right arm
what is the diagnosis
a- Aortic dissection
b- Takayasu’ s
c- Wegner’ s
d- PAN

62. An obese young male presented with SOB and sleeping difficulties
at night BP 150/ 80 on examination lungs normal loud S2, What is the
next step
a- ABG
b- Start ACEI
c- Start calcium blocker
d- Sleep studies
e- Echo

63. Young female presented with blurring vision left hemiparysis CT


white matter changes, how can you diagnose MS
a- Location of the lesion on MRI
b- Dissemination in place and time
c- CSF for olesoclonal bands
d- Nerve conductions studies
e- Visual evoked potentials

64. Regarding Ebola virus


a- Presentation depends on fever
b- Invariably fatal 50% only
c- Common cause of death is cerebral hemorrhage
d- Can be treated in the community

65. Young female with history of two spontaneous abortion


presented with left side hemiparesis she has a history of DVT 6months
ago, What would you do expect to find on echo
a- ASD
b- Patent foramin ovale
c- RVH
d- Normal echo

66. Primigravida presented with progressive joundus liver enzymes


are normal platelets 160,000, What is the diagnosis
a- Fatty liver of pregnancy
b- HELLP
c- Preeclampsia
d- Hepatitis
e- Cholestasis of pregnancy

67. Alcoholic patient was found in the street on examination he was


comatosed normal vital signs ALT 60, AST ?, ALP 180, ABGPH 2.1, PO2-
12. PCO2-3, HCO3-11, urine crystals, What is the diagnosis
a- Methanol poisoning
b- Ethanol poisoning
c- Ethalinic glycol poisoning
d- Acetyl salicelate poisoning
e- TCAoverdose

68. Young patient with past history of gastrojejunstomy presented


with carbopedal spasm , ca 6ml , Mg 0.6, What treatment will u give
a- Oral Ca
b- IVCa & Mg
c- Oral Mg

69. Young patient presented with pain in his muscle at the site of
exercise his urine also is black, what is the diagnosis
a- Mcardles
b- Poupe’ s
c- Vongeirch’ s
d- Mytocondrial
70. Young female presented with palpitation and seizure she has
eaten less carbohydrate since last week yesterday she developed
seizure and was started on pentin but seizure continued her serum
sodium was 120ml/ dl, what is the next step to improve her outcome
a- Stop gabapentin
b- Give dextrose
c- Give hypertonic saline

71. Aman presented with left knee swelling no fever no bleeding


under mucosa APTT64 PLT 150,000 factor 8 normal , What is the
diagnosis
a- Hemophilia A
b- Hemophilia B
c- ITP
d- Septicarthritis

72. Patient presented with anemia HB 10 MCV60 hypochromatic,


What is the diagnosis
a- Iron deficiency anemia
b- Thalasemia minor
c- Seckle cell anemia
d- Thalasemia major

73. Apatient presented with anemia and chronic diarrhea vitamin D


was low, What is the next investigation
a- Colonoscopy
b- CT abdomen
c- OGD
d- Abdominal US

74. Young patient presented with cereballar symptoms and signs he


has liver cirrhosis, What is the cause of cereballar syndrome
a- Acquired hepato cereballar degeneration
b- Alcoholic cereballar degeneration

75. Patient with polysythaemia vera, What is the treatment


a- Aspirin

76. Patient with infective endocardites started on benzyl pencilin and


gentamicin developed nausea and fatigability showed SCR 6.5, urea 180,
urine sodium , urine keratenin 3000Ug, What is the cause of renal
impairment
a- Prerenal failure
b- ATN
c- Glomerulonephritis

77. Elderly man presented with collapse CT showed hemorrhage in


parietal and frontal areas, what is the diagnosis
a- Hemorrhagic stroke
b- Atherosclerosis
c- Amyloid angiopathy

78. Patient presented with blurring vision she was hypertensive


funduscopy retinal hemorrhage INR-high immunoglobulin, what is the
diagnosis
a- Central retinal artery occlusion
b- Multiple myeloma
c- Waldenstron’ s macroglobulinemia

79. Patient with HIVpresented with confusion and hemiplegia MRI


periverticular white matter changes what is most likely the causative
organism
a- HIV
b- Toxoplasmosis
c- CMV
d- JCvirus
e- Herpes

80. Patient with asthma relapses he is on short dose steroid, what is


the next step
a- Ipratropium
b- Increase steroid dose
c- Solmetral
d- Oral thuephylline

81. Patient with ischiemic heart disease known to have multiple


goiter but normal thyroid function he under went KABGlater presented
with irritability TSH 0.02, T4 200, what is the diagnosis
a- Stress induced thyrotoxicosis
b- Sick euthyroid disease
c- Contrast induced thyrotoxicosis

82. Patient presented with fever joint pain involving hand and feet
also she has skin rash on examination lymphadenopathy friction rub tip
of spleen in felt, what is the diagnosis
a- Viral arthritis
b- SLE
c- Brucellosis
d- Adult onset stills disease

83. Elderly patient presented with fatigability on examination pale


spleen is 15cm BCM, What is the underlying diagnosis
a- BCR/ ABL mutation
b- 9-22 mutation
c- CLL

84. Young patient who is known cocaine addict presented with


convulsion, what is the treatment
a- No treatment
b- Beta blockers
c- Carbamazepine
d- Na valproat

85. Patient who is known to have atrial fibrillation presented with


bloody diarrhea the patient should undergo colonoscopy, What is the
next step
a- Continue warfarin at same dose
b- Stop warfarin and shift to heparin
c- Stop warfarin shift to heparin 3days before surgery

86. Patient presented with dysphagia Xray showed fluid level behind
the heart, what is the next step
a- OGD
b- Mannometry
c- Oesophageal PH
d- CT chest
87. Hypertensive patient started on ACEI one week later renal
function deteriorates, what is likely the diagnosis
a- Renal artery stenosis due to fibromuscular or dysplasia
b- Renal artery stenosis due to atherosclerosis
???
88. Hypertensive patient presented with albumin urea counter of his
albumin urea reduces incidence of
a- Visual deterioration
b- Renal deterioration
c- Cardiovascular risk

89. Patient with skin rash renal impairment HCV+ve, what is the
diagnosis
a- Cryoglobullnemia
b- ABPA
c- Silicosis most likely associated with increased ACE
d- TB
???

90. Patient with hypercalcemia, what investigations

a-CT chest

91. Patient with HF and arrhythmia what is the treatment


a- CD

Patient presented with elevated JVP no waves

a- CHB
b- SVO

92. Patient with pancytopenia and diarrhea spleenomegaly


a- Vitamin B12 deficiency
?????

93. Patient with primary billiary cirrhosis treatment


a- Deoxycholic acid

94. Patient with PSCbest investigation


a- MRCP
b- ERCP

95. PNH
a- Pancytopenia + thrombosis

96. Patient is prolactinoina started on dopamine agonist she


improved but after few months she deteriorated with high prolactinoina
size with infraintestinal extension, what is the next action
a- High dose of bronociptine
b- Transvicisral superficial recession
c- Radiation

97. Diabetic patient developed repeated hypoglycemia attack on


development of renal impairment he is on metformen, glyburide, how
are you going to manage this patient
a- Stop glyburide
b- Stop metformen
????
98. Patient with a history of GIT bleeding and a machinery murmur at
the back, what is the best underlying cause
a- HHT
b- PDA
99. A50 years patient with iron deficiency anemia low ferritin, which
investigation is the least helpful investigation
a- Urine analysis
b- Upper and lower endoscopy
c- Stool for occult blood
d- Investigation for celiac disease

100. A young male presented with bleeding per rectum upper and
lower endoscopy were normal, what investigation should be requested
a- Meckel’s scan

A patient with CAbronchus(squamous cell) and pleural effusion his lung


function test showed FEV1-1.7 which is regarded as contraindication for
surgical recession

a- His FEV1
b- Evidence of pleural and invasion
c- Presence of pleural effusion
101. Heparin induced thrombocytopenia (type2)
a- Usually develop in less than 7days
b- Usually associated with thrombosis
c- Early invasion 20,000 identical diagnosis
102. female who delivered recently presented with excessive sweating
and palpitation examination revealed a non tender enlarged thyroid
gland TSH <0.001, what is the best investigation to help in her diagnosis
a- Thyroglobulin
b- Thyroid isotope scanning
c- ESR
d- Anti thyroid antibodies
103. A diabetic patient presented with sensory symptoms in the lower
limps plus foot drop then he developed similar features in the upper
limp the cause is
a- Distal diabetic neuropathy
b- Vasculitis
104. In a patient presented with dyspnea lung function test showed
restrictive lung disease with high KCO, what is the most likely
underlying cause
a- Idiopathic pulmonary hemosidrosis
b- Ankyliosing spondylitis
c- Idiopathic lung fibrosis
d- Asthma

105. Most common manifestation of TB in HIVpatient


a- Pleural effusion
b- Apical fibrosis
c- Lymphadenopathy

106. Female presented with tremor and high facial hair the most
likely agent to cause these features
a- Phenytoin
b- Na valproate
c- Carbamezapine
107. Patient with GB, what is the best monitor for prognosis
a- Vital capacity
b- FEV1
108. 28years pregnant female presented with headache blurring of
vision investigation showed low PLT, low HB, what is the most
underlying cause
a- HELLP
b- TTP
109. Apregnant lady with high blood pressure no proteinurea started
on methyldopa but developed depression secondary to drug, what is the
best alternative antihypertensive
a- Lizinopril
b- Labetalol
c- Losartan

110. Afarmer presented with discharging sinuses investigations for TB


were negative what is the most likely underlying cause
a- Actinomyies
b- Staphylococcus aureus

111. Aknown case of sickle cell anemia presented with pneumonia


what is the most likely underlying cause
a- Staphylococcus aureus
b- Haemophilus influeinzae
c- Streptococcus pneumonia

112. Regarding LBBB which statement is true


a- The majority of patient have no underlying structural heart disease
b- It reduces the life expectancy of patients
c- In the presence of acute chest pain it is not an indication for
thrombolysis
113. Apatient 18weeks pregnant with prothoic valve on warfarin 3mg
her last INR is 2.5-3 what is the best action regarding her
anticoagulation
a- Switch warfarin to aspirin
b- Switch warfarin to clopidegrel
c- Continue with the same dose and maintain her INR between 2.5-3
?????

114. With HIVoropharyngeal candidasis what is the best treatment


a- Oral Itraconazole
b- IVganiclovi
c- IVamphetricin
115. What is the common presentation for herpes simplex virus type1
a- Asymptomatic
b- Genital ulcer
c- Stomato gingivitis
116. Patient with colonic carcinoma stage3 with brain metastisis, what
is the common type of convolution presented in such patient
a- Complex partial seizure
b- Secondary generalized seizure
c- Absence seizure
117. Patient presented with hypoacuthetic macule with enlaged nerves
biopsy showed granuloma with a few cells but no leprosy bacilli what is
the most likely underlying cause
a- Tuberculoid leprosy
b- Lepromatous leprosy
118. Common presentation of psoriatic arthropathy
a- Distal arthropathy 10%
b- RAlike presentation 30-40%
c- Axial arthropathy
119. Known case of celiac disease presented with abdominal pain and
loss of weight, what is the investigation
a- CT abdomen
b- Colonoscopy
120. Patient with diarrhea features of malabsorption arthropathy and
lymphadenopathy, what is the treatment your going to give for him
a- Gluteau-free diet
b- Tetracyclin
c- Prednislone
121. Renal transplant patient 6weeks post operative presented with
fever malaise and lymphadenopathy what is the most likely underlying
cause
a- CMV
122. Young male hypertensive on examination fullness of loins, what
investigation you

Want to request for this patient

a- Abdominal US

123. SLE nephritis type4, what type of treatment you want to provide
for this patient
a- Predinsolone
b- Predinsolone and cyclophosphamide

124. Patient with paracitamol poisoning measure monitor her


prognosis
a- Paracitamol level
b- Liver and ceratenin level
c- Alanine aminotransferase level
d- Alcohol level if she injected it with alcohol
??????

125. Female presented with haemptysis she is a nonsmoker chest Xray


revealed a mass, what is the most likely underlying neoplasm
a- Squamous cell carcinoma
b- Sinall cell carcinoma
c- Adeno carcinoma

126. COPDpt developed LVdysfunction , which sign is specific For HF


due to LVdysfunction
a- Basal crepitation
b- Pulsus alternaus
c- S4
d- Pansystolic murmur

127. What is the common disease for PEGtrue except


a- Chron’ s disease
b- Infective ascites
c- Osphegeal stricture ???
d- Previous gastric surgery

128. Patient with osteoporosis on HRT developed DVT which of these


treatments is contraindicated
a- Tamaxifen
b- Vit D, Ca supplement

129. Alcoholic pain killer presented with sensory neuropathy what is


the cause
a- Lead neuropathy
b- Alcoholic neuropathy
c- Hereditary sensomotor PN

130. Patient with HIVCD4 <200 + TB how are you going to treat him
a- Prazinanides ethambutol rifampeicen
GREY CASES 6th OCTOBER 2020 RECALL

D. hyder Ishag Adam

CASE NO (1)

Building constructor complains of fever ,nausea and vomiting for 3


weeks.

O/E:Abdomen,chest and CNS are normal

Investigations:

Deranged liver functions mainly hepatocellular features and renal


functions are also impaired.

Hb 11g/dl TWBC 11.2 Platelet 100

BFFM &ICT for malaria &Widal test for entrica &brucella all are
negative

1_What is most likely diagnosis ?


Leptospirosis

2-Mention one differentional diagnosis ?


Viral haemorrhagic fever

3_Mention one sensitive investigation?


Blood &CSF culture or PCR for leptospira interogans

4_Mention specific treatment ?


Doxacycline or penicillin
CASE NO (2)

Young female from southern Kordofan presented with fever associated


with sweating and no rigors and also complained of palpitation,
exertional shortness of breath, dark urine and weight loss .She has
history of recurrent malaria

O/E:

The pt looks unwell pale not jaundiced

There is soft systolic murmur at apex

Mild splenomegally

Hb 9.5 G/dl TWBC & PLT are normal

ESR70mm/h reticulocyte count 1.5%

1-Mention two differential diagnoses ?


Infective endocarditis

Systemic lupus erythematosus

2- Mention three another clinical signs?


Splinter haemorrhage

Roth,s spot

Janeway lesion

3-Mention three investigations ?


Blood Culture

Echocardiography

Antidouble stranded DNA


CASE NO(3)

Elder man known diabetic on oral hypoglycaemic drugs and on good


control .he presented with unsteadiness,wide base gait,nystagmus and
vertigo. He is heavy smoker

O/E:

Disdiadodochokinesia,proprioception and diminished reflexes .

Planter reflex downgoing and no weakness. BMI 18.5

RBG 135 mg/dl HbA1c 6.5% MRI brain unremarkable

1-What is your full clinical diagnosis?


Paraneoplastic syndrome with cerebellar syndrome

2-What is underlying aetiology ?


Small cell lung cancer

3-Mention two relevant investigations?


Bronchoscopy with biopsy

HRCT chest

4-Mention two further approaches to reach the diagnosis?


Syphilis serology and serum B12 and folate

Anti HU
CASE NO (4)

Elderly man presented to A&E with acute centeral chest pain radiated
to the neck and back after an argument with his wife .

O/E: PR 120 BP 220/160

ECG was done revealed only tachycardia

His chest pain was relieved after given an analgesia but unfortunately
his chest pain recurred again after awhile .ECG was done again which
revealed ST elevation in V1---------V4

There is right hand pallor.BP190/100

1-What is your full clinical diagnosis ?


Aortic Dissection type (A) complicating with anteroseptal myocardial infarction and right hand
ischaemia

2-Mention three investigations ?


CT chest with contrast or MRI

Echocardiography

3-what is the appropriate management?


Control blood with betablocker(eg.labetalol iv)

Surgical correction
CASE NO (5)

Patient with HIV developed PCP and received septrin and developed
hypersentivity then switch to clindamycin and primiguine, his RFT is
impaired s.creatinine 1.8.Hb 10 plt 120

Then added dapsone for prophylaxis of PCP ,patient became dyspnic


So2 85% PH 7.32 HCO3 19 PCo2 33mmHg PO2 normal, cyanosed and
jaundiced .

Hb 7g/dl plt 100 retics count 10%

1-What is the most likely diagnosis?


dapsone induced methaemoglobinaemia ,possibly patient may have G6PD deficiency

2-What is the confirmatory test?


G6PD assay and serum level of methaemoglobulin

3-what is an immediate intervention?


Stop dapsone and primiguine(sulfur containing drugs)

Hyperbaric oxygen

Exchange transfusion

4-What is the treatment for long term?


Avoidance of sulfur containing drugs

Neublised pentamidine as prophylaxis of pcp

HAART continuation
RECALL OSCE 6th October 2020

D.Hyder Ishag Adam

1-
koilonychia and pallor

Ascaris lumbricoids
2-Picture

neurofibromas and café au lait on back

a-what is the diagnosis ?

Neurofibromatosis type 1

b-mention two causes of hypertention ?


-phaeochromatosis - fibromascular renal artery stenosis

3-picture

a-what is the diagnosis ?

cutanous larva migrans

b-mention two drugs ?

ivermectin and albendazole


4-ECG of patient with chronic cough

a-What is an ECG diagnosis ?

b-mention two causes of chronic cough ?

5-CXR :Aortograph

a-discribe :narrowing of the descending aorta(coarctation of the aorta)

b-mention three clinical signs?

a- Hypertention
b- Radiofemoral Delay
c- Rib notching

6-Picture

a-discribe the lesion ?

Two big frontal well demarcated ulcers

b-mention three causes?

a- Syphilis
b- Cutanous leshmaniasis
c- leprosy

7-ECG

a- What is an ECG diagnosis?


Inferior myocardial infarction

b-Patient developed hypotention ---what are causes of deterioration of patient within


48hours?

-Right Ventricular infarction

-Complete heart block


8-CXR

a-What are the findings?

Cardiomegally –upper lobe diversion –left atrial enlargement

b-what is the diagnosis ?

Left ventricular failure

9-CT CHEST
10-Picture shows :

a-What does the picture show?

Atrophic glossitis

b-Mention two causes of this sign?

Coeliac disease

Pernicious anaemia
11-CT ABDOMEN

a-what does it show ?

hydatid cyst with daughter in the left lobe of the liver

b-Mention two modalities of the management ?

-Albendazole for 1-3months

_PAIR procedure(puncture-aspiration-injection-reaspiration)
12-Blood film

a-what does it show ?

spherocytes-target cells –elleptocytes

b-What is the cause of abdominal pain?

Pigment gallstone causing cholecystitis

13-CXR

a-What are the findings?

Bilateral hilar shadowing-reticulonodular shadowing and cavity

b-Mention two causes?

-Tuberculosis

-Sarcoidosis
14-Two pictures

a- what is the diagnosis?


Leperomatous leprosy

b-Mention two drugs ?

-Dapsone - clofizamine
15-Pictures show

conjunctival haemorrhage and jaundice


a-Name the picture (b)-fly?

Aedes egypti

b-Mention three differential diagnoses ?

chikungunya

Yellow fever

Dengue fever

16-picture

a-what are the findings?

shows upper and lower lips pigmentation with blue and red colour involving teeth and
gum

b-what is the diagnosis ?

AML??
17-Picture

a-What does the picture show ?

Right scapular swinging

b-What is the diagnosis ?

long thoracic nerve palsy(serratus anterior weakness)


18-picture

a-What does the picture show?

Deviation of the uvula to the right side

b-what is the diagnosis ?

9th cranial nerve palsy


19-picture

a-what does picture show?

Right foot showing fourth toe gangrene with big toe vasculitic changes

b- Mention two causes ?


Diabetes Mellites
Thromboangitis obliterans
Grey cases recall July 2017
By Batol Gurashi
1- 50 years old male who is known diabetic presented with 10 days history of high
grade fever with rigors. He took full course of artimether without improvement.
On examination he is in pain febrile temp 39 PR 90 b/min RR 25 b/min
No lymphadenopathy chest examination there is tenderness in the area extending
from the right 7th intercostal space and down there is also dullness in the same area
cardiovascular examination normal abdominal examination there is tendereness in
the right hypochondrium, no splenomegallay liver span is 17 cm, negative shifting
dullness
Investigations HB 13 gm/dl WCC 13000 platelets 300000 LFT AST 40 ALT 50 bilirubin 1
total protiens 7 serum albumin 4 RFT normal
What is the diagnosis?

Suggest an alternative diagnosis?

Mention three investigations to your first diagnosis?

What is your management regarding preferable diagnosis?

2- 65 years old patient with history of diabetes and hypertension presented with
sudden lower limb weakness and he unable to walk
on examination he is fully conscious normal pupils PR 90 b/min irregular irregular
intact cranial nerve there is audible carotid bruit examination of the motor system
the abnormality is confined to lower limb there is hypotonia and hyporeflexia power
grade 2 in both limbs planter reflexes is mute sensation is normal for vibration and
joint position. There is palpable suprapubic mass
Investigation CBC normal lipid profile cholestrol 300 TGA 400

What is the diagnosis?

What is investigation of choice?

What is the underlying mechanism for the diagnosis?!

What is a simple measure you will do in the ER?


3- 30 years old patient presented with hypertension that failed to control with
medication he is on hydrochlorothiazide, amlodipine and lisinopril and still his BP
190/90
O/E he is obese but no other significant signs investigation done
Urea 40
S cr 1
K 2.5
Na 140
HCO3 34
Cl 115
What is likely diagnosis?

What is an alternative possible diagnosis?

Mentions three investigation you will request for your first diagnosis?

Outline the patient management?

4- 50 years old male presented with 6 months history of fatiguability and loss of weight
he has knee joint pain he has history of appendicectomy 1 year ago, his mother has
diabetes and his father died of MI at age of 80 years
O/E he is pale but not jaundiced and has maculopapular rash over his face neck
upper trunk and extensor surface of his upper limbs
cardiovascular examination JVP is 5 cm, hearts sound is normal but there is
pansystolicmurmer at left lower sternal edge
chest examination no bibasal crackles abdominal examination he has hepatomegally
and positive shifting dullness
He has lower limb oedema
Investigatios HB 11 gm/dl WCC 6000 platelets 350000
LFT normal RFT normal CXR normal
What is the diagnosis?

How you will confirm it?

5- patient presented with foot drop he has history of trauma to which he received
multiple blood transfusions examination he has purpuric rash over his legs and right
foot drop investigations ANA negative positive RF low C4 normal C3
What is the diagnosis?
What is the name given regarding his foot drop ?

What investigation you will request to confirm the diagnosis?


‫بسم هللا الرحمن الرحيم‬
RECALL MD EXAM 8.2021

DR.YOSRA HAMAD ABD ELRAHIM

BEST OF 4:

1. Pt with aortic stenosis , then developed heart failure and his


EF% decreased, What is the sign will be seen ?

Decreased in second heart sound

Increase in murmur

2. Pt with VSD present with tonsillitis. His murmur gets Harsher


from before many years, what your action?

Cardiac catheterization

Refer for surgical intervention

Penicillin prophlyactic

Observation

3. Pt with chest pain,ECG show inferior MI with 3mm ST elevation,


after Thrombolysis ST elevation became 0.5 mm on asprin,
statin and heparin,there is 2 episode of VT 6 to 10 seen in
monitor Vitals stable and pt free of pain, What is treatment ?

PCI

Beta blocker

Amiodarone

Pacemaker

4. Which disease of following associated with blunt y descend?

Constrictive pericarditis

Restrictive cardiomyopathy

Cardiac tamponade

Tricuspid regurgitation

5. Pt with anti phospholipid syndrome presented with SOB


suddenly ECG show ST depression in v1-v3 and mildly elevated s.
Troponin , what is appropriate step ?

CTPA

PCI

ECHO

6. Pt with heart failure after delivery received diuretic ,beta


blocker, spironolactone, and still symptomatic ,EF 25% what is the
best step?

CRT
ICD

ACE i

digoxin

7. Pt develop narrow complex tachycardia with absent p wave


,has h.of asthma , has normal vitals and has no chest pain ,what is
the treatment?

Adenosine

betablocker

verapamil

amiodarone

8. PT presented with acute chest pain,his vital BP 90/60 and


tachycardia and tachypenic.. ECG show rt V.strain pattern,Best
treatment :

Thrombolysis

Anticoagulant

Embolectomy

9. Pt recently was in farm present with cough, fever, jaundice


and flushing. CXR: feature of pneumonia deranged RFT, Deranged
LFT diagnosis:

Q fever

Leptospirosis

Legionella
Mycoplasma

10. Pt with history of asthma and drop foot with mild renal
impairment what is the diagnosis?

Wegener granulomatosis

Churg strauss syndrome

PAN

11. Lady with DVT on warfarin discovered to has pregnancy test


positive what is the best treatment?

Continue warfarin

Stop warfarin and give heparin in first TM ,then warfarin in

second TM and heparin in last TM

Give heparin in all trimesters

Delivery with cesarean section

12. Post MI pericarditis within 2 days what is the best


Management?

Steroid

Aspirin

13. What Most common arrhythmia that occurs 2 hours post MI ?

SVT

VT

AF
VF

14. Which One of the following drugs NOT used in IHD?

Labetalol

Atenolol

Bisoprolol

carvidelol

15. Young pt present with syncopal attack , murmur increase with


valsalva . Has abnormal BP during exercise, v. Thickness 2.5 cm

From following what is NOT ass with poor prognosis?

Family history

Early age at onset

V. thickness of 2.5 cm

Low BP during exercise

16. Young female pt with H of dizziness and repeated fell down


when playing and dark room ,on examination she has red eyes ,
no tremor or nystagmus also has mother has similar symptoms
when she washes her face what is the diagnosis?

ataxia telangectasia

Spinocerebllar degeneration

MS

17. In acute ischemic stroke what is the best investigation ?


MRI

CT

DWI

18. Pt with weakness and cortical blindness what is the


management ?

Heparin

Aspirin

Warfarin

19. Pt with dizziness and unsteadiness the symptoms aggravated


when turn on bed ,no nystagmus what is the diagnosis ?

Cerebellar ataxia

BPPV

Labrynthnitis

20. Pt with unilateral hearing impairment and loss of corneal


reflex at the same side where is the lesion ?

Cerebellopontine angle

Brain stem

21. Pt c/o hot sensation in throat ,tonsil and edge of mandible


how to manage ?

Amitryptalin

Gabapentin
Carbamazepine

22. Pt presented with hemiplegia on examination and


investigation he has AF what is the management ?

Thrombolysis

Aspirin

Warfarin

Heparin

23. pt with facial sensory loss and contralateral hemisensory loss


what is the diagnosis?

Lateral medullary syndrome

24. Pt with RT hearing loss and on examination there is nystagmus


when pt follow finger moving to the RT side of examiner, where is
the lesion?

Lt cerebellar lesion

Cerebellar vermis lesion

Rt vestibule

25. female known epileptic on Na valporate was founded to be


pregnant what is your action ?

Continue valporate

Switch smoothly to lamotrigen

Delivery with C/S


26. Pt with long history of asymmetrical weakness with
fasciculation with mild wasting no ataxia , sensory intact ,NCS
show nerve conduction block what is the diagnosis ?

GBS

CIDP

MMN

MND

27. All correct about carpal tunnel syndrome except:

Atrophy of thinner muscle

Loss of sensation in the ulner aspect

28. What is the type of ischemic stroke having high tendency to


transform into haemorrhagic?

Small vessels

Large vessels

Cardio Embolic

Thrombotic

29. : Drug newly discovered to be having role in stroke recovery:

Amantadine

Piracetam

baclofen
30.pt with severe headache awake him from sleep ,with nasal
congestion and eye redness repeated for 3 times ,comes every
year (seasonal) what is the management?

Codeine

Gabapentin

Indomethazine

Aspirin

31. pt with history of forgetfulness, urinary incontinence and


ataxic gait what is the diagnosis ?

Normal pressure hydrocephalus

CJD

MSA

LBD

32. What is the disease ass with ptosis without ophthalmoplegia ?

Myotonia dystrophia

Myasthenia gravis.

33. Which following malignancy metastasis to brain lead to


haemorrhagic stroke:

Lung CA

RCC

Non melaninotic skin cancer


Colonic cancer

34.From the followingwhich is affected in of pneumonia and ILD ?

Alveolar exchange

Vascular involvement

35. Pt presented with SOB ,PFT show normal FIV1/FVC and


decreased TLCO and KCO what is the diagnosis?

Asthma

Polycythemia

HF

Anemia

36. pt with fever and SOB investigation show eosinophilia, feature


of ILD and restrictive cardiomyopathy what is the diagnosis?

hyperesonophilic syndrome

Chronic eosiophilic pneumonia

Churge strauss syndrome

37. pt with Liver impairment and SOB when sit from lying flat ,
CXR showed increased vascularity in lower lobes what is the
diagnosis?

Hepatopulmonary syndrome

A1AT D

AVM
38.pt with SOB ,PFT revealed increased FIV 1/FVC and decreased
TLCO and normal KCO what is the diagnosis ?

Asthma

Lung fibrosis

Pneumoctomy

39.HIV pt with SOB and haemoptysis investigation showed +ve


galactomannan test , most likely diagnosis is?

Aspergilloma

ABPA

Invasive aspergillosis

40. Pt presented with increasing SOB for long time CXR:decreased


lung volume biopsy revealed feature of usual interstitial
pneumonia with neutrophil predominant what is best ttt?

Predisolone

Perfinedone.

Lung fibrosis

Asthma

41.Pt with COPD treated with oxygen before ABG what is


appropriate oxygen saturation level?

98-98

92-100
88-92

95-100

42.Pt received 5 units of FFP then developed severe SOB many


hours after transfusion, O/E widespread Rhonchi and CXR with
bilateral pulm. Infilterate , PCWP is normal , what is the diagnosis?

Pulmonary edema

TRALI

ARDS

Severe pneumonia

43. pt with low ph ,high co2 , low o2 , low HCO3 from the
following what is the correct ?

Fully compensated respiratory acidosis

Compensated metabolic acidosis

Mixed respiratory and metabolic acidosis

Mixed respiratory and metabolic alkalosis

44. Pt with RA for commencement of methotrexate , has contact


with TB Mantoux test –ve ,what is investigation of choice?

CXR

IGRA essay

Sputum for AFB


45.Pt come from travel C/O dry cough and then become ill and
has SOB ,CXR: bilateral infiltrate has deranged LFT the best
investigation is ?

Mycoplasma Serology

HRCT

Urinary legionella antigen

46. HIV pt with dysphagia and CD4 150 the most likely diagnosis
is?

CMV

Candidasis

Esophageal cancer

47.pt with high grade esophageal metaplasia what is the best ttt?

Esohagectomy

Annual endoscopy

Resection and radiofrequency ablation

48. pt with feature of malabsorption ,high MCV , anti endomyseal


–ve , xylose +ve what is the diagnosis?

Celiac

SBBO

Tropical sprue

Whipple disease
49. you called to see a pt in OBS ward delivered recently ,
developed haematemesis ,has h.of using OCP ,O/E: obese , ascitis
and splenomegaly what is the diagnosis?

Liver cirrhosis with portal HTN

portal vein thrombosis

Fatty liver of pregnancy

50. Pt with lymphoid tissue associated tumor best ttt is?

Chemotherapy

H.pylori eradication

Surgical resection

Radiotherapy

51. 50 yrs old pt with epigastric pain and iron deficiency anemia,
O/E: he is slim ,underwent endoscopy and colonoscopy ended
without taking biopsy due to pt discomfort ,what to do?

Repeat endoscopy and colonoscopy

Red cell isotope scan

White cell isotope scan

Anti tissue Trans glutamase

52.Pt with chronic watery diarrhoea ,it continue even during


fasting ,inv: hypokalemia what is the ttt?

Octreotide
cholestyramine

53. Pt with ulcerative colitis presented with mild to moderate


bloody diarrhoea what is the best ttt?

Local steroid

local MSA

IV steroid

Oral MSA

54.Pt presented with abdominal pain and ascitis , ascetic fluid


send for analysis and start antibiotic but no improvement ,Ascetic
fluid revealed high neutrophil and waiting for culture, what to do?

Change antibiotic

Stop antibiotic

Continue antibiotic till culture result

Do abd. CT

55. Pt with gout on allopurinol discover to have h.pylori positive


what is the best management?

Omeprazole clarythromycin amoxicillin

Omeprazole,clarythromycin,mitronidazol

Omeprazole ,tetracycline,bisthmus,mitronidazole

Omeprazole, Levofloxacin, clarythromycin


56. Pt with dementia and diarrhoea ,on examination has scaly
erythmatous ,dark skin lesion what is the diagnosis?

Zinc deficiency

Iron deficiency

B 12 vit deficiency

Niacin deficiency

57. Pt with autoimmune diseases ,from the following which one


suggest autoimmune hepatitis?

ASMA

ANA

ds DNA AB

AMA

58. Pt known alcoholic present with abd distension,on exam:


ascitis, spenomegaly hepatomegaly ,Inv : high ( Hb . Twbs .plt ) -
AST more than ALT , HBV + ve ,what is the diagnosis?

Alcoholic liver disease

Liver disease due to Hepatitis B

Budd chiari syndrome

59. Pt with recurrent peptic ulcer ,his father has history of


parathyroidectomy to recurrent stones , pt s.Ca normal , PTH
increased what is the cause of his symptoms?

Pheochromocytoma
VIPOMA

Gastrinoma

glucagonoma

60. Asymptomatic pt discovered to have diverticulosis

What is the next step ?

antibiotic

surgury

High fiber diet advice

Follow up

61. Pt presented with diarrhoea diagnosed as gardiasis recurrent


3 times inspite full ttt what investigation to do?

HIV

Immunoglobulins level

Celiac antibodies

62. Pt with feature of malabsorption , low ca and iron ,


antiendomeseal AB –ve what is the diagnosis?

B12 deficiency

Celiac

SBBO

Tropical sprue
63. Most serious complication of malaria is:

ARDS

Cerebral malaria

Black water fever

64. lymphocyte proliferation test used in :

malaria

leishmania

trypansomiasis

falirasis

65. Rabies is diagnosed before death by :

CSF culture

Saliva analysis

corneal ampression test

66. best ttt for lepra reaction type 2 is :

High dose steroid

thalidomide

stop leprosy medication

67. From the following which virus of haemorrhagic fever has


human to human spread:

Hanta virus
Crimean-Congo virus

Marburg virus

Dengue virus

68. Pt diagnosed with brucellosis has suspicion of TB also what is


the best ttt for brucellosis?

Doxacyclin ,rifampicin

Streptomycin, doxacyclin

Streptomycin ,rifampicin

69. Which helminth not need human for spread :???

Taenia

Echinococcus

70. Best tt for multidrug resistant TB?

Moxifloxacine,quinolone ,ethamputol,pyrazinamide?

Pyrazinamide,ethmputol,rifampicin,isoniazide

Pt recurrent malaria what is ttt is added after completion of


standard malaria ttt?

Quinine

Artemether

Primaquine

Mefloquine
71. Sowda disease best diagnosed by?

Diurnal blood film

Skin snip

72. From the following which drug associated with malaria


recrudescence?

Artemether

Mefloquine

Primaquine

73. traveler decide to visit schistosoma endemic area of Sudan


what is best drug to give?

Praziquantel

Artemether

Oxaminquine

Metrifonate

74. Ivermectin one dose is treatment of choice for :

Loa loa

Onchoserciasis

Ascaris

75. Which lead to increase urine PH?

hypoproteinemia
DKA

Klebsiella

76.Pt with liver disease ,how to assess his kidney function?

Urea

creatinine

cystatin C protein

chromogranin B

77.pt with diabetes has proteinuria on lisinopril 20 mg come for


follow up, he is asymptomatic, his BP140/90 , eGFR 42 , urine
with 3000 protien what is the next step?

Add ARBs

Increase the dose of lisinopril

Continue same dose

Stop lisinopril and give amlodepine

78 .which suggest tubular rather than pr renal AKI?

Urine osmolarity >350, urine Na <40

Urine osmolarity <350, urine Na >40

Urine osmolarity >350, urine Na >40

Urine osmolarity <350, urine Na <40

79. Pt presented with pulmonary edema , has decresed GFR , U/S


showed bulky kidneys what is the cause?
HF

Uremia

Amyloidosis

80. Familial hyperoxaluria associated with?

Stone formation

RTA

81. Pt known BPH on ttt , hospitalized for pneumonia then


developed increased urea and creatinine and palpable bladder ,
what is your action ?

CT abdomen

Catheter insertion

Renal biopsy

Abd. U/S

82.HIV pt allergic to sulpha , presented with SOB received ttt


during admission pt developed increased K and cratinine, low
HCO3, high CL ,what is the cause of his hyperkalemia?

Lactic acidosis

Rhabdomyolysis

Drug induced

83. Acid base status in Rhabdomyolysis ?

Hyperkalemia and metabolic alkalosis


Mixed respiratory and metabolic acidosis

Hyperkalemia and metabolic acidosis

Hypokalemia and respiratory acidosis

84. DM pt with sickle cell disease what is the test used for follow
up ?

HBA1c

Fructosamine

85. Female with amenorrhea for ,has prolactin 3000 , TSH high ,
T4 low , what is the diagnosis ?

Macroprolactinemia

Primary hypothyroidism

Pituitary microadenoma

86. Pt with pheochromocytoma first drug to control HTN ive crisis


is ?

Labetalol

Phenoxypenzamine

Doxazocine

87. Diabetic pt with many autoimmune diseases what is the best


investigation?

Anti GAD AB

Anti beta islet cell antibody


88. Obese female with hirsutism inv: high LH,FSH ,morning
cortisol slightly elevated and normal at midnight ,

normal urinary B 17 hydroxyprogestrone , What is the


investigation to confirm the diagnosis?

ACTH level

CT abdomen

Abdominal U/S

89. Young pt with HTN ,no abnormalities on examination,inv:


hypokalemic alkalosis , what is the diagnosis?

Cushing syndrome

Primary adrenal hyperplasia

Fibromuscular dysplasia

pheochromocytoma

90. Pt presented with polyuria polydepsia Underwent head


surgury before many weeks , S.Na high , urine osmolarity low ,
S.Ca 10 , glucose normal ,What is the most likely diagnosis ?

Primary hyperparathyroidism

Dm

Transient Cranial DI

91. In dermatomyositis, which of the following associated with


malignancy?

Skin ulcer
Increased CK

Anti Mi 2 AB

ILD

92. In RA which is correct ?

Suppluxation proximal radioulnar joint is common

Ruptured tendon is common

DIP joint more common affected

Heberden‘s nodes more common

93.Pt with SLE what is most specific ?

ANA

Anti smith

Anti centromer

Anto Ro ss

94. HTNive pt on aspirin presented with joint pain X ray revealed


intra articular calcification , what is the diagnosis?

Gout

Pseudogout

haemoarthrosis

95. Pt with DVT start warfarin , on day 2 developed brown


discoloration in buttock , plt 250 ooo , what is the cause?
HIT

Protein C deficiency

Anti thrombin III dwficiency

96. Pt with dark urine at the morning with h. of DVT , what is best
investigation?

Urinary porphyrin

Acid lysis test

(they didn’t mention flowcytometry)

97. Female with menorrhagia and easy bruises , her sister also has
menorrhagia , has high APTT and normal PT , what is the most
likely diagnosis?

Antiphospholipid

Von willebrand disease

Cristmath disease

98. Asymptomatic pt during routine investigation discovered has


TWBCs 35000, lymphocytes 31000, and has splenomegaly, also

CD 19, CD 20 , CD5 what is the diagnosis?

AML

ALL

CLL

Hodgking lymphoma
99. Sickeler on chronic opiates presented with severe abd. Pain
and jaundice , what is the most likely diagnosis?

Vaso-occlusive crisis

Cholecystitis

Opiates seeking

100. Female using OCP on investigation has very low platelet and
has no bleeding what to do ??

Platelet transfusion

Repeat test using heparinized tube


GREY CASES:

1. pt presented with joint pain , has history of recurrent hospital


admission with abd. pain which resolve after treatment, on
examination has knee joint swelling with effusion and tenderness

Aspiration of joint fluid clear (normal) ANA and RF negative

s. ferritin normal

 What is the diagnosis?

Familial Mediterranean fever

 Give two differential diagnosis ?


Rheumatoid arthritis
Adult onset still disease
 Investigation ?(they need one option)
Genetic testing
Metaraminol provocation test
 Management?
Colchicine
2. young pt presented with fever ,headache ,confusion for 2
days ,pt also presented with headache and fever before
1week received antibiotic but not improved ,pt has no
contact with pt of TB ,heavy smoker, on examination pt
slim and pale ,temp 38,ESR 45 has neck rigidity ,bilateral
brisk reflex and extensor planter reflex, other systems
unremarkable . Inv : CSF : high protein and very low glucose
and high total with lymphocyte predominant ,gram stain and
ZN stain both show no organism
MRI : enlarged ventricles with periventricular enhancement
and high intensity lesions more around temporal horn

 What is working diagnosis?


Tuberculous meningitis complicated by communicating
hydrocephalus

 What is the more specific investigation


PCR for mycobacterium tuberculosis

 Outeline of management?
Corticosteroid
Antituberculous
Transfer for neurosurgical center for consideration of
ventricular drainage

 Outline prognosis?
The mortality of tuberculous meningitis remains as high as
15-30 % .
3. pt presented with joint pain and cough with haemoptysis
Investigation : CXR show pulmonary infilterate , PFT show
restrictive pattern with high transfer factor .

 Give 3 differential diagnosis ?

Systemic lupus erythematous


Good pasture syndrome
Wegener granulomatosis

 Investigation?

ANA ,Anti DsDNA antibody


Anti GBM antibody
ANCA

 If pt has renal impairment how to manage?


IV methylprednisolone + cyclophosphamide
4. pt known of autoimmune hypoadrenalism on
corticosteroid and fludrocortison presented complain of
gaining weight and sleepiness on examination obese ,
bradycardic ,…

 What is the diagnosis?


Autoimmune hypothyroidism
 Give 3 investigation?
TFT:T4,T3,TSH
Thyroid antibodies
Thyroid US/ radioisotope scan
 In addition to his chronic illness what is the complete
diagnosis?
Autoimmune polyglandular syndrome type 2
 What is the treatment you have to add?
Levothyroxin
5. young male recently travel to Halfa presented with fever 2
days and bilateral lower limb weakness . On examination LL
hypertonia , hyperreflexia , loss of sensation till umbilicus ?
pt has eosinophilia
 What is the diagnosis?
Transverse myelitis due to schistosomiasis
 Give one differential diagnosis?
Potts disease of the spine
 Investigation?
MRI spine
Investigation for schistosoma (rectal snip)
 Outlines of management?
IV methylprednesolone then oral prednisolone
Praziquantel .
OSCE:

1. Pt with Atrial fibrillation

 What is the diagnosis?


Lingual artery thrombosis due to Atrial fibrillation

2.MRI brain of pt from South Sudan present with convulsion

 What is the diagnosis?


Neurocystocercosis
 What is the management?
praziquantel or albendazole + steroid
3.Describe ?

Extensive juxta-articular periostitis in DIPJs , (pencil-in-cup)


 What is the diagnosis?
Psoriatic arthritis

4.

 What is the diagnosis?


Charcot joint (neuropathic arthritis)
 What is the diagnosis if the pt has meiosis?

Neuro-Syphilis (tabes dorsalis)

5. Blood film of pt with confusion and renal impairment


describe ?

Anisocytosis ,reticulocytosis and fragmented RBCs (schisocytes)

 What is the haematological diagnosis?


Microangiopathic haemolytic anemia due to (TTP)
6. CT abdomen

 What is the finding?


Calcification of spleen and mesenteric lymph nodes
 What is the differential diagnosis?
Histoplasmosis, TB, sarcoidosis

7. Pt presented with fever and murmur ,

 What is the finding?


Erythema marginatum
 What is the diagnosis?
Rheumatic fever
 Mention other signs?
Subcutaneous nodule
Sydenham chorea
Migratory arthritis

8.

 What is the diagnosis?


Rt lung with total homogenous opacity (white out )
 What is the causes?
Total lung collapse
Pleural effusion
Malignancy
9. pt with hepatitis C infection

 What is the diagnosis?


Porphyria cutanea tarda

10. CT chest

mention the signs which can seen in this pt?


Raised JVP
kussmaul sign
pulsus paradoxus
(pericardial calssification _ constrictive pericarditis)
11.

 What is the diagnosis?


Ventricular bigeminy
 What is the cause?
IHD

12. Pt has developed this skin lesion 3 month after febrile


illness

 What is the diagnosis?


PKDL
 Management?
Sodium stibogluconate+ paromomycin
Amphotercine B
13.

 Give anatomical diagnosis?


varicella zoster (shingle) in T10 distribution
 What is the cause ?
HZV infection
14.

 this worms is removed after incision of pt skin


what is the diagnosis?
Myasis
 What is the ttt:
Petroleum cause asphyxiation of maggot facilitates
removal by tweezers

surgury
15.

 What is the diagnosis?


Vitreous haemorrhage
 What are the causes?
Proliferative diabetic neuropathy
Accelerated hypertension
Trauma

16.
 What is the diagnosis?
Diverticulosis
 Mention 2 complication?
Bleeding
Infection
Obstruction

17. Pt with normal TFT

 What is the diagnosis?


Pretibial myxedema
 What is the other signs of the underlying disease?
Graves ophthalmopathy
Graves dermopathy
18.

 What is the finding?


Gynaecnomastia
 Mention 2 neoplastic causes?
Testicular cancer
Estrogen secreting adrenal tumor

19.

(Actually the arrow was more superior near the superior edge of
scapula)
 What is the muscle?
Serratus anterior
 What is the nerve supply?
Long thoracic nerve (C5_C7)

20.

 Pt with epilepsy, What is the finding?


Strawberry haemangioma in distribution of
ophthalmic division of the trigeminal nerve

Gum hyperplasia due to phenytoin


 What is skull x.ray shows?

Cerebral calssification
AVM (carotid artery)
 What is the disease ?
Sturge Weber syndrome

‫مع تمنياتي لكم بدوام التوفيق وال تنسونا من صالح الدعاء‬

‫سبحانك اللهم وبحمدك‬


‫اشهد أن ال اله اال انت‬
‫استغفرك واتوب اليك‬
OSCERECALLMARS2021

N.B.Answer
smaynotbeaccur
ate

1-Whati
stheabnor
mal
it
y?2causes?

Er
ythemanodosum -hy
popi
on

TB-Sar
coi
dosi
s-ul
cer
ati
vecol
i
tis-Behcet
2-Whati
stheabnor
mal
it
y?Dx?

Cal
cinosi
s..Scl
eroder
ma

3-Whati
stheabnor
mal
it
y?2causes?

Bi
l
ater
alcar
pal
tunnel
..Amy
loi
dosi
s-Acr
omegal
y
4-Dx?2causes?

HCC.
.Chr
oni
cHBV/
HCV-Al
cohol
i
cli
verdi
sease..

5-Descr
ibet
heabnor
mal
it
y?2causes?

Bi
lat
eral
groundgl
assopaci
ti
esesept
alt
hickeni
ng(
crazypav
ing)
..Pul
monar
yal
veol
ar
pr
otei
nosis-COVI
D19
6-Whati
stheabnor
mal
it
y?2causes?

consol
i
dat
ion+?
?cav
itat
ion/
lungabscess

TB-St
aphaur
eus-Kl
ebsi
ell
a

7-Whati
stheabnor
mal
it
y?2causes?

Fat
tyl
i
ver
..Al
cohol
i
cli
verdi
sease-NAFLD/
NASH
8-ECG?t
tt?AFef
astv
ent
ri
cul
arr
esponse-Bbl
ocker
s

9-Whati
sthel
esi
on?Wher
eist
hel
esi
on/
Dx?

Lt12t
hCNpal
sy

10-Namet
hepar
asi
te&t
hef
ly?di
sease?

Onchocer
cav
olv
ulus-Bl
ackf
ly.
.Onchocer
ciasi
s(r
iverbl
i
ndness)
11-Whati
stheabnor
mal
it
y?2causes?

Rot
hspot
..I
E-Vascul
i
tis

12-Per
ipher
alBl
ood.
.3abnor
mal
it
ies?Dx?
?

Sci
stocy
tes.
.Hel
metcel
l
s..Hy
pochr
omi
a

?
?Hemol
yti
canemi
a
13-Descr
ibet
heabnor
mal
it
y?Dx?

Eggshel
lcal
cif
icat
ion-Si
l
icosi
s

14-Whati
stheabnor
mal
it
y?2cl
ini
calsi
gns?

LtCer
vical
rib.
.muscl
ewast
ing-absentorweakpul
se-r
aynaud'
sphenomenon
15-Whati
stheabnor
mal
it
y?Dx?

Thi
sptdev
elopedbackpai
n..Why
?

Ker
atoder
mabl
enor
rhagi
ca.
.React
ivear
thr
it
is

AsseAnky
losi
ngspondy
li
ti
s

16-Dx?2modesofpr
esent
ati
on?

Par
asagi
tt
almeni
ngi
oma(
cal
cif
ied).
.Spast
icpar
apar
esi
swi
thoutsensor
yaf
fect
ion-si
gnsof
hi
ghICP.
17-PH=7.
3-HCO3=13-S.K=2.
9

Whati
stheabnor
mal
it
y?Dx?

Nephr
ocal
cinosi
s..RTAt
ype1

18-But
cherpr
esent
edwi
tht
hisl
esi
on.
.

Whati
sthel
esi
on?Dx?

Eschar
..Cut
aneousAnt
hrax
19-Whati
stheabnor
mal
it
y?Dx?

Buccal
pigment
ati
on.
.Addi
son'
sdi
sease
RECALL BOF 5TH OCTOBER 2020

D. Hyder Ishag Adam


1-COVID 19 with which of the following has higher mortality :

a- Hypertention
b- Diabetes Millites
c- Ischaemic heart disease

2-COVID19, steroid (dexamethasone) has prognostic benefit in those following patients:

a- In ICU ventilated patients


b- Asymptomatic

3-Regarding to prevalence of COVID19:

a- 50% of population have symptoms


b- 5% of population have symptoms

4-which one of the following viral haemorrhagic fever transmitted by arthropode except

a- Dengue fever
b- Ebola
c- Yellow fever
d- Rift valley fever

5- Patient with mantoux test 7mm induration, in which statement we can anti TB:

a- Comorbid with HIV

6-What is the genetic factor that has preventive mechanism against plasmodium vivax

a-sickle cell trait

7-Malaria has causal and suppressive prophylaxis,which one of the the following is causal
prophylaxis

a-doxacycline

b- atovaquone/proguanine(malerone)
c- quinine
e- mefloquine
8-characteristic clinical feature of cavernous sinus thrombosis

a- Early papilloedema
b- Hyperaesthesia on frontal area

9-First line treatment of myoclonus

a- Sodium valporoate
b- Carbamazepine
c- Ethuximide
d- Levetiracetam

10- patient presented with recurrent blackout with immediate recovery and normal
investigations(ECG-CT brain-ELECTOLYTES)

a- Cardiac arythmias
b- Vasovagal attack
c- Addison disease

11-patient with crohn,s disease has neumoturea ,what is the best investigation to reach the
diagnosis :

a- CT abdomen
b- Barium swallow
c- Enterograhy
d- Sigmoidoscopy

12- Female has osteoporosis and received Hormone replacement therapy(tamoxifene) and
developed deep vein thrombosis then stop tamoxifene.which one of the following is
contraindicated :

a- Calcitonin
b- Alendronate
c- Etodronate
d- Raloxifene

13-orbicularis oculi ---lagophthalmos :caused by :

a- onchocerciasis
b- loa loa
c- wuchereria banchorufti
14- patient developed cholestatic jaundice after liver transplantation

a- common duct stricture


b- Ca pancreas

15-Nurse has exposed to HIV and has been given HIV prophylaxis ; follow up will be as
following

a- 6 weeks _12weeks and 6 months


b- 4weeks 6weeks and 8 months

16-Treatment of strongloides stercolaris

a- Albendazole
b- Praziquentil

17- Which one of the following ABGs is of stable COPD

a- PH7.35 pco2 po2

18- patient with eosinophilic esophagitis has complained of

a- Heart burn
b- Dysphagia
c- Wheezy chest
d- Skin itching

19-patient with chronic diarrhea and weight loss 5Kg for 4months and all investigations
including D-xylose are normal what is the diagnosis

a- coeliac disease
b- IBD
c- Whipple disease
d- Tropical sprue

20- patient received triple eradication of H.pylori .what is the most sensitive investigation for
follow up

a- Disappearance of symptoms
b- Urea breath test
c- Negative stool for H.pylori

21-patient with burn .CVP is normal ,deteriorating kidney function due to


a- Acute tubular necrosis
b- Hypovolaemia
c- Sepsis

22- 18years old presented with hypokalaemic hypocholoraemic metabolic alkalosis with
normal blood pressure investigation hypercalciuria and hypocalcaemia . what is the diagnosis

a- Barter syndrome
b- Gittleman syndrome
c- Renal tubular acidosis type 1
d- Renal tubular acidosis type 2

23- A lady is a known to have rheumatoid arthritis ,hyperlipidaemia, in postpartum period


,Rheumatoid arthritis is controlled with methotrexate ,hydroxychloroquinine,aspirin and
statin.Recently added NSAID conseguently RFT deteriorate then NSAID was stopped .Which
of the drug should be stopped :

a- Methotrexate
b- HCQ
c- Aspirin
d- Statin

24- patient with crohn,s disease received infliximab and developed deterioration of vision
and fundoscopy revealed haemorrhage along retinal artery .What is the treatment of this
condition that causing visual disturbance :

a- Ganciclovir
b- Photocoagulation

25- ASD with :

a- Ventricular tachycardia
b- Complete heart block
c- Atrial fibrillation

26- Scenario of typical presentation of multiple myeloma . what is the cause of renal
impairment:

a- Hypercalcaemia
b- Amyloidosis

27- What is the feature of severe aortic stenosis:


a- Ejection systolic murmur radiated to the axilla
b- S4

28- Patient with collapsing pulse and feature of pulmonary hypertention what is the
diagnosis:

a- Patent Ductus Arteriosus


b- Aortic stenosis
c- Mitral stenosis

29- Pt with clinical presentation of brucellosis and has renal impairment .what is the safer
regimen:

a- Doxacycline and streptomycin


b- Cotrimoxazole and doxacycline
c- Doxacycline and rifampicin

30- patient with lymphoadenopathy and blast cells on peripheral blood picture .what is the
best investigation:

a- Lymph node biopsy


b- Immunophenotyping
c- Bone marraw aspiration

31- Patient with the feature of IBD and has dilated biliary system on ultrasound . What is the
best next investigation :

a- MRCP
b- ERCP
c- CT abdomen

32- Patient with HIV and presented with CNS symptoms. CT brain showed homogenous
enhancing and given likely diagnosis .what is the treatment:

a- Radiotherapy
b- Surgery
c- Chemotherapy
d- AntiTB

33- What is the feature of tuberculous pericarditis :

a- Absent x descent
b- Absent v descent
c- Steep y descent

34- Patient with features of multiple sclerosis ;what is investigation

a- Visual evoked potential

35- pt with clinical presentation of multiple sclerosis what is most sensitive investigation

a. MRI brain
b. Lumber puncture
c. Nerve conduction study

36- Feature of lateral medullary syndrome what is the diagnosis

a- Left lateral medullary infarction


b- Right lateral medullary infarction

37- Patient with DM type 2 with the feature of nephritic syndrome and fundoscopy is normal
. what is the cause of nephropathy

a- Diabetic nephropathy
b- Membranoproliferative glomerulonephritis
c- Focal segmental glomerulonephritis
d- Membranous glomerulonephritis

38- Female in 34 weeks gestation presented with headache and normal BP . low HB low
platelet and reticulocytes 6%(MAHA) given likely diagnosis what is the best management

a- Plasma exchang
b- IV immmunoglobulins
c- Prednisolone
d- Magnesium sulphate

39- Pregnant lady presented with severe headache ,impaired liver functions and no
convulsion highBP .what is the treatment

a- MGSO4
b- Termination of the pregnancy

40- Feature of the polycythaemia rubra vera . what is the treatment

a- Venesection with aspirin


b- Venesection with hydroxycarbamide
41- Female teacher HBV positive ,she is worry and wish to know the state of viral replication ;
what is the best investigation

a- HBV DNA
b- HbeAg
c- HbcAb

42- Patient presented with haematemesis with low blood pressure after resuscitation the
patient became stable with normal blood pressure but still there is minimal bleeding .what is
the next best step:

a- OGD with banding


b- Blackmore sengstaken tube
c- Terlipressin

43-treatment of HCV

a- Tenofovir and ribavirin


b- Entacavir
c- Pegillated interferon alfa
d- Sofosbuvir/velpatasvir400mg/100mg

44- Clinical feature of hypothyroidism(which is the most obvious one) and mild raised
prolactin .what is the appropriate management

a- Give thyroxine for hypothyroidism


b- Dopamine agonist

45- 72 years old man presented with stroke .brain scan showed bleeding .

a- Amyloid angiopathy
b- Hypertention

46- Phosphodiestrase inhibitor 5(PD5),non responsive in case of :

a- Angiotensin converting enzyme (ACE)


b- Take with meal

47- which of the following is not induced by drug

a- Bronchiectasis
b- Pulmonary fibrosis
c- Pleural effusion
d- Pulmonary haemorrhage

48-Patient with inferior myocardial infarction developed pleuritic chest pain after 2days
(pericardial rub) . what is treatment

a- High aspirin dose


b- colchicine
c- Steroid

49- Patient with myocardial infarction received streptokinase and ECG received after lysis ST
elevation .5mm .what is the next appropriate step

a- Relysis
b- Percutanous Coronary Intervention
c- Tenectaplase
d- Alteplase

50- Patient with lipodystrophy and haematuria and proteinuria &low C3 .what is the
diagnosis

a- Membranous GN
b- Membranoproliferative GN
c- FSGN

51- Patient with systemic sclerosis and renal crisis given captopril and still RFT is impaired
.what is the next step

a- Stop captopril
b- Switch captopril to amlodipine
c- Continue with captoril and add prednisolone

52- Patient with dry eyes and mouth ,arthritis and erythematous rash

SSA,SSB,RF,ANA are positive Labda/kappa ratio is normal.what is the diagnosis

a- Sjogren,s syndrome
b- Mixed connective tissue disease
c- Rheumatoid arthritis with Sjogren
d- Systemic lupus Erythematosus

53- Patient with asymmetrical polyarthritis and mainly distal interphalangeal joint(DIP).what
is the diagnosis
a- Rheumatoid Arthritis
b- Psoriatic Arthritis

54- Patient with feature of posterior column affection(impaired vibration and


proprioception) and pupil reacted with only with to accommodation what is the diagnosis

a- Tertiary syphilis
b- Subacute combined degeneration of spinal cord

55- Patient presented with right lumbar mass ,hamaturia , abdominal pain and recurrent
urinary tract infection .what is best investigation to reach the diagnosis

a- CT abdomen
b- Abdominal us
c- Renal biopsy

56- Prosthetic valve endocarditis .what is the best regimen

a- Vancomycin and rifampicin


b- Benzylpenicillin for long time

57- Patient with recurrent ear and nasal bridge pain ,saddle nose CXR is normal .what is the
diagnosis

a- Wegener granulomatosis
b- Relapsing polychondritis

58- Patient with DM on metformin and NPH, his FBS 80 mg/dl HbA1c 7.5% and patient
experiences hypoglycaemic attack at the morning. What is the the next appropriate step:

a- Basal insulin 0.2 u/kg


b- Glargine insulin (lantus)
c- Premixed insulin 50 U once aday

59- Patient with goiter and has upper airway obstruction(stridor).what is the best test to
monitor his condition

a- Peak expiratory flow rate


b- Flow volume loop

60- pregnant lady with thyrotoxicosis and had relapse .what is the best treatment

a- Methimazole /carbimazole
b- Propylthiouracil
c- Radioiodine therapy

61- Patient with head trauma and features of hypopituitarism (low testosterone,low
aldosterone,high TSH)received corticosteroid and also has feature of hypothyroidism
(irregular cycle,constipation and weight gain).what is the next step

a- Increase the dose of corticosteroid


b- Give levothroxine

62- patient has atrial fibrillation presented with malena and taking warfarin so he received
vitamin K. what is an another management should be given

a- Fresh frozen plasma


b- Cryoprecipitate

63- patient on Rivaxaban and preparing for colonoscopy.Rivaxaban should be stopped before
procedure by

a- 2days
b- 3days
c- 7days

64- NOAC is preferable because of

a- No need for dose adjustment

65- Adjuvant treatment in severe malaria beside antimalaria

a- Gentamycin
b- Glandamycin
c- Levofloxacin
d- Amikacin

66- Patient presented with carcinoid tumour and CT abdomen showed left adrenal mass
1.6cm .what is the next step

a- CT guided biopsy
b- MIBG

67- Patient received medication including paracetamol and developed cholestatic jaundice
.which one of the following drugs is responsible

a- Paracetamol
b- Coamoxiclav
c- Phenytoin

68- Patient presented with productive cough ,fever,weight,esinophlia and CXR revealed
bilateral hilar shadowing .what is the diagnosis

a- Strongloides stercolaris
b- Allergic bronchopulmonary aspirogillosis

69- Patient with COPD on SAMA&SABA,referred to you complaining of dyspnoea.what is the


next step in management

a- Inhaled corticosteroid
b- LABA
c- LAMA
d- Non invasive ventilation

70- patient presented with acute attack of COPD&received medication&clinically patient


improved even SO2 94% but chest radiograph is worse than before at initial
presentation.what is the next appropriate step

a- CT chest
b- Change antibiotic
c- CXR

71- Patient with brainstem stroke and has dysphagia

a- Ambiguous nucleus

72- 30years old young female presented with behavioral changes and memory problem and
also she has fever.what is the best investigation

a- MRI brain
b- Lumber puncture

73- patient with haemolytic feature in CBC &has recurrent menorrhagia,Hb8g/dl,low


platelet,high LDH,high ferritin,low TIBC,MCV,HB electrophoresis show Hba1,HbF.what is the
diagnosis

a- Sickle cell trait


b- Iron deficiency anaemia
c- Thalassaemia minor
74-Patient with liver cirrhosis and tuberculosis started anti TB then liver
enzymes(transaminases)raised to double what is the plan of action

a- Stop antiTB
b- Stop INH&rifampicin and continue with ethambutol and pyrizanimide
c- Continue antiTB with the follow up after one month
d- Continue only with INH&RH

75- Patient diagnosed with Bell,s palsy and received prednisolone and acyclovir ,after three
months she came with left side of mouth twitching with blinking her eyes .what is the
diagnosis

a- Focal seizure
b- Facial nerve regeneration

76- Patient with autosomal dominant polycystic kidney disease ,worry about inheritance of
disease to his two children.what is the percentage

a- 0%
b- 25%
c- 50%
d- 75%

77- Patient complains of dyspnoea at night and received drug of weight reduction PH Po2
PCO2. What is the diagnosis

a- Obesity
b- Pulmonary hypertention

78- Patient underwent parathyrodectomy due to primary hyperparathyroidism ,then serum


calcium became low 7mg/dl& given calcium and vitamin D but not responding.what is the
diagnosis

a- Hungery bone disease


b- Secondary hypoparathyroidism

79- Risk factor of multidrug resistant TB

a- Comorbid with HIV


GREY CASES 6th OCTOBER 2020 RECALL

D. hyder Ishag Adam

CASE NO (1)

Building constructor complains of fever ,nausea and vomiting for 3


weeks.

O/E:Abdomen,chest and CNS are normal

Investigations:

Deranged liver functions mainly hepatocellular features and renal


functions are also impaired.

Hb 11g/dl TWBC 11.2 Platelet 100

BFFM &ICT for malaria &Widal test for entrica &brucella all are
negative

1_What is most likely diagnosis ?


Leptospirosis

2-Mention one differentional diagnosis ?


Viral haemorrhagic fever

3_Mention one sensitive investigation?


Blood &CSF culture or PCR for leptospira interogans

4_Mention specific treatment ?


Doxacycline or penicillin
CASE NO (2)

Young female from southern Kordofan presented with fever associated


with sweating and no rigors and also complained of palpitation,
exertional shortness of breath, dark urine and weight loss .She has
history of recurrent malaria

O/E:

The pt looks unwell pale not jaundiced

There is soft systolic murmur at apex

Mild splenomegally

Hb 9.5 G/dl TWBC & PLT are normal

ESR70mm/h reticulocyte count 1.5%

1-Mention two differential diagnoses ?


Infective endocarditis

Systemic lupus erythematosus

2- Mention three another clinical signs?


Splinter haemorrhage

Roth,s spot

Janeway lesion

3-Mention three investigations ?


Blood Culture

Echocardiography

Antidouble stranded DNA


CASE NO(3)

Elder man known diabetic on oral hypoglycaemic drugs and on good


control .he presented with unsteadiness,wide base gait,nystagmus and
vertigo. He is heavy smoker

O/E:

Disdiadodochokinesia,proprioception and diminished reflexes .

Planter reflex downgoing and no weakness. BMI 18.5

RBG 135 mg/dl HbA1c 6.5% MRI brain unremarkable

1-What is your full clinical diagnosis?


Paraneoplastic syndrome with cerebellar syndrome

2-What is underlying aetiology ?


Small cell lung cancer

3-Mention two relevant investigations?


Bronchoscopy with biopsy

HRCT chest

4-Mention two further approaches to reach the diagnosis?


Syphilis serology and serum B12 and folate

Anti HU
CASE NO (4)

Elderly man presented to A&E with acute centeral chest pain radiated
to the neck and back after an argument with his wife .

O/E: PR 120 BP 220/160

ECG was done revealed only tachycardia

His chest pain was relieved after given an analgesia but unfortunately
his chest pain recurred again after awhile .ECG was done again which
revealed ST elevation in V1---------V4

There is right hand pallor.BP190/100

1-What is your full clinical diagnosis ?


Aortic Dissection type (A) complicating with anteroseptal myocardial infarction and right hand
ischaemia

2-Mention three investigations ?


CT chest with contrast or MRI

Echocardiography

3-what is the appropriate management?


Control blood with betablocker(eg.labetalol iv)

Surgical correction
CASE NO (5)

Patient with HIV developed PCP and received septrin and developed
hypersentivity then switch to clindamycin and primiguine, his RFT is
impaired s.creatinine 1.8.Hb 10 plt 120

Then added dapsone for prophylaxis of PCP ,patient became dyspnic


So2 85% PH 7.32 HCO3 19 PCo2 33mmHg PO2 normal, cyanosed and
jaundiced .

Hb 7g/dl plt 100 retics count 10%

1-What is the most likely diagnosis?


dapsone induced methaemoglobinaemia ,possibly patient may have G6PD deficiency

2-What is the confirmatory test?


G6PD assay and serum level of methaemoglobulin

3-what is an immediate intervention?


Stop dapsone and primiguine(sulfur containing drugs)

Hyperbaric oxygen

Exchange transfusion

4-What is the treatment for long term?


Avoidance of sulfur containing drugs

Neublised pentamidine as prophylaxis of pcp

HAART continuation
RECALL OSCE 6th October 2020

D.Hyder Ishag Adam

1-
koilonychia and pallor

Ascaris lumbricoids
2-Picture

neurofibromas and café au lait on back

a-what is the diagnosis ?

Neurofibromatosis type 1

b-mention two causes of hypertention ?


-phaeochromatosis - fibromascular renal artery stenosis

3-picture

a-what is the diagnosis ?

cutanous larva migrans

b-mention two drugs ?

ivermectin and albendazole


4-ECG of patient with chronic cough

a-What is an ECG diagnosis ?

b-mention two causes of chronic cough ?

5-CXR :Aortograph

a-discribe :narrowing of the descending aorta(coarctation of the aorta)

b-mention three clinical signs?

a- Hypertention
b- Radiofemoral Delay
c- Rib notching

6-Picture

a-discribe the lesion ?

Two big frontal well demarcated ulcers

b-mention three causes?

a- Syphilis
b- Cutanous leshmaniasis
c- leprosy

7-ECG

a- What is an ECG diagnosis?


Inferior myocardial infarction

b-Patient developed hypotention ---what are causes of deterioration of patient within


48hours?

-Right Ventricular infarction

-Complete heart block


8-CXR

a-What are the findings?

Cardiomegally –upper lobe diversion –left atrial enlargement

b-what is the diagnosis ?

Left ventricular failure

9-CT CHEST
10-Picture shows :

a-What does the picture show?

Atrophic glossitis

b-Mention two causes of this sign?

Coeliac disease

Pernicious anaemia
11-CT ABDOMEN

a-what does it show ?

hydatid cyst with daughter in the left lobe of the liver

b-Mention two modalities of the management ?

-Albendazole for 1-3months

_PAIR procedure(puncture-aspiration-injection-reaspiration)
12-Blood film

a-what does it show ?

spherocytes-target cells –elleptocytes

b-What is the cause of abdominal pain?

Pigment gallstone causing cholecystitis

13-CXR

a-What are the findings?

Bilateral hilar shadowing-reticulonodular shadowing and cavity

b-Mention two causes?

-Tuberculosis

-Sarcoidosis
14-Two pictures

a- what is the diagnosis?


Leperomatous leprosy

b-Mention two drugs ?

-Dapsone - clofizamine
15-Pictures show

conjunctival haemorrhage and jaundice


a-Name the picture (b)-fly?

Aedes egypti

b-Mention three differential diagnoses ?

chikungunya

Yellow fever

Dengue fever

16-picture

a-what are the findings?

shows upper and lower lips pigmentation with blue and red colour involving teeth and
gum

b-what is the diagnosis ?

AML??
17-Picture

a-What does the picture show ?

Right scapular swinging

b-What is the diagnosis ?

long thoracic nerve palsy(serratus anterior weakness)


18-picture

a-What does the picture show?

Deviation of the uvula to the right side

b-what is the diagnosis ?

9th cranial nerve palsy


19-picture

a-what does picture show?

Right foot showing fourth toe gangrene with big toe vasculitic changes

b- Mention two causes ?


Diabetes Mellites-----Thromboangitis obliterans
Best of five question recall
MD exam July 2017
By : Batol Gurashi
1- 50 years old male presented to emergency room with severe central chest pain ECG
done in the ER revealed ST segment elevation in lead 2, 3 and AVF he was admitted to
the CCU given thrombolysis and become stable. Three days later he developed severe
SOB O/E he is dyspneic RR 30 b/min PR 100 b/min BP 70/50 with raised JVP chest
examination there is crackle all over, cardiovascular examination normal heart sound no
murmers.
What could be the cause of deterioration?
1- pump failure
2- rupture papillary muscles
3- VSD
4- cardiac tamponade
5- mitral reguirgitation
1- 42 years female diabetic patient for 15 years of average control presented complain of
upper limb pain for 6 weeks some time weakness in her manual work examination of her
upper limb revealed normal power but exaggerated reflexes she has positive tinel test more
in the left what is your diagnosis
1-mononeuritis multiplex
2-myelopathy
3-compression neuropathy
4-cervical radiculopathy
5-vasculitic neuropathy
2- 40 years obese lady, she took a medicine to lose weight presented with SOB O/E she is
morbidly obese dyspniec at rest chest examination is clear cardiovascular normal except for
loud second heart sound ABG done showed PO2 is 8 KPa and PCO2 is 7 KPa what could
be the diagnosis
1-primary pulmonary hypertension
2-secondary pulmonary hypertension
3-obesity hypoventilation
3- There is bacteria called wolbachia endobacteria discovered to have an effect of
development of loa loa , which of the following drug is used to treat this bacteria so as to
eradicate loaliasis?
1-co amoxiclav
2-doxycycline
4- 35 years pregnant female in the 34 weeks of her pregnancy presented with SOB for 2 days
she is previously well O/E she is dyspniec at rest PR 100b/min BP 100/70 raised JVP chest
examination there is bibasal crackles cardiovascular examination there is galloping she has
lower limb oedema she immediately started IV diuretics, in addition to diuretics which of
the following will be required for treating this patient

1-hydarlazine

2-ramipril

3-digoxin

4-bisoprolol

6-which of the following may need an adjuvant steroid in treatment

1-strongyloides stercolaris

2-loa loa

3- onchocerca volvolus

4-wuchereria bancrofti

7-which of the following is treated with single dose of ivermectin

1-loa loa

2-onchocerca volvolus

3-stongyloides stercolaris

4-ascaris lumbricoides

8- which of the following has some action against schistosomula

1-praziquantel

2-oxamniquine

3-artemether

4-metrifonate

9- 30 years old male with history of severe muscular disease he is bed ridden presented with
fever and cough O/E he is febrile temp 39 ,oxygen saturation is 92 chest examination there is
crackles what investigation you will immediately require

1-ABG

2-CXR

3-CRP

10-Which of the following medication is absolutely contraindicated in bronchial asthma

1-adenosine

2-verapamil
3-bisoprolol

11- 45 years male presented with his wife as she described him as being strange for three days
he developed odd behavior and he was wandering around the house without aim he has just
have flu like illness one week ago O/E he is confused not febrile no neck rigidity intact cranial
nerve normal chest cardiovascular and abdomen what is the drug of choice in his case

1-cefriaxone

2-cefotaxime

3-acyclovir

12-30 years old female nurse unfortunately she sustained needle stick injury from patient with
hepatitis B she presented I month later with fever, vomiting and jaundice LFT done ALT 700
AST 600 ALP 130 bilirubin 10 , which one of the following will be positive in this patient

1hepatitis B surface antigen and anti core immunoglobulin G

2- hepatitis B surface antigen and anti core immunoglobulin M


3- hepatitis B surface antigen and hepatitis e antigen
4- hepatitis B surface antigen and hepatitis e antibodies

13-we say this is multidrug resistant tuberclosis when the patient has resistance to

1- INH,rifampicin,pyrazinamide,ethambutol
2- INH and rifampicin
3- INH,rifampicin,pyrazinamide

14-46 years old laborer with flu illness 2 weeks ago presented with inability to stand

O/E flaccid paralysis reduced reflexes planter equivocal and sensation is normal

Investigations TSH less than 0.01 , what is the investigation of choice?

1-serum K

2-MRI spine

3-LP

4-EMG

15-which of the following is expected regarding a 68 years male type 2 DM diagnose with RTA
type 4

1- aminoaciduria
2- fludicortizone treatment is effective
3- increase GFR is expected
4- increase urinary bicarbonate
5- normal renal handling of K and H

16- 3o years old male working in pet shop presented with dry cough and severe SOB for 3 days
then he developed severe diarrhea and abdominal pain which of the following could be the
causative organism

1- legionella
2- H5N1

17-this is the iron study of the patient with anemia his serum iron is ….. (low) TIBC ….. (low)
serum ferritin ……..( high) what is likely cause of his anaemia

1-iron deficiency anaemia

2-sideroplastic anaemia

3-anaemia of chronic disease

4-hemolytic anaemia

18-35 years old female known case of antiphopholipid syndrome presented complain of severe
chest pain her ECG showed ST segment depression in V1 to V3 and her troponin is positive what
is the diagnostic investigation

1-coronary angiography

2-CTPA

3-echocardiography

19-65 years old male presented with chest pain ECG done in ER is shown below ( anterolateral
STEMI) troponin is positive he has history of stroke 3 months ago what will be the best
treatment

1-thrombolysis

2-emeregency Catheter

3-Heparin

20- Patient with COPD he has FEV1 of 33 according to the gold criteria for severity of COPD this
patient has

1-mild

2-moderate

3-severe

4-very severe
21-which of the following can be caused by plasmodium vivax

1-cerebral malaria

2-severe anaemia

3-acute renal failure

4-hypoglycemia

22- 66 years old male presented with fever and cough O/E he is conscious but febrile RR 30

BP 70/50 chest examination revealed signs of consolidation his urea is 11 mmol what is his CURB
score?

1-3

2-4

3-5

23- 30 years old male presented with fever and productive cough which is preceded by cold
sore around his mouth O/E he is febrile chest examination there is signs of consolidation in the
right lower zone what is the most likely organism ?

1- Staph aureus
2- Kliebsiella
3- Strept. Pneumoniae
4- Pseudomonas aurginosa

24-65 years old patient presented with painful right knee . he has history of congestive cardiac
failure to which he use to take hydrochlorothiazide , lisinopril and bisoprolol and spironolactone
O/E he is in pain has raised JVP chest has bilateral basal crackles and lower limb oedema
investigation uric acid 11 what is the best management regarding his knee problem ?

1- IM Diclofenac
2- IV colichicine
3- Intraarticular steroid

25- 35 years old female has history of sudden loss of vision twice she also developed left sided
weakness which resolved completely, which of the following will maximally help her remission

1-steroid

2- interferon alpha

3-alemtuzumab

4-glatimer
5-natalizumab

26- 55 years diabetic patient accompanied by his wife who said that he use to forget the things
surrounding him and keep asking same question many time during the day. His symptoms last
for the last six months no history of trauma, what could be the cause?

1-early demenitia

2-transient global amnesia

27-30 years old healthy patient who get trauma in his chest while playing football he is
asymptomatic chest x ray done in ER revealed pnemothorax with rim of air less than 2 Cm what
is your management

1-aspiration

2-chest tube

3-Reassure and discharge home review him in clinic later

28-25 years old female came complain of palpitation sweating and tremor which relieved by
eating food it occurred four times before O/E normal except for BMI 26

Investigation RBS 55 mg/dl insulin (normal) what will be your next plan

1-glucose monitoring at home

2- 72 hours fasting test

3-CT abdomen

29-54 years old male presented with right leg swelling for one day he has no significant history

Investigation proved right lower limb DVT . CBC Hb 13 gm/dl WCC 9000 plts 1,000,000 RFT
normal what is appropriate investigation

1-bone marrow trephine biopsy

2-protien electrophoresis

3-JAK2

30-15 years old boy presented with left knee swelling examination revealed knee effusion which
is proven to be blood when aspirated investigation done: CBC (normal), APTT (high) ,

PT (normal), bleeding time (normal), factor 8 (normal) what is the diagnosis ?

1-haempophilia A

2-haemophilia B

3-Vonwillibrand disease
31-56 years old patient presented to emergency room complain of haematemesis he has end
stage renal disease on haemodialysis . four unit of blood is prepared to be given and endoscopy
will be done , which of the following will be of prognostic value if given with blood

1-platelets transfusion

2-FFP

3-Vit K

4-octeriotide

5-cryopreciptate

32-know HIV patient presented with visual impairment fudus examination revealed exudate
alongside retinal vessel what is appropriate management?

1-ceftriaxone

2-ganciclovir

3-flucytocine

33-50 years patient who is known to have decompensated chronic liver disease on
spironolactone and laculsoe 30 ml three times a day presented with confusion and he has
positive flapping tremor what is your next action

1- Increase dose of lactulose


2- Rifaximine

34-what is true regarding leflunamide

1-relative contraindicated in pregnancy

2-irreversible alopecia

35- 34 years old diabetic patient presented with generalized body swelling 24 hours urinary
protein is 6 gms what is your management

1- ACE inhibitor
2- Prednislone
3- Renal biopsy

36-32 years old female presented for health checkup she is just taking Oral contraceptive pills
investigation CBC Hb 12 gm /dl WCC 5000 Platelets 21,000. peripheral blood picture showed
clumps of platelets what is your next action

1- Start prednislone
2- Give platelete
3- Bone marrow aspirate
4- Platelets antibody level
5- Repeat the CBC after putting blood in heprinized tube
37- 50 years old female who is taking chemotherapy for her breast malignancy presented with
generalized body swelling she has protienuria renal biopsy done it showed glomerualar
subepithelial deposition of C3 and Ig G what is the renal diagnosis

1- Membranous GN
2- Minimal change GN
3- Diffuse proliferative GN
4- Focal glomerulosclerosis

38-40 years diabetic male presented with haemoptysis CXR done revealed multiple cavities with
halo sign he has positive galactomannan test what is the diagnosis?

1- Aspergilloma
2- Invasive aspergillosis
3- Exterensic allergic alveolitis

39- 50 years old male presented with recurrent abdominal pain after meals and diarrhea bulky
stool for the last six months he also has significant loss of weight , he is alcoholic O/E nothing
significant investigation CBC Hb 13gm/dl MCV 105 WCC 5,000 plts 250,000 normal RFT normal
LFTs how you will confirm the diagnosis?

1-colonoscopy

2-CT abdomen

3-upper GI endoscopy

40- 30 years old male he has history of recurrent UTI presented with loin pain and haematuria
US abdomen done it showed multiple renal stone bilaterally what condition may this man
complain of

1- Medullary sponge kidney


2- Medullary cystic disease
3- Polycystic kidney disease
4- Horseshoe kidney

41- 42 years old male investigated for previous history of DVT and PE CBC Hb 9 gm/dl

WCC 2000 plts 100,000 how you will confirm the diagnosis

1- Acid lysis test


2- Blood immunophenotyping
3- Bone marrow biopsy

42- 24 years pregnant lady underwent checkup investigation her TFT is shown: ( normal FT3
normal FT4, low TSH (0.1) what will be your action

1- Start antithyroid treatment


2- Request antithyrotopin hormone
3- Reassurance
43- 41 years old lady incidentally found to have adrenal mass while investigated for other issue

She is asymptomatic , how to exclude subclinical cushing?

1- 24 hours urinary cortizol


2- Low dose dexametasone suppression test
3- Salivary gland cortizol level
4- Dexamethasone every 6 hours and measure cortizol level

44- 19 years college student who is absent from the class for three days found by his colleague in
his room collapsed on examining him he has tremor and his pupil is dilated sluggish reaction his
BP 140/90 PR 100 b/min what would be the cause of collapse?

1-ethanol intoxication

2-ectasy

3-CO poisoning

45- 53 years know HIV presented with forgetfulness his mininmental examination is 10 out of 30
his CT brain is normal what is your treatment?

1- Acyclovir
2- Start him HAART
3- Flucytosine

46- 43 years old known HIV who recently started HAART and treated treated for pneumocystis
carnii infection with pentamidine, is investigated for hyperkalemia He is found to have BP 90/60
RFT normal urea and creatinine, his K is 6 mmol Na 130mmol what is the likely cause?

1- addison disease
2- pentamidine side effect
3- HAART side effect

47- 80 years old female who had history of complex surgery for upper limb fracture presented
with right wrist pain and swelling she underwent diagnostic aspiration from her joint what do
you think it will be positive in this fluid

1- congo red dye


2- weakly positive birefringence
3- negative birefringence

48- 42 years female presented with loss of lipido O/E she has breast atrophy. Her last menstrual
period was 18 years ago when she delivered her last daughter who couldn’t lactate What will be
the cause ?

1- prolactinoma
2- pituitary apoplexy
3- autoimmune ovarian failure
49- 30 years old male newly discovered diabetic what will be positive?

1- Anti insulin antibodies


2- Glutamic acid decarboxylase

50- 65 years old male who underwent small bowel resection presented with loose stool his
investigation revealed CBC Hb 10 gm/dl MCV 110 WCC 6,000 PLTS 350,000 LOW B12
normal folate what is your diagnosis?

1- crohns disease
2- bacterial overgrowth
3- pernicious anaemia

51- 25 years old lady who is asymptomatic discovered to have mitral prolapse she will undergo
upper GI endoscopy for dyspepsia, what about antibiotic prophylaxis?

1- No need because it is minor valvular lesion


2- No need because it is minor procedure

52-54 years old presented 6 weeks after renal transplantation with fever and generalized
lymphadenopathy what is the most likely organism?

1- EBV
2- CMV

53-29 year’s old male presented with nausea and vomiting for 3 days O/E he has dry mucous
membrane investigation RFT urea 130 S.cr 3.5 k 5.6 urinary sodium 20 urinary creatinine 3000

What is the cause of his renal impairment ?

1- Acute tubular necrosis


2- Renal azotemia due to vomiting

54- which of the following its entry through the skin?

1- Ascaris lumbricoides
2- Loa loa
3- Strogyloides stercolaris

55- 47 years old with kidney transplant, he is on immunosuppressive therapy on tarcolimus


cyclosporine and steroid he presented with tonsillitis which of the following is contraindicated?

1- Ciprofloxacin
2- Erythromycin
3- Coamoxiclav

56- 28 years old lady known case of mythenia gravis on prednisolone 10 mg and azathioprine 50
mg presented with increasing difficulty of swallowing for three days and it reached the
maximum what is your next action?

1- Increase dose of steroid


2- Increase the dose of azathioprine
3- IV IG

57- which of the following complication of typhoid occur during the third week?

1- Myocarditis
2- Rose spot

58- 30 years female presented with dry eyes O/E she has little saliva and enlarged
submandibular glands which of the following will be positive

1- Anti SSA/Ro
2- Anti dsDNA
3- RF

59- 27 years old female presented with motion sickness she was given promethasine but it has
little effect what will be an alternative medications?

1- Hyosine
2- Ondasetron

60- what is true regarding new oral anticoagulant

1- Increase risk of bleeding


2- Difficult reversal of its effect
3- Contraindicated in valvular lesion

61- 50 years old male presented with progressive limb weakness for 3 months O/E fully
conscious and has intact cranial nerves examination of limbs there is wasting , power grade 2
with hypotonia and hyporeflexia planter reflex is mute bilaterally sensation is intact how u will
confirm the diagnosis

1- EMG
2- NCS
3- MRI spinal cord
4- Muscle biopsy
62- 21 years old female presented with high grade fever and rash for 3 days she has clear
medical background O/E she is very ill temp 40 BP 90/60 she has bleeding from her
hypertrophied gums and ecchymosis in her skin chest CVS abdomen normal examination
investigation revealed Hb 8 gm/dl wcc 2000 Plts 80000 fibrin degradation product is high and
fibrin( low) , how would you will reach the diagnosis

1- Peripheral blood smear


2- Bone marrow examination
3- Protien electrophoresis
63- 40 years old male has history of chronic sinusitis and recurrent upper respiratory tract
problem O/E nothing apart from saddle nose, investigation : Urea 200 mg/dl Scr 6 mg /dl urine
analysis contain RBCs what is the likely diagnosis?

1- Goodpasture syndrome
2- Wegeners granulomatosis
3- Churg strauss syndrome
64- 30 years old female presented with worsening renal function she has history of upper
respiratory problem examination nothing significant investigation RFT urea 180 S. cr 5 urine
contain RBCs and proteins p ANCA (antimyeloperioxidase) is positive what is likely diagnosis?

1- Wegeners granulomatosis
2- Churgstrauss Syndrome
3- Microscopic polyangitis
4- Goodpasture syndrome
5- PAN
65- what is true regarding hyperreactive malaria syndrome

1- Common in male
2- Increase in immunoglobulin G
3- Treated with quinine for years
66- 25 years old pregnant lady presented with vomiting and severe palpitations she has history
of weight loss for 6 months O/E she is febrile with dry mucous membrane PR 155 b/min BP
90/60 what is your immediate action?

1- Iv fluids and antiemetics


2- Iv dexamethasone
3- Iv fluids and propyothiouracil
67- 34 years old lady presented with and she had decrease level of consciousness and agitation
she had history of weight gain and constipation for 6 months examination she is obese confused
and irritable temp 35.5 PR 62 b/min chest cardiovascular and abdomen examinations are
normal what is your immediate management?

1- IV Dexamethasone

2- IV thyroxine
3- IV dextrose
68- 70 years old lady presented with fatigue and she is unable to raise her arm to comb her hair
investigation done S Ca low, phosphate low , Alp high what is your management ?

1- Ca supplements
2- Vit D and Ca supplements
3- Steroid
4- Hormone replacement therapy
69- 40 years old lady who is known case of chronic viral hepatitis presented with purpuric rash
her investigation showed RF positive and low c4 what is the diagnosis

1- PAN
2- Cryoglobulinemia
70- Gut associated lymphoid tissue treatment in patient with history of dyspepsia

1- Helicobacter eradication
2- Chemotherapy
3- Radiotherapy
4- Surgery
71- patient with fatigability more pronounced in the evening underwent investigations CXR
showed mediastinal mass what investigation you will request

1- Anticholinesterase antibody
2- Ct chest
3- NCs
72- pregnant lady known case of rheumatoid arthritis presented with painful swelling in her
fingers joint what treatment you will start

1- Sulphasalazine
2- Methotrexate
3- Leflunamide
73- patient has hypopigmented skin lesion on treatment for it presented with redness and
swelling in the site of the skin lesion what is the cause

1- Leprae reaction type 1


2- Leprae reaction type 2
74- Patient has hypopigmented skin lesion on treatment presented with pain and tenderness
over the site of lesion and nerves he was given steroid but it has little benefit what is the
additional drug you will add
1- Clofazimine
2- NSAIDs
75- Patient known to have coarctation of aorta who was treated in his childhood presented
with fatigue and dyspnea examination Bp 130/50 pulse large volume has systolic murmer
what will be the cause
1- Bicuspid valve with AR
2- Bicuspid valve with AS and AR
3- Recurrence of coarctation
76- 17 years lady presented with syncopal attack for the last 3 months it last for seconds
while standing it preceeded by dizziness and disorientation her sister state that the last
attack continued for 2 minutes ended with complete recovery what is your investigation
to know the nature of the attack
1- EEG
2- 24 hrs holter monitoring
3- CT brain
4- Vestibular testing
77- Patient with schizophrenia presented with high grade fever and rigidity O/E he is febrile
temp 40 and has rigidity all over his body what is your immediate action?
1- Diazepam
2- Dantrolene
78- Why pyridoxine is given with INH ( mechanism of action )
1- INH inhibit conversion of methionine to cystathionine ??????????
2- ?????????????????????????
79- Patient with antiphospholipid syndrome developed DVT and was put on warfarin with
target INR 2 to 3 developed another DVT what is your plan?
1- Lifelong warfarin with target INR 2-3
2- Lifelong warfarin with target 3-4
80- 15 years girl presented with haeamturia and abdominal pain she has purpuric rash over
the extensor surafce of her lower limbs what is diagnosis?
1- cryoglobulinemia
2- Henoch schonlien purpura
3- PAN
81- 60 years old male presented with palpitations he is not diabetic nor hypertensive he has
no history of stroke nor TIA O/E he is not dyspiec PR 82 b/min, irregular irregular chest
cardiovascular and abdomen examination is normal what is drug of choice
1- warfarin
2- Aspirin
82- 30 years female presented with fatigability and epistaxis O/E she is pale no
lymphadenopathy no organomegally investigations CBC Hb 8.6 gm/dl Wcc 5,000 Plts
80,000 coombs test positive what is diagnosis?
1- Autoimmune haemolytis anaemia
2- Evans syndrome
3- SLE
83- 30 years old patient presented to emergency as he is involved in road traffic accident he is
asymptomatic however his CXR showed that his left hemidiaphragm is elvated how you
will confirm the diagnosis?
1- Pulmonary function test
2- ABG
3- Sniff test
4- Pulse oximetry
5- CT scan for his chest
84- Patient with symptoms and signs of fibromyalgia what is the treatment?
85- Patient with symptoms of anaemia and feature of autoimmune disease (Autoimmune
haemolytic anaemia) what next investigation?
Antiglobulin test

‫مع تمنياتي للجميع بالتوفيق‬


MD exam internal medicine smsb second part
Recall of July 2018 exam (by anonymous)

Please note that ALL information mentioned in this document, questions and
answers are subject to either mistake, wrong answer or maybe inaccurate
recall.
Use accredited updated references to study the overall subject and diseases
in each question.
MCQs were documented as summary with positive findings only with the
suggested answer after reviewing the latest oxford, pastest, medscape,
Sudanese and WHO protocols and uptodate for each question.
MCQs tricky and updated
Study diseases mentioned in all questions from updated references and
guidelines and latest local Sudanese and WHO protocols. They may present
in future exams around the topics.
Search accredited websites and references to view many pictures for each
OSCE question

Please pray for all who helped collect, write and distribute this document to
all candidates.

Best to all candidates.


Grey Cases MD July 2018 Recall

Case (1)

30s yrs male


few days history of abdominal distention and pain
history of oral aphthos ulceration in the past
now has tender hepatomegally and scrotal ulcer
CBC LFT unremarkable
1
what is the diagnosis of the current condition
2
how can you confirm the diagnosis
3
what is the underlying diagnosis
4
mention 2 management options for this patient

Case (2)

20s yrs male


known IV drug user
has fever anorexia wt loss and cough for the last month
now high grade fever
temp 38c
pansystolic murmur left sternal edge
scattered crackles bilaterally on auscultation
WBC high (~13)
RFT normal
CXR multiple rounded opacities with ill defined margins noted
1
what is the most likely diagnosis
2
mention another possible diagnosis
3
mention 3 relevant investigations to support your diagnosis

Case (3)

60 yrs male history of recurrent choking


no history of cranial nerves symptoms
toung deviated to the left with tongue wasting on its left side
uvula deviated to the right
reduced palatal movement on the right
reduced power of right fingers extension grade 3
reduced power of left quadriceps grade 3
extensor planter response on right only
1
what is the most likely diagnosis
2
mention one investigation to help reach the diagnosis
3
what is the next management step

Case (4)

30s yrs male seen at the clinic after workup investigation


past history of hypertension on antihypertensive
examination is unremarkable
CBC unremarkable plt slight low
Alb 3 AST 25 ALT 30 ALP RFT normal
Abd u/s irregular liver size 6cm
fibroscan showed grade 3 fibrosis
HBsAg positive
HBeAg positive
HBc IgG positive
1
what is the diagnosis
the pt came back after several months for followup complaining of fatigue
AST 450 ALT 450 Alb 3 ALP normal INR 2.1
2
mention 2 possible causes of his second presentation
3
mention 4 investigations to support your diagnosis

Case (5)

30s yrs female with progressive exertional SOB for that last one year now
has SOB on minimal exertion
dizziness when standing up from sitting position
past history of pulmonary embolism several years ago given anticoagulation
completed 6 month
has 2 children their pregnancies were normal delivery
BP RR PR O2Sat all normal
soft systolic murmur on the left sternal edge
loud p2 APTT57
1
what is the diagnosis of her current presentation
2
mention 2 possibilities for the causes
3
mention 4 relevant investigations

Suggested Diagnoses for Grey Cases

1
Budd chiary syndrome acute hepatic vein thrombosis secondary to behcets
disease
2
Infective endocarditis vs staphyloccocal pneumonia
3
?MND
?Pseudobulbar palsy
?Balbar palsy
4
Compensated liver cirrhosis CHILD A, secondary to chronic hepatitis B
virus (now EASL phase2)
complicated by HCC vs Hep Delta virus super infection after excluding Hep
B flare
5
Symptomatic secondary pulmonary hypertension
caused by either chronic thromboembolic disease associated with lupus
anticoagulant antiphospholipid syndrome
or
?ASD ?Factor V leiden disease ?SLE

OSCE MD July 2018 Recall

1
CXR left sided total white out trachea deviated to the left
this pt complains of fever for several months
-what is the abnormality
left total lung collapse
-what is the most likely cause
pulmonary tb

2
ECG st depression v2 v3 v4 with doniment tall R wave
also st depression in II III AVF
reverse tick sign? Subtle irregularity
this pt complain of discomfort
-what is the abnormality
?posterior STEMI ?Digoxin toxicity
-what is the immediate management
?PCI ??Digibind

3
pic of pellagra rash on neck v shaped
this pt presented to TB followup clinic
-what is the diagnosis
Pellagra
-what is the cause
Niacin deficiency

4
barium follow through of a pt showing narrowing and filling defect in the
terminal ilium
-what is abnormality
terminal ilium filling defect? stricture?
-mention 2 causes
chrons dis abdominal TB

5
pedigree of disease inheretense only males are affected (some males are not
affected) and there is skipping of entire generation
-what is the mood of inheretense
?xlinked recessive
-mention 2 disease examples
G6PD Hemophilia Duchene

6
fundoscopy showing typical BRAO
-what is the fundus abnormality
BRAO
-mention 3 relevant investigations
Echo carotid Doppler inflammatory markers temporal artritis investigation
7
peripheral blood film showing typical hyper segmented neutropihils
this pt Hb9 mcv105 plt200 wbc6
-mention 3 abnormalities
hypersegmented nuetrophils hypochromic RBCs macrocytic RBCs
polychromatic RBCs
-what is the hematological diagnosis
?macrocytic anemia or ?megaloblastic anemia

8
CT chest
showing homogeneous hyperdensity of most of rt lung with marked air
bronchogram extending from medially and small subtle not obvious
cavitations with small fluid level ? abscess
-mention 2 abnormalities
?consolidation air broncho
?cavitations fluid level abscess
-mention 2 possible causes
?staph pneumonia
?pulm TB

9
pic of rt hand of white female with subtle hyper pigmentation in knuckles
and ???proximal interphalengeal joint swelling??
this pat complains of dizzy spells
-what is the abnormality
-mention 3 relevant investigations
?Addison’s inv
?Rh arthritis inv

10
eyes pic of left horners signs
cxr with cervical rib
-what is face abnormality
left horner syn
-what is x ray abnormality
cervical rib ??fractured
-what physical sign to find in limbs
Absent radial pulse

11
pic of pt knees with multiple skin colored lesions around patella and tibial
tuberosities
this pt has diabetes and complains of abdominal pain
-what is the abnormality
xanthomas
-what is the cause of abdominal pain
mesenteric ischaemia

12
pic of pt face
-mention 3 abnormalities
saddle nose deformity thickined skin loss of eye brows hair bilaterally
-what is the diagnosis
leprosy (most likely LL)

13
pic of brain imaging with multiple ring enhancing lesions
-what is the most likely diagnosis
cerebral toxoplasmosis
-mention another diagnosis
cerebral lymphoma
-what is the choice management of the likely diagnosis
sulphadiazin and pyramethamin

14
pic of face with malar and nasal reddish lesion rash and pic of chest x ray
-what is the face abnormality
lupus pernio
-mention 2 x ray abnormalities
bilateral hilar lymphadenopathy and bilateral infiltration
-what is the diagnosis
sarcoidosis

15
pic of pt leg with multiple different sized round reddish raised lesions from
knee to ankle involving front and sides of the leg
this pt complains of dairhoea and abdominal pain for several month
-what is the abnormality
erythema nodosum
-what is the underlying diagnosis
inflammatory bowel disease

16
pic of pt heels with subtle redness on both heels and pic of pelvic xray
-what is the lower limb abnormality
Achilles tendonitis
-what is the hip xray abnormality
sacroiliitis
-what is the diagnosis
ankylosing spondylitis

17
pic of both pt hands and feets together showing all nails with obvious pedal
clubbing and no obvious finger clubbing
-what is the abnormality
?pedal clubbing without finger clubbing?? HPOA???
-what is the cause
?PDA with right to left shunt?? squamous cell ca lung??? idiopathic
pulmonary fibrosis????? bronchiectasis

18
hand pic with clawing and multiple fingers and palm ulcerations
-mention two abnormalities
clawing of the hand multiple traumatic lesions and ulceration secondary to
loss of pain abd temprature
-what is the diagnosis
syringomyelia

19
pic of pt mouth with dark pigmentations in the lips and around the mouth
and pic of endoscopy showing numerous small yellowish lesion visible
though out the mucosa
-mention 2 abnormalities
dark lip and mouth pigmentation multiple hamartomous polyps
-what is the diagnosis
puetz jegar syndrome
-mention 2 management options
tonics iron supplements surgical polyp removal surveillance genetic
screening

20
chest x ray with clear dexrocardia
-mention three abnormalities
dextrocardia bilaterally cystic honey combing bronchiectatic changes
bilateral infiltration loss of gastric bubble on the right side
-what is the diagnosis
kartagener syndrome
MCQs MD July 2018 Recall

1
oncocerciasis best investigation
-skin sample

2
ascaris best treatment that gives best results
-Albendazole

3
pt with enlarged nerve what is leprosy best investigation
-skin biopsy to visualize bacilli

4
best treatment for pt with scabes and ascaris together
-Ivermectin

5
best treatment for erythema nodosum leprosum
-thalidomide

6
pt with muscle pain and swelling but no weakness
-trichenellosis

7
pt with eusinophilic respiratory symptoms what is the cause
-Ascaris
8
pt with visceral lieshmaniasis given full treatment and improved after
6month he developed sysmptoms what is the best investigation for
lieshmaniasis for him
-lymphnode biopsy

9
pt had contact with TB pt now his mantoux is more than 10mm CXR normal
what is the next step for him
-induced sputum for acid fast bacilli

10
which of the following is a sign of sever malaria
-WBC more than 12 (WHO sever malaria handbook 3rd edition page14)

11
pt post inferior MI developed cardiogenic shock picture without bradycardia
his PCWP via central line is 4 (lower limit of normal) what is the best initial
management
-normal saline

12
pt referred from psychiatry with polymorphic ventricular tachycardia what is
the best treatment
-IV magnesium

13
asthmatic pt with picture of churge strauss what is the drug that caused
condition worsening
-montilukast

14
pt with long duration dyspeptic symptoms not responding to PPI now OGD
done showed two ulcers in the duodenum him gastrin level is 70 (high) what
is the diagnosis
-H pylori infection

15
pt with CLL wbc was 50000 more than 3 month ago now wbc is 79000 what
is the management
Chlorambucil

16
pt with CLL developed CVA MRI brain showed multiple none enhancing
lesions in one cerebral hemisphere what is the diagnosis
-vessel vasculitis

17
female pt with relapsing MS on beta interferon but wants to switch to
another treatment that help reduce her relapse while having the most
minimal side effects what is the best treatment for her
-glatiramir

18
pregnant pt with picture of carpal tunnel had similar episode in her last
pregnancy treated successfully with splinting what is management now
-splinting

19
pt with COPD on salbutamol prn inhaler only FEV more than 60% FVC
more than 60% what is the next step in management
-LABA

20
pt 3 months post kidney transplant with constitutional symptoms and
deranged liver enzymes what is the diagnosis
-CMV
21
pt post thyroid surgery low ca and normal PTH what is the diagnosis
-hungry bone syndrome

22
young male with pathological fracture and small testes low testosterone and
bone score of -3 what is the management
-alendronate

23
old male pt long standing diabetic has chronic anemia in last 3 yrs ranged
Hb readings around 9 now Hb is 9 and normal MCV and high ferritin and
low transferin saturation and low transferin what is the diagnosis
-anemia of chronic disease

24
pt with liver cirrhosis and HCV positive and HBV negative what is the
treatment
-sofosbuvir and ledipasvir

25
pt with repiratory symptoms 3 days ago now has haematuria what is the
diagnosis
-IgA nephropathy

26
pt with purpuric lesions in lower limbs and RBC in urine what is the
diagnosis
-henoch schoenlien purpura

27
pregnant pt with headache and confusion high urea high creatinin low
platelet low Hb no sig deranged enzymes what is the diagnosis
-TTP
28
pt with fatigue has joint pain and left hypochondial pain has spleenomegally
and pancytopenia what is the diagnosis
-felty syndrome

29
pt known case of myelodysplasctic syndrome on regular monthly blood
transfusion target to keep his Hb above 10 now has picture of iron overload
now Hb is 10 what is the management
-iron chelating agent desferoxmin

30
pt developed SOB post colectomy has history of membranous GN what
investigatiin to confirm the diagnosis
-CTPA

31
younge male with headache painfull red eye and mouth ulcer CSF
leukocytosis what is the diagnosis
-behcets disease

32
young female with progressive fatigue and SOB and diplopia on lateral gaze
what is the investigation of choice
-acetylecholin Ab

33
male pt with choking on swallowing facial weakness and in coordination
loss of pain and temperature in both lower limbs what is the diagnosis
lateral gaze impaired
-?
34
male pt type 2 diabetes on metformin and pioglitazone had MI several
months ago now has SOB and edema HA1c 7.4 BMI28 what is the
appropriate management
-switch to empagliflozin

35
elder male came for investigation protien 9 albumin 2.9 bone marrow less
than 10% plasma cells what is the diagnosis
-MGUS

36
psychiatric pt on lithium given thiazide for hypertension what will happen
next
-muscle twitch

37
elder pt recently giver coamoxclav for respiratory infection now has watery
greenish offensive dairhoea colonoscopy showed colitis what is the
management
-oral metronidazole

38

stroke pt given statins and anti TG treatment came with muscle pain CK
doubling and high transaminases what is the diagnosis
-drug induced myopathy

39
pt with picture of ureteric colic pH7.2 k2.2 urine pH6.7 abdominal xray
showed calcification on right and left upper quadrants what is the diagnosis
-distal type 1 RTA
40
pt with IBD underwent bowel resection now has abdominal pain radiating to
the groin high MCV what is the diagnosis
-bacterial overgrowth

41
pt with picture of acute fever skin rash in lower limbs hepatospleenomegally
what is the diagnosis
-acute schistosomiasis

42
young female with recurrent attacks of hypoglycemia relieved by glucose
her mother has type 2 diabetes started supervised prolonged fasting but
stopped when sever hypoglycemia occurred and measured insulin
21(upto20) cpeptide 2.2(upto2.0) proinsulin is pending what is the next step
-measure urinary sulfonyleurea

43
elder male with wt gain and picture suggestive of cushings what is the
diagnosis
-small cell lung cancer

44
young female found to have prolonged APTT57 which does not correct after
50:50 mixing with plasma what is the diagnosis
-lupus anticoagulant

45
young male with acute paraparesis with sensory level and sphinctric
disturbance reduced reflexes and flexor planter response what is the
diagnosis
-idiopathic myelitis
46
pt with flushing reduced level of consciousness and diarrhoea and skin rash
what is the diagnosis
-pellagra

47
prosthetic valve pt presented with very high INR and gross haematuria given
3 units of plasma then developed picture of acute pulmonary edema and
respiratory distress what is the diagnosis
-TRALI

48
young female with progressive SOB right limb BP 200/90 left limb BP
170/90 absent carotid pulse what is the diagnosis
-takayasu disease

49
pt with 1yr duration chronic cough bibasal crackles hepatomegally what is
the diagnosis
-sarcoidosis

50
pt with picture of haemochromatosis what is the followup test during
treatment
-ferritin

51
pt with picture of NASH what is the associated condition
-insulin resistance

52
pt had history of irregular pulse now presented with hemiplegia and sinus
pulse TSHlow T3normal T4normal what is the current status
-subclinical hyperthyroidism
53
pt with picture of granulomatosis with polyangiitis and pulmonary
haemorhage what is the treatment
-steroid plus cyclophosphamide

54
pt asthmatic pt with joint pain and skin rash what is the diagnosis
-churge straus syndrome

55
female pt right knee left ankle pain and hand joints pain and tender abdomen
hand tenderness on tendon insertion but no actual joint swelling what
investigation to reach a diagnosis
-?

56
pt with picture of infective endocarditis complicated by heart failure started
treatment for IE blood culture revealed staf what is management
-refer pt for valve replacement

57
pt with chronic heart failure on multiple treatment and followup now has
wide QRS what is the next step
-biventricular pacing

58
pt with mitral stenosis now is symptomatic has pulmonary hypertension and
valve area is 0.9 what is the next step
-mitral valve replacement

59
pt with picture atypical of atrial mexoma what is the treatment
-surgical resection of the mass
60
pt recently started on phenytoin for secondary epilepsy now has weakness
and cerebellar signs what is the cause
-phenytoin toxicity

61
young pt with HepB virus infection and CLD HbsAg and HbcIgG positive
what the recommended approach in HCC survalence
-alfa fetoprotien every 3 months

62
pulmonary function test FEV FVC FEV/FVC KCO
picture of restrictive defect with high KCO what is the diagnosis
-?

63
pt with picture of systemic sclerosis and FEV FEV/FVC pulmonary function
suggestive of restrictive defect with normal KCO what is the diagnosis
-chest skin restriction tightness

64
pt with ABG low pH and high PCO2 and high HCO3 what is the balance
disturbance
-partially compensated respiratory acidosis\

65
pt with low oxygen saturation and normal paO2 what is the diagnosis
-methhaemoglobinemia

66
pt with picture of polycythemia had blood sample sent from his GP arrived
and showed s.K6.4 what is the next step
-repeat serum potasium again at the hospital
67
pt known case of CKD on regular treatment and followup presented with
picture of troublesome peripheral neuropathy now s.cr near 10 what is the
best management
-prepare to initiate and continue regular haemodialysis

68
alcoholic pt with epigastric pain and loose motions serum amylase is not
high abdominal xray showed calcification what is the treatment
-pancreatic enzymes suppliments

69
pt known hyperthiroidism on carbimazole on start of surgery developed
fever 40c rigidity what is the diagnosis
-thyroid storm

70
pt with long standing diabetes has picture of nephrotic syndrome apart from
ACEI what treatment to offer
-pridnisolone

71
symptomatic pt with picture of polycythemia rura vera what is the best
management
-venesection plus aspirin

72
pt with past history of irregular puls now has dark face color what is the
diagnosis
-amiodarone toxicity

73
pt with pseudomonus infection picture what is the best antibiotic
-?
74
pt with parkinsonian picture and gaze palsy what is the diagnosis
-progresive supranuclear palsy

75
pt with multiple finger infarction and picture of rynauds what is the
diagnosis
-buergers disease

76
old pt with anemia mouth ulcers and diarhoea what is the diagnosis
-coeliac disease

77
pt with picture of type 2 HIT platelet less than 100 what is the best
management
-stop heparin and start danaparoid

78
pt with hypogonadotrophic hypogonadism and anosmia what is the
diagnosis
-kallman syndrome

79
elderly pt with past history of treated subdural haematoma now has episodes
flucuating level of consciousness lasting for 30min what is the diagnosis
-subdural haematoma

80
pt with picture of familial hypocalcemic hyper calcuria what is the
management
-no treatment required
81
pt known hypertensive on thiazide still high blood preasure what is the next
step
-?

82
pt with APCKD has positive family history of APCKD with cerebral
aneurysm what is the nest management step
-screen pt with MR angiography

83
pt with picture of SIADH and symtomatic hyponatremia persisting despite
fluid restriction what is the next step
-desmopressin

84
pt with recurrent frontal headache which now occurs every day what is the
diagnosis
-paroxysmal hemicrania

85
pt with recurrent UTI given antibiotics now urine has no growth only
leukocytes in urine what is the diagnosis
-urinary TB

86
pt with picture of ITP not responding to medical treatment now platelet
count is very low what is the next step
-eltrombopag

87
which of the following TB associated conditions by caused by
hypersensitivity
-phlyctenular conjunctivitis
88
pt with picture of dupuytrens contracture which drug can be responsible
-phenytoin

Clinical SPACES MD July 2018 recall

Cycle 1 Panel 1
1. Long case CNS spastic paraparesis + peripheral neuropathy PMH of Ca
prostate
2. MS + pulmnary Htn
3. Hepatosplenomegally Thalassemia
4. Lung fibrosis
5. Scleroderma
6. Emergency acute respiratory destress (asthma vs COPD, pneumothorax,
pulmonary edema, PE)
7. Communication pt with IDA on recurrent blood transfusion refusing
invasive investigation GIT consultant want more investigation talk to patient

Cycle 1 Panel 2
1. Long case CNS spastic paraparesis with sensory level + dorsal column
PMH of RA on immunosupression and steroids
2. CLD spleenomegally ascitis
3. Lung fibrosis
4. Mixed aortic valve disease + Mixed mitral valve disease (4 lesions)
5. Rheumatoid hand
6. Emergency same as prev panel
7. Communication same as prev panel
Cycle 2 Panel 1
1. Long case CVS Rheumatic heart disease
2. CLD + portal Htn
3. Lung fibrosis
4. Flaccid paraparesis with sensory level
5. Acromegally
6. Emergency acute gouty monoarthritis in HF pt on treatment
7. Communication newly +ve HBV BBN counceling

Cycle 2 Panel 2
1. Long case CVS Rheumatic heart disease + thromboembolic complication
2. CLD + portal Htn
3. Bronchiectasis
4. Unilateral cerebellar syndrome with pyramidal involvement
5. Acromegaly with hand symptoms
6. Emergency same as prev panel
7. Communication same as prev panel

Cycle 3 Panel 1
1. Long case Abdomen portal Htn due to schistosomiasis bleeder
2. Bilateral bronchiectasis due to lung fibrosis
3. MS with pulmonary Htn
4. Bilateral cerebellar syndrome with pyramidal signs
5. Scleroderma
6. Emergency pt with recurrent weakness wich occurs mainly when he eat
high sugar Familial Hypokalemic periodic paralysis
7. Communication BBN pt has astrocytoma inoperable team decided for
palliative treatment talk to his wife
Cycle 3 Panel 2
1. Long case abdomen Schistosomiasis portal Htn bleeder
2. Lung fibrosis
3. AVR
4. Pyramidal weakness + peripheral neuropathy
5. Arthritis mutilans
6. Emergency same as prev panel
7. Communication same as prev panel

Best to all
1-metrifonat
2-ivermectin

‫ﺑﺴﻢ ﷲ اﻟﺮﺣﻤﻦ اﻟﺮﺣﯿﻢ‬

2nd PART EXAM JUNE 2009 –INTERNAL


MEDICINE

COLLECTED BY:

DR WANOUR
BEST OF FIVE
1) A British man has been diagnosed as having schistomiasis. He
wish to take a drug which only acts on S. haematobium
a) praziquantel b) metrifonate c) oxamniquine
d) Artemisinin
e)…………..

answer1
From Hunters tropical medicine
(oxamniquine-for s. mansoni-----)
(metrifonate is an anticholensterase organophosphate com-
pound-for s.haematobium)

2) 30 years old women after she visit a rural area the patient pre-
sented with sever itching on her hand and fore arm on exami-
nation there was brows on affected areas the treatment of
choice is
a) ivermectin
b) topical malathion c) oral chlorpheniramine .
d)oral predinsolone
e) topical hydrocortisone
answer
a) ivermectin

scabies www.visualdx.com
Typical infestation — The essential lesion is a small, erythematous, nondescript papule, of-
ten excoriated and tipped with hemorrhagic crusts (picture 2A-B). It is not a dramatic lesion
and not always easy to see. More striking, when present, is the burrow. Pathognomonic
when correctly identi ied, the burrow is a thin, grayish, reddish, or brownish line that is 2 to
15 mm long (picture 3A-B). Burrows are often absent, however, or obscured by excoriation
or secondary infection. Miniature wheals, vesicles, pustules, and rarely bullae may also be
present.
3) An American man was diagnosed as having malaria he received
full dose of an anti malaria and became a symptomatic one
week later before he return back to America a bloods film
check showed gametocytes and shizont what is your advice to
him
a) reassure
b) seven day course of injectable artemether
c) oral artesunate
d)
e)

primiquine

4) A diagnosis of tubercloid leprosy was established and the pa-


tient was commenced on dapsone & clofazimine then the pa-
tient developed fever, arthralgia ,skin rash and nodules how to
deal with him
a) stop dapsone, & clofazimine,
b) give steroids and continue his medications
c) change to rifampicin
d)thalidomide
e)………………………..

For reviw????? Kumar


b) give steroids and continue his medications

Treatment of lepra reactions. This is urgent, as irreversible

eye and nerve damage can occur. Antileprosy therapy must

be continued. Type II lepra reactions (ENL) can be treated

with analgesics, chloroquine, clofazimine and antipyretics.

Thalidomide was prevously used for ENL and is effective.

However, because of its teratogenicity, it is no longer


recom-mended by WHO. Prednisolone 40–60 mg daily for
3–6

months is effective in type I reactions.

(harison)Therapy for Reactions TYPE 1 Type 1 lepra reactions are best treated

with glucocorticoids (e.g., prednisone, initially at doses of 40 to 60

mg/d). As the inflammation subsides, the glucocorticoid dose can be

tapered, but steroid therapy must be continued for at least 3 months

lest recurrence supervene. Because of the myriad toxicities of pro-longed glucocorticoid therapy

type 2 1 to 2 weeks) of glucocorticoids


(initially 40 to 60 mg/d) are often effective. With or without therapy,

individual in lamed papules last for 1 week. Successful therapy is

defined by the cessation of skin lesion development and the disap-pearance of other systemic signs
and symptoms. If, despite two courses

of glucocorticoid therapy

5) The method which can confirm past infection with


schistosomiasis
a) Katozo test
b) antibodies
c) antigens
d) abdominal u/s
e) rectal snip

e) rectal snip

6) Traveler to southern state presented with painful lower limb


tender thigh muscles swellings the most causative agent is:
a) Loa loa
b) onchocerciasis (river blindness)
c) w bancofti (filariasis)
d) Dracunculiasis (Guinea worm)
e) cystocercosis

Loa loa

Short-term residents of endemic areas often have more severe

Manifestations of the disease.


7) Patient with hepatitis C infection his serum HCV antibody was
positive
a) the patient is immune
b) acute infection
c) chronic infection
d) ….
e) ….

b) chronic infection

8) HIV patient developed sever itchy skin lesion on his the scalp,
face, and upper trunk what is this skin lesion
a) seborrheic
b) dermatitis
c) c) …
d) . d) ….
e) e) ….

Eosinophilic folliculitis (kumar) p=188


presents with urticarial lesions particularly on the face, arms

and legs.

9) Patient with HCV infection with mild liver impairment


a) pegylated interferon & lamovudine
b)c)d)e)

10) 60 years old man known with COPD presented to refer clin-
ic for regular follow up his ABG showed: a)b)c)d)e)
a) Ph pco2 po2 HCo2 b) Ph pco2 po2 HCo2 c) Ph pco2
po2 HCo2 d) Ph pco2 po2 HCo2 e) Ph pco2 po2 HCo2
11) 45-years-old man alcohol consumer and long standing
dyspepsia and heart burn presented with 2 day history of
worsening sever epigastric abdominal pain which is radiating
to the back there was vomiting and diarrhoea investigations Hb
11.5 wcc 24500 s.lipase(amylase) 2000 Ca 2 alb 33
CXR: free air under the right hemidiaphragm
a)pancreatitis due to anterior duodenal perforation
b) pancreatitis due to posterior duodenal perforation
c)
d)
e)

12) Patient with TIA


a) CT brain is investigation of choice
b) Scope of surgery increase if vertebra-basilar vessels were af-
fected
c) 50% stenotic carotid artery is an indication for endartrectomy
d)
e)

The initial evaluation of suspected TIA and minor nondisabling ischemic stroke includes
brain imaging, neurovascular imaging, and a cardiac evaluation. Laboratory testing is helpful
in ruling out metabolic and hematologic causes of neurologic symptoms.
13) 29 years old woman with features of iron de iciency anmia
of peripheral blood picture but not responding to full course
iron, diagnosis is:
a) sidroblastic anaemia
b) thalassemia major
c)
d)
e)

kumar
The commonest causes of failure of response
to oral
ironare:
■lack of compliance
■continuing haemorrhage
■incorrect diagnosis, e.g. thalassaemia trait.

14) During survey in Darfur for leishmaniasis a patient was


found with DAT test and leishmanin test (+),most probable this
indicate:
a) Active disease
b) Recovered or subclinical
c) Cured
d) Post clinical infection
e) Post cutaneous leishmaniasis
From Sudanese national protocol guide lines:

DAT L.S.T clinical condition


test
-ve -ve Suspected case during survey
+ve -ve Infection or subclinical
+ve +ve Recovered or subclinical
-ve +ve Cure V.L, post clinical infection, post
cutaneouse Liesh

15) A 55 year old man is referred to the hospital for further


assessment having noticed a growth of papules from the nail
folds. A doctor confirms that they are facial angiomas (adenoma
sebaceum).
There is also a history of seizures and renal impair-
ment. What is the most likely other clinical finding?
a) subungual fibromas
b)
c)herdetery heamorrhgic telengectesia
d)
e)
Answer: tuberous sclerosis. The diagnosis is tuberous sclero-
sis. This is an inherited (autosomal dominant)
hamartomatous condition in which there are facial angiomas
(adenoma sebaceum), subungual fibromas, angiomyolipomas,
cardiac rhabdomyomas, pulmonary lymphatic involvement,
skin changes such as shagreen patches and ashleaf macules.
16) An 18-year-old female college student is evaluated in the emergency
department because of a seizure that occurred that morning. According to
her roommate, the patient woke up at her usual time and had several epi-
sodes of brief bilateral arm jerks before she fell and lost consciousness. Her
whole body stiffened for 5 to 10 seconds, and then she had rhythmic clonic
jerking of her arms, legs, and trunk for about 2 to 3 minutes. She remained
unresponsive for 10 minutes and then gradually regained complete con-
sciousness over the next hour. The patient remembers the arm jerks but has
no recollection of the seizure. She states that she has been having arm jerks
in the morning for several months, which have become worse recently. She
has been staying up late every night for the last week studying for final ex-
aminations. Neurologic examination is normal.
17) Which of the following is the most likely diagnosis?

a-Partial seizure with secondary generalization

b-Absence epilepsy

C-Syncope

D-Juvenile myoclonic epilepsy

E-Pseudo seizure

Answer and Critique (Correct Answer = D)


Key Point
Juvenile myoclonic epilepsy is a primary, genetic, generalized epilepsy that typically
manifests with myoclonic jerks followed by a generalized tonic–clonic seizure.
The patient had myoclonic jerks that led to a generalized tonic–clonic seizure. This is a
typical presentation of juvenile myoclonic epilepsy, an idiopathic epilepsy syndrome.
Affected patients have a normal neurologic examination and normal development. Un-
like other idiopathic epilepsies, remission is rare even after years without seizure, so
lifelong treatment is necessary. Patients are sensitive to sleep deprivation, alcohol, and
stress. It is often necessary to specifically ask the patient whether there have been any
other episodes of jerks involving the head, extremities, or trunk. Patients may have been
experiencing these for some time but not have associated the myoclonus with seizures.
Unlike juvenile myoclonic epilepsy, which is a primary, genetic, generalized epilepsy,
partial seizures originate from a focal part of the brain and their clinical presentation
depends on their neuroanatomic location. A partial epilepsy may present with a simple
partial seizure (consciousness not impaired) or complex partial seizure (consciousness
impaired) before progressing to a generalized tonic–clonic seizure, although some pa-
tients with partial epilepsies present with generalized tonic–clonic seizures only. The
patient with partial epilepsy may show a focal abnormality on electroencephalography
or on MRI. Idiopathic generalized epilepsies present with only generalized onset sei-
zures, which may be convulsive or nonconvulsive. Absence seizures are nonconvulsive
generalized onset seizures that mainly affect children. Absence seizures manifest with
sudden onset of staring, with or without eye blinking or lip smacking.
The other options are unlikely. Patients with syncope often have several myoclonic
jerks while unconscious, but they recover quickly once blood pressure is re-established.
Nonepileptic events (pseudoseizures) vary in presentation but are often associated with
moaning, crying, and arrhythmic shaking of the body

(A20-year-old university student was admitted to the Cas-


ualty Department early morning while she was preparing
to go to an UN exam, having presented with loss of con-
sciousness and an episodes of fits whilst in her room. Her
friend who accompanied her had witnessed the attack
and described her as abnormal movement of her extrem-
ities then suddenly dropping to the floor and shaking her
arms and legs. On admission 30 minutes later, she was
still shaking her limbs (not mentioned incontinent of urine or injured herself
or bitten her tongue). She did not take any regular medication.
abnormal movement of her extremities in the early
morning had been for several months. When asked she
mentioned that she was aware of her abnormal move-
ments of her limbs but unaware the episodes of fits

On examination, her blood pressure was 140/80 mmHg,


pulse was 92 beats per minute and regular, temperature
was 36.8°C and. Her eyes were tightly shut and impossi-
ble to open passively and she was shaking both arms and
legs.
Given the above history and clinical findings, what is the
most
a) Pseudo seizure
b) Tonic-clonic seizure
c) Complex partial seizure
d) Vasovagal syncope
e)

a) Pseudo seizure

23)
A 55-year-old gentleman presented to the outpatient clinic
after being referred by his General Practitioner. He com-
plained of feeling lethargic and had lost 15 kg in weight. He
also complained of profuse sweating, especially at night and
also had some upper abdominal discomfort. On examination,
the spleen was palpable 12cm below the left costal margin.

Investigations revealed:

Haemoglobin 9.7 g/dL

White cell count 17.4 x 109/L

Neutrophils 14.0 x 109/L

Lymphocytes 1.5 x 109/L

Monocytes 0.8 x 109/L

Basophils 0.7 x 109/L


Eosinohils 0.4 x 109/L

Platelets 550 x 109/L

The blood film was reported as follows: The neutrophils are left
shifted with numerous myelocytes present. There is an occa-
sional promyelocyte but no blasts. There are also a number of
nucleated red blood cells. There is a thrombocytosis with plate-
let anisocytosis

The best drug to convert haematological indices is:

a) Glivec (Imatinib)
b) Alpha Interferon
c)
d)
e)

Comments:
The blood film shows chronic myeloid leukaemia. Unlike CLL,
CML will usually progress to frank leukaemia quite rapidly, so
treatment is needed. You can not undertake a period of observa-
tion, unless age and other co-morbidity made treatment inap-
propriate. NICE have now approved the tyrosine kinase inhibi-
tor- Glivec- as first line treatment for chronic and accelerated
phase CML. Interferon is no longer used as first line, unless in
the context of a clinical trial. If the patient had been in blast cri-
sis phase, then AML type chemotherapy as well as Glivec would
be the choice. If remission is not achieved with Glivec, then in a
patient under 60-65 years an allogeneic transplant would be
considered if there was a matched sibling donor, in a 50year old
patient or younger a matched unrelated donor transplant would
be considered too
24) Patient with inferior MI developed 6-10 ventricular ectop-
ic how to deal
a) Amiodarone
b) beta blocker
c)
d)
e)

MAKSAP
If symptoms can be clearly correlated with PVCs, treatment may be appropriate, alt-
hough many patients respond well to reassurance. First-line therapy is almost always
a β-blocker or calcium channel blocker. Class IC and III agents also can be useful. Ra-
diofrequency ablation may be appropriate for patients with severe symptoms that are
refractory to drug therapy

25) A 71-year-old woman with type 2 diabetes mellitus is hospitalized with new congestive
heart failure. She had an inferior wall myocardial infarction treated with percutaneous
coronary intervention 3 years ago. A recent echocardiogram showed a left ventricular
ejection fraction of 35%. A stress myocardial perfusion scan demonstrates a large, fixed
inferior defect and a small, reversible anterolateral defect. Her cardiovascular disease is
treated medically with furosemide, lisinopril, a statin, aspirin, and metoprolol. Her dia-
betes regimen is glyburide, 10 mg twice daily, and metformin/rosiglitazone, 1000 mg/4
mg twice daily. Capillary blood glucose values have recently been in the 140 to 180
mg/dL (7.77 to 9.99 mmol/L) range, with an occasional result in excess of 200 mg/dL
(11.1 mmol/L). Her hemoglobin A1c has recently climbed to 7.6%. Combination thera-
py with metformin/rosiglitazone is stopped in the hospital.
26) Which of the following would be the most appropriate discharge antihyperglycemic
regimen for this patient?
A Continue glyburide alone
B Continue glyburide and add acarbose
C Stop glyburide and begin insulin therapy
D Continue glyburide and begin metformin; add insulin
E Continue glyburide and begin rosiglitazone; add insulin
Answer and Critique (Correct Answer = C)
Key Point
The insulin sensitizing drugs metformin and the thiazolidinediones are contraindicated
in patients with advanced heart failure.
The management of the patient with both diabetes and heart failure is particularly chal-
lenging. The insulin sensitizing drugs metformin and the thiazolidinediones are contra-
indicated in patients with advanced heart failure. Metformin should not be used in pa-
tients whose heart failure is advanced to a degree that requires drug therapy. This cau-
tion emerges from the observation that a significant proportion of the lactic acidosis
cases reported in post-marketing surveys involved patients with underlying heart fail-
ure. The mechanism likely pertains to hemodynamic impairment and resultant de-
creased tissue perfusion and increased lactate production. Potentially, decreased renal
blood flow in such patients may also decrease metformin elimination, heightening the
risk of lactic acidosis. The thiazolidinediones are not indicated in patients with New
York Heart Association class II-IV heart failure. This caution results from this class of
drug's known side effect of increasing fluid retention. Case reports of worsening heart
failure or new heart failure have emerged, although a recent observational study sug-
gested that mortality was actually reduced in heart failure patients treated with
thiazolidinediones (with a similar observation in patients treated with metformin).
However, until further data becomes available—preferably in the form of a randomized
clinical trial—it is prudent to continue to avoid these medications in such patients. This
patient's glucose levels are elevated on her current therapy, and they will certainly
worsen upon discontinuation of metformin and rosiglitazone. Continuing glyburide
alone is therefore inappropriate. Acarbose is a relatively weak antihyperglycemic agent
and will probably not adequately reduce glucose levels. Changing the patient to insulin
is the intervention most likely to improve her glycemic control and decrease the risk of
future complications.
29)
Patient underwent radio nuclear scanning 3 month post inferior
MI due to chest pain he is known diabetic and on oral
hypoglysaemic . scanning show an old infarction inferiorly and
new resolving anterior. Hb Ac1 8%, his control of DM would be
by:
a) Stop oral and give s.c insulin
b) Continue on same drug
c) Increase current dose
d) Add other drug
30) Obese FBS 210 , high lipid Bp 135/90
a) Life style modification
b) Metformin and ACE inhibitor
c) Lipid lower drug
d)
e)

31) Alcoholic developed tremens


a)give thiamine
b)
c)
d)
e)

BENZODIAZPINE
32) Schizophrenic on haloperidol developed neuroleptic ma-
lignant syndrome
a) Bromocriptine
b)
c)

33) patient with migraine preventing drug he is known asth-


matic
a)ergot
b)zolmitryptan
c)propanolol
d)

KUMAR
Prophylaxis. The following are used continuously when
attacks are frequent:
■pizotifen (5HT antagonist) 0.5 mg at night for several
days, increasing to 1.5 mg (common side-effects are
weight gain and drowsiness)
■propranolol 10 mg three times daily, increasing to 40–
80 mg three times daily contraindicated in asthma
■amitriptyline 10 mg (or more) at night.
Sodium valproate, methysergide, SSRIs, verapamil, topira-mate, nifedipine
and naproxen are also used. Gap junction
blockers are being used in trials.

34) tiredness, collapse with postural hypotension. past history


of cranial surgery. Na 129 cortizol low other pituitary hormone
levels were normal treatment with:
a)dessmopresin
b) hydrocortisone
c) thyroxin
d)
e)

b) hydrocortisone

35) patient with hyperthyroidism commenced on


neomercazole his WCC base line was 8000 neutrophils 5000 one
month later the patient came with WCC 5500 neutrophils 2200
what is next step:
a)reassure
b)stop the drug
c)change to other drug
d)give antibiotics
e)

36) on routine follow up of patient ,two month after a success-


ful kidney transplant, urinanalysis showed ++ alb .what is likely
cause:
a)ATN
b)lymphocele
c)recurrence of GN post streptococcus
d)cyclosporin toxicity
e)

‫ﺗﻔﺘﯿﺶ‬

Focal segmental glomerulsclerosis


Patients with mesangial proliferation on biopsy of the native kidney and those with recur-
rence in a prior allograft are also more likely to develop recurrence [45]. White recipients of
African American kidneys may be at particularly high risk of recurrence [40]. On the other
hand, blacks appear to represent a lower risk group in which recurrence is less frequent
[37,40]

37) considerining the diagnosis of multiple myeloma not


monoclonal gammopathy which is most helpful
a)plasma cells less than 7%
b)bone scanning
c)IgG serum spike more than 2 g
d)Bence Jones more than 2 g
e)
Diagnostic criteria for multiple myeloma and related disorders
Multiple myeloma (all 3 criteria must be met)
Presence of a serum or urinary monoclonal protein
Presence of clonal plasma cells in the bone marrow or a plasmacytoma
Presence of end organ damage felt related to the plasma cell dyscrasia, such as:
- Increased calcium concentration
- Lytic bone lesions
- Anemia, or
- Renal failure
Smoldering (asymptomatic) multiple myeloma (SMM, both criteria must be met)
Serum monoclonal protein ≥3 g/dL and/or ≥10 percent to <60 percent bone marrow clonal
plasma cells
No end organ damage related to plasma cell dyscrasia (see list above)
Monoclonal gammopathy of undetermined significance (MGUS, all 3 criteria must be
met)
Serum monoclonal protein <3 g/dL
Bone marrow plasma cells <10 percent
No end organ damage related to plasma cell dyscrasia or a related B cell
lymphoproliferative disorder (see list above)

monoclonal gammopathy is characterized by the following find-


ings: absence of symptoms; M component (whether IgA, IgG, or
IgM) <3 g/dL; fewer than 10 percent plasma cells in the bone
marrow, and absence of lytic lesions, anemia, hypercalcemia,
and renal insufficiencya)b)c)d)e)
38) A 22-year-old woman presented with hirsuitism and
oligomenorrhea for the last 5-years. She is very anxious about
her irregular menses and gaining weight recentely. Examina-
tion is essentially normal apart from coarse dark hair being
noticed under her chins and over her lower back.
Investigations during the follicular phase:

serum androstenedione 10.1 nmol/L (0.6–8.8)


serum dehydroepiandrosterone sulphate 11.6 µmol/L (3–12)
serum 17-hydroxyprogesterone 5.6 nmol/L (1–10)
serum oestradiol 220 pmol/L (200–400)
serum testosterone 3.6 nmol/L (0.5–3)
serum sex hormone binding protein 32 nmol/L (40–137)
plasma luteinising hormone 3.3 U/L (2.5–10)
plasma follicle-stimulating hormone 3.6 U/L (2.5–10)
What will aid more to reach a diagnosis:
a)uls abdomen
b)further hormonal analysis
c)CT brain
d)e)
PCOs is associated with a raised LH:FSH ratio, with insulin re-
sistance and hyperandrogenism as evidenced by raised
androstenedione and slightly raised testosterone. Elevated pro-
lactin concentrations although a feature of PCOs is not specific of
the diagnosis and may suggest microprolactinoma. Although in-
sulin resistance is a feature of PCOs, a raised insulin concentra-
tion is rather irrelevant and no one would measure this in clini-
cal practice. It is often elevated in association with testosterone
secreting tumours. Metformin has been shown to increase the
rate of conception in PCOs through improved insulin sensitivity
39) A girl with RA on cyclosporine and methotrexate came
complaing of pain on her shoulder which is provoced just on
touch no features of active disease and so normal ESR CRP your
action would be :
a)change drugs
b)amitriptiline
c)
d)
e)

‫ﻓﺘﺶ‬
40) Loss of pain and temprure .Lateral medullary syndrome
a)vertibrobasillar
b)c)d)e)

41) Mitral tissue valve replacement proposed for


tonselectomy,allergic to penicillin
a)oral clindamycin
b)iv clindamycin
c)no need
d)
e)
Dental, oral, or upper respiratory tract procedures — The primary antibiotic regimen for
most patients, including those with prosthetic valves, is amoxicillin, 2 g orally 30 to 60
minutes before the procedure; a second dose is not necessary [2,30].

A different regimen is warranted in the following circumstances; all drugs are given 30 to
60 minutes before the procedure [2]:

 Patients who are allergic to penicillins or ampicillin can be treated with cephalexin (2
g) or azithromycin or clarithromycin (500 mg) or clindamycin (600 mg).
 Patients who are unable to take oral medications can be treated with 2 g of intrave-
nous or intramuscular ampicillin. Patients allergic to penicillin can be given
cefazolin or ceftriaxone (1 g intravenously) OR 600 mg of intravenous or intramuscu
clindamycin.

Prosthetic valve with CVA his INR controlled between 3.5 -4.5
a)increase warferin
b) keep INR 4.5 – 5.5
c)basket carotid
d)change to heparin
e)
Thromboembolism despite antithrombotic therapy — The above recommendations for an-
tithrombotic therapy in patients with mechanical or bioprosthetic heart valves reduce but do
not eliminate thromboembolic events. Randomized trial data are not available on the opti-
mal approach for patients who develop thromboembolic events on standard therapy.

This issue was most completely addressed in the 2006 ACC/AHA guidelines, which rec-
ommended the following approach to increasing the intensity of therapy when it can be
safely performed [10]:

 Among patients with a goal INR of 2.0 to 3.0 — increase the goal INR to 2.5 to 3.5
 Among patients with a goal INR of 2.5 to 3.5 — the goal INR may need to be increased
to 3.5 to 4.5
 Among patients treated with warfarin plus aspirin (75 to 100 mg/day) — if the higher
dose of warfarin does not prevent further thromboembolic events, the aspirin dose
may need to be increased to 325 mg/day
 Among patients treated with warfarin but not aspirin — add aspirin (75 to 100
mg/day)
 Among patients treated with aspirin alone — the aspirin dose may need to be in-
creased to 325 mg/day, clopidogrel added (75 mg/day), and/or warfarin added

In comparison, both the ACCP Consensus Conference and the ESC guidelines made more
limited recommendations. The 2008 ACCP guidelines recommended that, among patients
with mechanical valves, aspirin (50 to 100 mg/day) should be added to warfarin therapy
and/or warfarin therapy should be upwardly titrated (to targets similar to those in the
ACC/AHA guidelines) [11]. The ESC guidelines recommended that aspirin be added to
warfarin therapy only after treatment of identified risk factors and a full evaluation and op-
timization of warfarin therapy have proven unsuccessful [12].

Valve thrombosis is discussed separately

42) Recent DVT in lady with PH of DVT & HITs skin lesions
after heparin .to give
a)warferin
b)heparin then warferin
c)hirudin then warferin
d)e)
reatment — For patients with an intermediate or high pretest probability of HIT, in whom a
solid phase immunoassay has been found to be positive, we recommend the immediate use
of an alternative nonheparin anticoagulant (eg, lepirudin, argatroban, danaparoid,
fondaparinux, bivalirudin) (Grade 1B). Any of these agents can be used in patients whose
renal and hepatic functions are both normal (algorithm 1 and table 3). (See
'Treatment' above and 'Initial intervention' above.)

 We suggest that patients with abnormal hepatic function and normal renal function be
treated with lepirudin, danaparoid, or fondaparinux, while those with abnormal re-
nal function and normal hepatic function receive argatroban at standard doses or
lepirudin at reduced doses (Grade 2C).
 For patients in whom both renal and hepatic function are abnormal we suggest treat-
ment with argatroban or bivalirudin at reduced doses (Grade 2C).
 We suggest that patients with HIT be anticoagulated for at least two to three months
in the absence of a thrombotic event and three to six months if such an event has oc-
curred (Grade 2C). Warfarin can be started once the patient has been stabilized with a
nonheparin anticoagulant and the platelet count has recovered to ≥150,000/microL.

To accomplish this, low initial doses of warfarin, rather than high "loading" doses, should
be started. The nonheparin anticoagulant should be continued for at least five days along
with warfarin, until the platelet count has stabilized and the INR has reached the intended
target range

A 64-year-old woman is evaluated in the emergency department for a 4-day history of pro-
gressive leg weakness and numbness and a 1-day history of urinary incontinence. She has
also had increasingly severe midback pain for the past 2 months. She has a history of breast
cancer diagnosed 2 years ago, treated with surgery and local radiation therapy. Her only
current medication is tamoxifen.

Physical examination shows normal mental status and cranial nerves. Strength in the arms
is normal. Legs are diffusely weak, 3/5 proximally and 4/5 distally. Sensory examination
shows diminished pin sensation from the nipples downward; vibratory sense is severely
diminished in the feet. Reflexes are 2+ in the biceps and triceps and 3+ in the knees and
ankles. An extensor plantar response is present bilaterally. Anal sphincter tone is dimin-
ished.

Which of the following is the most appropriate diagnostic study at this time?

A CT of the lumbar spine


B Electromyography and nerve conduction studies
C MRI of the brain
D MRI of the entire spine
E Plain radiographs of the entire spine
This question has been answered. To clear answers, open the Answer Sheet.
Answer and Critique (Correct Answer = D)
Key Points

 A spinal cord disorder should be considered in any patient with bilateral motor and
sensory dysfunction in the extremities in the absence of signs or symptoms of brain
or brainstem dysfunction.
 Spinal cord compression due to epidural metastasis is a neurologic emergency for
which urgent MRI of the entire spine is appropriate.
This patient has bilateral weakness and upper motor neuron signs in the legs, sensory loss
below the T4 level, and sphincter dysfunction; there are no signs or symptoms of brain or
brainstem dysfunction. These findings are consistent with a spinal cord process. Given her
history of breast cancer, metastatic spinal cord compression from an epidural metastasis is
most likely and represents a neurologic emergency that must be excluded by urgent imag-
ing. Although the distribution of her pain and her sensory level suggest that her lesion is at
the thoracic level, MRI of her entire spine is most appropriate because sensory levels can be
unreliable for localizing the site of an epidural tumor. In addition, patients with metastatic
epidural tumor can have multiple sites of disease in the spine. MRI gives excellent images
of both the spinal cord and the vertebrae, whereas CT does not adequately show the sub-
stance of the spinal cord and epidural region, making MRI the modality of choice in this
case.

CT scan of the lumbar spine is not an appropriate imaging choice in this patient. Because
the spinal cord ends at around the L1 vertebral body, the spinal cord would not be visual-
ized if imaging of the lumbar spine alone is performed. Plain radiographs of the spine can
visualize some bony metastases and fractures but are not sensitive for the site of cord com-
pression and do not image the spinal cord. Electromyography and nerve conduction studies
are helpful in diagnosing peripheral nerve and muscle diseases but have no role in the eval-
uation of spinal cord disorders. MRI of the brain may be needed to assess for asymptomatic
brain metastases but is not as urgent as spine imaging.

45 Patient with lower limb weakness and sever pain in


mid back and P.H of breast ca .sensory level up to nipples
how to study:
a)MRI cervical spine
b)MRI whole spine
c)bone scanning
d)e
43) Patient with PH of MI presented with SOB he is on aspirin
& atorva .give
a)ACE inhibitor
b)Spironolactone
c)
d)
e)
44) Asthmatic with productive cough .Eosinohils count 10%
a)do provocation test
b)sputum for AAFB
c)precipitant
d) CT chest
e)

c)precipitant

45) A child with CVA .CBC haemolytic anaemia x –linked of his


brother will be affected if both parents were affected
46) b)his sons will not be affected

c)d)e)
47) A man presented with weakness of his lower limbs OlE
weakness of both thigh no sensory deficits.CSF protein is
high.how to treat:
a)immunoglobulin iv
b)steroids
c)pulse methylpredinsolone
d)cyclosporine
e)

a)immunoglobulin iv

48) BT normal APTT 55


a)von willebrand
b)factor v
c)factor V111
c)factor V111

49) Patient with typhoid fever for 18 days received antibiotics


but not treated well presented very ill the most most probable
complication has occurred is:
a)DIC
b)myocarditis
c)renal failure
d)inestinal perforation
e)
50) Patient from algazeera with spleenectomy the most im-
portant vaccine he should receive is:
a)antimalari
b)H .infeunzae
c) pneumococcus
d)
e)

Experts recommend that people without a functional spleen have certain vaccinations to
reduce the risk of sepsis. Although these vaccines do not prevent all infections with pneu-
mococcus, HIB, and meningococcus, they can protect most people and reduce the severity
of infection in others.

 Pneumococcal vaccine — The pneumococcal vaccine is recommended for all people


who do not have a functional spleen. Two types of pneumococcal vaccines are availa-
ble, PPSV23 and PCV13. Many young children are given PCV13 as part of their routine
infant immunizations. PPSV23 is recommended for adults and children ≥2 years. (See

When possible, the vaccine should be given at least two weeks before planned surgical
removal of the spleen. The vaccine should be given 14 or more days after emergency
removal of the spleen (eg, due to trauma).
 Haemophilus influenzae B (HIB) vaccine — Most adults have been exposed to HIB and
are therefore immune. However, because there is a small risk that the person is not
immune, experts advise people without a functional spleen to have the HIB vaccine if it
was not given previously (it became available in the United States in 1988). If a person
is unsure if the vaccine was given during childhood, a blood test can be done to deter-
mine if he/she is immune.
 Meningococcal vaccine — The meningococcal conjugate vaccine (MCV4, Menactra) is
recommended for people without a functional spleen who are between 2 and 55 years
of age. Another meningococcal conjugate vaccine (MenACWY, Menveo) can be used in-
stead of MCV4 in people from 11 to 55 years of age. People who are greater than 55
years of age should be given the meningococcal polysaccharide vaccine (MPSV4,
Menomune).

As with the pneumococcal vaccine, the HIB and meningococcal vaccines should be given
at least 14 days before a scheduled splenectomy or at least 14 days after emergency remov-
al of the spleen.

 Influenza vaccine — Influenza (the flu), a highly contagious viral infection, is a com-
mon cause of pneumonia and other bacterial infections. Therefore, a once yearly influ-
enza vaccination (usually given in the fall) is recommended for people without a func-
tional spleen

51) Swine treatment


a)acyclovir
b) Amantadine
c)d)e)
52) Prenancy + brucellosis treatement of choice
a)rifampicin
b)c)d)e)

Brucellosis does not appear to cause abortions in humans as it does in animals, but bacte-
remia associated with the disease may cause premature labor and fetal wastage [59].

Two regimens have been suggested for the therapy of brucellosis in pregnancy, although
published data to support efficacy are sparse [60]:

 Rifampin — 900 mg once daily for six weeks


 Rifampin — 900 mg once daily plus trimethoprim-sulfamethoxazole (TMP-SMX; 5
mg/kg of the trimethoprim component twice daily) for four weeks

When therapy with the sulfa-containing regimen is given during the last week prior to de-
livery, attention needs to be given to the possibility of kernicterus in the infant. If these reg-
imens are not tolerated, use of doxycycline should be considered in conjunction with dis-
cussion regarding risks and benefits. (See "Tetracyclines", section on 'Pregnant or breast-
feeding women'.) Aminoglycosides alone (eg, streptomycin) have poor efficacy in brucello-
sis and may cause damage to the fetal cranial nerve VIII

53) SS +dyspepsia
a)b)c)d)e)

54) Lymphoma after radiotherapy pt drowsy& maliase +renal


impaiment s.cr 2.3 give
a)b)c)d)e)

55)
a)cefataxime + gentamycine
b)cefuroxamine
c)clinamycine + gentamycine
d)e)
56) lens ectopia +difficulty vission
a)marfans synd
b)homocystenuria
c)d)e)

57) pt with crush chest pain decresendo murmur Bp 160l90


(oartic dissection?????????)
a)MRI chest with enhancement
b)echo
bc)d)e) European Society of Cardiology guidelines: diagnostic imaging in acute aor-
tic dissection

Recommendation Class
1. Transthoracic echocardiography followed by transesophageal echocardiography I
2. Computed tomography II
if detection of tears is crucial IIb
3. Contrast angiography
to define anatomy in visceral malperfusion and to guide percutaneous interventions II
in stable patients IIa
routine preoperative coronary angiography III
in hemodynamically unstable patients IIb
4. Magnetic resonance imaging IIa
in hemodynamically unstable patients III
5. Intravascular ultrasound IIa
to guide percutaneous interventions IIb
Classification
Class I: Conditions for which there is evidence and/or general agreement that a given pro-
cedure or treatment is useful and effective.
Class II: Conditions for which there is conflicting evidence and/or a divergence of opinion
about the usefulness/efficacy of a procedure or treatment.
Class IIa: Weight of evidence/opinion is in favor of usefulness/efficacy.
Class IIb: Usefulness/efficacy less well established by evidence/opinion.
Class III: Conditions for which there is evidence and/or general agreement that the proce-
dure/treatment is not useful and in some cases may be harmful

58) pt with multi nodular goiter for 6 month his TFT remain
the same also he is asymptomatic T3,T4 were normal ,TSH low
0.1 he needs
a)start radio iodine
b)radioiodine scan
c)uls
d)neomercazole

SUMMARY AND RECOMMENDATIONS

 Subclinical hyperthyroidism is defined biochemically by a low serum TSH concentra-


tion but normal serum free T4 and T3 concentrations. Patients with subclinical hyper-
thyroidism typically have few or no symptoms of hyperthyroidism. (See
'Diagnosis' above.)
 The most common causes of subclinical hyperthyroidism are treatment with T4 (ex-
ogenous) and autonomously functioning thyroid adenomas and multinodular goiters
(endogenous) (table 1). (See 'Causes' above.)
 Subclinical hyperthyroidism is associated with an increased risk of atrial fibrillation
and, primarily in postmenopausal women, a decrease in bone mineral density. (See
'Clinical findings' above.)
 Patients receiving thyroid replacement therapy for the treatment of hypothyroidism
and who have TSH concentrations below normal should have their dose adjusted to
maintain a normal serum TSH concentration (approximately 0.5 to 5.0 mU/L). (See
'Patients on T4 for the treatment of hypothyroidism' above.)
 For patients with thyroid cancer and in some patients with benign nodular thyroid
disease, subclinical hyperthyroidism is the goal of thyroid hormone therapy. In these
patients, the benefits of TSH suppression are thought to outweigh the risks of subclini-
cal hyperthyroidism. (See 'Patients on suppressive levothyroxine therapy' above.)
 For patients with endogenous subclinical hyperthyroidism at high risk for cardiac or
skeletal complications (ie, the elderly) and who have a TSH concentration less than 0.1
mU/L, we recommend treatment of the underlying cause of subclinical hyperthyroid-
ism (Grade 1C).

For similar patients who have TSH values between 0.1 and 0.5 mU/mL, we suggest
treatment if the bone density is low and/or if the radionuclide scan shows one or more
focal areas of increased uptake (Grade 2C). If bone density is normal and the thyroid
scan fails to show a focal area of high uptake, we typically observe patients. In ob-
served patients, we measure TSH, free T4, and T3 every six months. (See 'Patients at
high risk for complications' above.)
 For patients with endogenous subclinical hyperthyroidism at low risk for cardiac or
skeletal complications (young individuals, premenopausal women) and TSH values
less than 0.1 mU/mL, we suggest treatment if the radionuclide scan shows one or
more focal areas of increased uptake (Grade 2C). For low risk patients who have a TSH
value between 0.1 and 0.5 mU/L, we suggest observation (Grade 2C). We measure
TSH, free T4, and T3 every six months. (See 'Patients at low risk for complica-
tions' above.)
 The treatment options for patients with subclinical hyperthyroidism are the same as
those for overt hyperthyroidism and depend upon the underlying etiology. (See
'Treatment options' above.)

Use of UpToDate is subject to the Subscription and License Agreement.

e)
59) the 1st indicator of megaloplastic anaemia response to B12
is:
a)low s.k
b)Hb raise
c)
d)
e)
Response to treatment — Laboratory studies should be monitored after cobalamin therapy
to document a hematologic and metabolic response:

 Elevated levels of serum iron, indirect bilirubin, and LDH fall rapidly within the first
one to two days following treatment with parenteral vitamin B12; bone marrow
erythropoiesis also changes from megaloblastic to normoblastic during this period. In
addition, the patient might note an improved feeling of well being, long before there
are any changes in the degree of anemia.
 Hypokalemia during the early response is due to the marked increase in potassium
utilization during production of new hematopoietic cells. It may be profound in those
who are severely anemic at the time of treatment, but its clinical relevance is uncer-
tain [54]. Such patients should be monitored during their initial response, and severe
hypokalemia treated with potassium supplementation. (See "Causes of hypokalemia",
section on 'Increased blood cell production'.)
 If the patient is anemic, there will be a reticulocytosis in three to four days, peaking at
one week, followed by a rise in hemoglobin and a fall in red blood cell mean corpuscu-
lar volume. The hemoglobin concentration begins to rise within 10 days and usually
returns to normal within eight weeks. A delayed response suggests the presence of an
additional abnormality or an incorrect diagnosis (eg, iron deficiency, infection, hypo-
thyroidism, malignancy).
 Hypersegmented neutrophils disappear at 10 to 14 days.
 Neurologic abnormalities, if present, improve over the ensuing 3 months, with maxi-
mum improvement attained at 6 to 12 months. The degree of improvement is inverse-
ly related to the extent and duration of disease

60) 30 years with 24 urine alb 4g treatment is:


a)high dose predinsolone
b)c)d)e)

61) Asbestose plague appeared on lat CXR on routine asymp-


tomatic pt inssurance chech next step is
a)compare with old
b)no action & follow up
c)do AP CXR
d)do biopsy
e) do CT chest

On CT, pleural plaques appear as focal pleural thickening (picture 2A-C). They can have a
table mountain, mesa, or nodular appearance and may impinge slightly on the adjacent lung
parenchyma. This impingement may cause a pulmonary subpleural curvilinear line adjacent
to the plaque, although this is rare.

CT clearly distinguishes pleural plaques from extrapleural fat and endothoracic fascia [5].
This is valuable because extrapleural fat may mimic a pleural plaque on a chest radiograph,
but a subsequent CT will reveal the true nature of the abnormality

62) Increase x descend duration JVP no elevated


,hepatosplenomegally +tender liver ,excess smoker and alcohol
consumer,diagnosis
a)ca bronchus
b)TB
c)
d)
e)

tb

63) Peptic ulcer not responding to PPI chech


a)ures test
b)gastrin level
c)x-oxylose
d)C14
e)
3
C-Urea breath test (Fig. 6.18). This is a quick and
reliable test for H. pylori and can be used as a screening
test. The measurement of
13
CO2 in the breath after
ingestion of
13
C urea requires a mass spectrometer. The
test is very sensitive (97%) and specific (96%). This test
is suitable for testing for eradication of the organism,
but may be falsely negative if patients are taking PPIs at
the time. A rapid release tablet which produces a result
in 15 mins is becoming available.
■ Stool antigen test. A specific immunoassay using
monoclonal antibodies for the qualitative detection of H.
pylori antigen is widely available. The overall sensitivity
is 97.6% with a specificity of 96%. It is useful in
the
diagnosis of H. pylori infection and for monitoring
efficacy of eradication therapy. (Patients should be off
PPIs for 1 week but can continue with H2 blockers.)

urea test
64) Pt with high s.k
a)ca gluconate
b)iv dextrose + insulin
c)d)e)

Ca gluconate
65) Haemoptysis + cough ,uls multiple liver masses more than
6 each of 2-3 cm in diameter what to do :
a)give dialy octeroids
b)give long acting octeroids
c)liver transplantation
d)chemotherapy
e)

66) Headache + HTN + angina BP170l140


a)treat as out patient
b)start with nitroglyseride spray
c)iv lasix
d)hydralazing
e)nitroprosside

67) 49 ys lady seen by several doctors complaing of


abdomennal distension & bloating usually conistipating
a)try glutein free diet
b)metronidazole
c)lactulose free diet
d)e)
SUMMARY AND RECOMMENDATIONS — Treatment of IBS varies with the severity
and type (diarrhea versus constipation predominant) of symptoms that are present.

Mild symptoms — Patients with mild or infrequent symptoms usually have little or no
functional impairment or psychologic disturbance. Thus, we suggest treatment should focus
upon the general measures described above (such as establishment of the physician-patient
relationship, patient education, reassurance, dietary modification, and, if bloating is not a
major factor, fiber supplementation) rather than specific pharmacologic therapy (Grade
2C). (See 'General principles' above.)

Moderate symptoms — Patients with moderate symptoms of IBS experience disruptions of


normal daily activities due to exacerbations of symptoms; these patients also may demon-
strate psychologic impairment.

We monitor patients' symptoms for several weeks to help identify precipitating factors,
such as lactose intolerance, excess caffeine, or specific stressors. Modifications in diet, be-
havioral changes, and psychotherapy may improve the clinical outcome.

Randomized controlled trials evaluating specific pharmacologic agents have demonstrated


their superiority compared with placebo. However, there have been few controlled trials
evaluating specific strategies for how these drugs should be used in conjunction with other
types of treatment (such as fiber therapy), how long they should be used, or whether they
should be given continuously or episodically. We often use pharmacologic intervention to
control symptom flares but also use continuous pharmacologic therapy (such as tricyclic
antidepressant drugs) for periods of months or years. Our choice of specific therapies is
based mainly upon symptoms and response to empiric trials (see 'Medications' above).

Given the modest benefit and relatively short-term follow-up demonstrated in the trials of
rifaximin, we suggest NOT using antibiotics routinely in patients with IBS (Grade 2B).
However, in patients with moderate to severe IBS without constipation (particularly those
with bloating) who have failed to respond to all other therapies, including a low carbohy-
drate diet and elimination of fermentable oligo-, di-, and monosaccharides and polyols
(FODMAPs), it is reasonable to consider two-week trial of rifaximin. (See
'Antibiotics' above and 'Carbohydrate malabsorption' above.)

Intractable symptoms — A small subset of patients with IBS present to tertiary care centers
with severe, unrelenting symptoms that are often associated with underlying psychiatric
impairment and frequent health care utilization. We suggest behavioral modification and
the use of psychoactive drugs in such patients (Grade 2C). (See 'Medications' above.)

68) To differentiate between ATN & prerenal which favour the


later
a)u.Na more than 600
b)c)d)e)
The major distinguishing findings between prerenal disease and ATN are therefore based
upon the results of the following measurements/assays, which are ranked in their order of
general usefulness [18-21]:

 Urinalysis
 Response to fluid repletion
 Fractional excretion of sodium (FENa) and urea (FEUrea)
 BUN/plasma creatinine ratio
 Rate of rise of plasma creatinine concentration
 Urine sodium concentration
 Urine osmolality
 Urine volume
 Urine-to-plasma creatinine concentration

We most commonly rely on the first three, which are used in combination with the clinical
setting to help diagnose the underlying disorder.

Urinalysis — The urinalysis with sediment

69) Pt with lymphadenopathy and cough +


hepatosplenomegally
a)lymph node biopsy
b)c)d)e)

70) Which investigation can exclude pul-embolism


a)normal VlQ
b)negative D dimer
c)CT chest
d)e)
71) (????Ca –lung,TB )cough SOB +pleural effusion simple way
to stablish a diagnosis
a)thorcocentesis
b)CT guide biopsy
c)sputum for AAFB
d)bronchoscopy
‫)ﻣﺮاﺟﻌﺔة‬
72) Lethargy ,pale Hb 5.4 rects 15% possible diagnosis is
a)PNH
b)c)d)e)

heamolytic anaemia
73) Bilateral ptosis wrinkled frontalis to over come ptosis di-
agnosis is
a)m dystrophica
b)c)d)e)

Bilateral ptosis
74) 44-year-old man was admitted to coronary care unit with
an anterior MI .this is his first MI he was treated with thrombo-
lysis and his chest pain settled he was non –insulin dependent
diabetic. He had no retinopathy ,neuropathy or nephropathy ,he
was a non smoker ,current treatment was gliclazide 80 mg bld
for the 1st 2 days he was put onto an iv infusion of insulin his
blood glucose was stick controlled ,he was know apyrexial ,pulse
70 ,Bp 140l80 blood glucose 10.9 Hb A1c 8.5 RFT normal .the
most way to manage his diabetes currently would be to:
a) Add metformin b) convert to metformin c)
convert to s.c insulin d) increase his gliclazide dose e)
leave him on the current dose of gliclazide
75) 30 years old man presented with lethargy and malaise 2
days after starting radiotherapy (chemotherapy) for non-
Hodgkin's lymphoma , he had abdominal discomfort on exami-
nation his temperature was 37c , pulse 95 blpm Bp 140l78 his
JVP was not elevated ,heart sounds and chest was normal , the
abdomen was soft not tender with hepatosplenomegally .
Investigations:
Hb 13.8 gld WCC 10.2 platelet 345 K 6.1 Na 130
B.urea 38.5 s.creatinine 450 Ca 2 s.uric acid ›10 urinaly-
sis: red cell 2+ + granular cast LFT normal
The prophylactic medication this patient should have received
is:
a) Allopurinol b)furosemide c)gentamycin d)e) (1.60)
76) 2000 patients with stroke were randomly allocated to
treatment with either aspirin or placebo at the end of 1 year 6
patients in the aspirin group had died, compared with 14 of the
placebo group. The number needed to treat ( NNT) to prevent
one death is:
a) 6 b) 80 c) 125 d) 250 e) 100

NNT is the number of patients who would need to be treated


with aspirin to prevent one death at the end of one year it is re-
ciprocal of: I l ARR (absolute risk reduction)
Relative risk of death in aspirin group= 6 l 1000 = 0.006
Relative risk of death in placebo group=14 l 1000=o.o14
ARR=0.014-0.006=0.008
NNT=1l 0.oo8=125
77) A 43 years old HIV positive man was referred because of
chronic watery diarrhoea that had been going on for 5 month.
He had associated abdominal pain, decreased appetite and
weight loss of 6 kg in that time period. He had no other medical
problems and was on no medication. He appeared cachexic but
the rest of physical examination was unremarkable. Blood Hb
12.8 WCC 4.2 platelets 190 CD4 300 vi-
ral load 12000
Of the following the most likely cause of his symptoms is:
a) cryptosporidia b) cytomegalovirus colitis
c)microsporidia d) Non-Hodgkin's lymphoma e)
mycobacterium avium intracelllare

‫ﻣﺮاﺟﻌﺔ‬

(All the pathogens may cause diarrheal disease in HIV


positive patients but at this CD4 count only non-Hodgkin
lymphoma would be a possibility, all of the others are un-
likely to present until the patient has a CD4 count ‹100)
78) A 50-year-old man presented with lethargy and
weight loss.
Investigations showed:
Hb 9.4 g/dL
WBC 54.2 x 109/L
Platelets 79 x 109/L
This of the following is in favour of myelofibrosis rather
than CML:
a) normal leukocyte alkaline phosphatase (LAP) score
b) positive Philadelphia chromosome (Ph)
c) Bone marrow blasts
d) bone marrow fibrosis
e) Com-
ments:

The candidate should recognize anemia, high WBC and


thrombocytosis with a leucoerythroblastic blood film.
The patient has ‘B’symptoms- weight loss and night
sweats with splenomegaly.

The differential is really between the myeloproliferative


disorders Chronic Myeloid Leukaemia, Myelofibrosis and
Essential Throbocythaemia. CML and myelofibrosis can
be very difficult to distinguish in the early stages on mor-
phology alone or on clinical findings and the most useful
and least invasive for the patient is molecular analysis
(PCR) of the peripheral blood- most cases of CML are
usually associated with BCR-ABL translocation ( t(9:22)),
and this can be detected by PCR, whereas this is not seen
in Myelofibrosis or ET. If present it can usually be detect-
ed in the peripheral blood, however in practice one
would still eventually proceed to a bone marrow exami-
nation to assess morphology and you would still also per-
form conventional cytogenetics on the bone marrow (this
is done on a bone marrow sample rather than peripheral
blood because the cellularity tends to be greater in the
BM, giving lower failure rates of the test).

LDH, NAP and USS are not going to help with making the
diagnosis, neither is immunophenotyping, as there are no
blasts in the blood and this is not an acute leukaemia
There are several other causes of bone marrow fibrosis
that should be considered.The diagnosis of myelofibrosis
is generally tenable if and only if the patient shows all of
the following features on initial presentation .Nucleated
RBCs in the peripheral blood Teardrop RBCs in the pe-
ripheral blood smear Early WBC forms in the peripheral
blood Palpable splenomegaly
diagnostic criteria for myelofibrosis with myeloid metaplasia

Major criteria
 Diffuse bone marrow fibrosis,
 Absence of Philadelphia chromosome or bcr-abl re-
arrangement in peripheral blood cells
 Splenomegaly
Minor criteria
 Anisopoikilocytosis with tear-drop erythrocytes
 Circulating immature myeloid cells
 Circulating erythroblasts (nucleated red blood cells)
 Clusters of megakaryoblasts and anomalous
megakaryocytes in bone marrow sections
 Presence of myeloid metaplasia
(The diagnosis of myelofibrosis with myeloid metaplasia is acceptable in
the presence of all three major criteria plus any two of the minor criteria OR
the presence of the first two major criteria and any four minor criteria.)
The LAP score aids in the differential diagnosis of chronic
myelocytic leukemia (CML) versus leukemoid reaction;
aids in the evaluation of polycythemia vera, myelofibrosis
with myeloid metaplasia, and paroxysmal nocturnal
hemoglobinuria
Low scores have been associated with:
CML, PNH, thrombocytopenic purpura, and hereditary
hypophosphatasia.
High scores have been seen in:
(1)polycythemia vera
(2)myelofibrosis
(3) aplastic anemia
(4)mongolism
(5)hairy cell leukemia
(6)leukemoid reactions
(7) neutrophilia either physiological or secondary to in-
fection
(8) It is also increased in Hodgkin disease.
79) Non STE angina after the course of CCU (heparin –
lisinopril-atenolol) what is next :
A)PCI
b)stress test
c)discharge
d)e)
80) Syncope + epilepsy ? HOCM echo showed septal hypertropy
best treatment :
a)ICD
b)c)d)e)

Risk factors for sudden death:


■massive left ventricular hypertrophy (> 30 mm
on
echocardiography)
■family history of sudden cardiac death (<
50 years old)
■non-sustainedventricular tachycardia on 24-hour
Holter
monitoring
■prior unexplained syncope
■abnormal blood pressure response on exercise
(flat or
hypotensive response).
The presence of these cardiac risk factors is
associatedwith
an increased risk of sudden death, and patients
with two or
more should be assessed for implantable
cardioverter–
defibrillator (ICD). In patients in whom the
risk is less, amio-darone is an appropriate
alternative.

81) RBCs + PROTEIN in urine Diagnosis


a)IgA nephropathy
b)c)d)e)
82) HTN in young his brother died with CVA most underline
cause is:
a)APCKD
b)c)d)e)
83) Pt jaundiced palbable liver high bilirubin normal enzymes
Cause is
a)niacin
b)hepatitis
c)d)e)

84) (??Obstructive sleep apnea) obesity day time somnolence


treatment
a)continue day time o2
b)c)d)e)
85) (lights criteria)
a)LDH
b)cholesterol
c)alb more than 0.5 of the serum
d)e)
86) Respiratory function test o2=52 Co2 60 Ph …
a)repeat the test
b)c)d)e)
87) Type 4 MGN treatment
a)cyclosporine = steroids
b)c)d)e)
88) An obese , failure of life style ,cholestrolol 215 next step
a)fibrates is the best
b)treat if LDL became more than 130
c)d)e)

89) Bahrelgazal pt with fever neck stiffness recived antibiotics


fever subsided but sill stiff neck WCC high –neutrophils the
cause is
a)T gondii
b)malaria??????
c)viral????
d)bacterial????
e)
90) Correlation
a)B12- dementia
b)c)d)e)
91) Serum B12 low IF given iv and showed response diagnosis
is
a)atrophic gastritis
b)bacteril over growth
c)celiac disease
d)e)
92) Colon ca hemicoloctomy done =liver problem
a)chemotherapy to ca colon
b)c)d)e)
93) Tuberculus peritonitis best yield diagnosis by
a)PCR ascetic fluid
b)ZN stain of ascetic fluid
c)PCR tissue biopsy
d)e)

94) 10 days after an admission to hospital still there was cough


and ++ alb in the urine with blood film eosinophilia he needs the
following investigation to stablish adiagnosis
a)CXR
b)precipitants
c)cold agglutination
d)e)

95) Resting tremor with fainting attacks


a)multi-system atrophy
b)c)d)e)
96) A lady with vomiting and diarrhoea treated and developed
down flexed head
a)torticolis
b)tradive dyskinasia
c)d)e)
97) Sore throat ,skin rash ,fever lymphadenopathy
,splenomegally .diagnosis
a)adult still disease
b)c)d)e)
98) Hepatoslenomegaly ascites ,pericarial effusion signs
(‫)ﻣﻜﺮر‬a)b)c)d)e)

99) Open case TB contact his mantoux test 13 mm asympto-


matic ,CXR clear,sputum negative ,action
a)treat
b)prophylaxis
c)repeat mantoux after 6 month
d)e)
100) 2 weeks after sre throat pt developed …
a)IgA nephropathy ‫ﻣﻜﺮر‬
b)c)d)e)
101) To diffrentiate SLE & RA .the later show
X-Ray wrist
a)subluxation at MCP
b)osteoporetic changes
c)d)e)
102) Nehrotic syndrome with bland urine seen in
a)amyloid
b)c)d)e)
103) SLE with diffuse proliphrative GN treat with
a)immunosupressive ‫ﻣﻜﺮر‬
b)c)d)e)
104) Hypertension on thiazide ACE inh ,amlodipine ,poorly con-
trolled , pt with BPH add
a)beta blocker
b)doxacin
c)increase ACE
d)e)
105) Alb ++, eosinophilia cause is
a)acute tubular necrosis
b)analgesic interstesial nephritis
c)d)e)
106) a)b)c)d)e)
107) a)b)c)d)e)
A 74-year-old woman with longstanding hypertension and rheumatoid arthritis
presented with dyspnoea. On examination she was in atrial fibrillation and was
normotensive.

The jugular venous pressure was elevated. She had bilateral pitting lower limb oe-
dema and ascites. Her echocardiogram showed normal left ventricular systolic
function and bi-atrial enlargement.

What is the most likely diagnosis?

Available marks are shown in brackets


1 ) Constrictive pericarditis

2 ) Hypertensive heart disease

3 ) Hypothyroidism

4 ) Lymphatic obstruction

5 ) Pulmonary fibrosis
option analysis

Comments:
1) is true

The combination of SOB, atrial fibrillation, lower limb oedema, ascites, raised JVP
and bi-atrial enlargement with normal systolic ventricular function is typical of con-
strictive pericarditis. Hypertension is another cause of diastolic dysfunction but this
lady is normotensive and hypertension would not create such dramatic clinical signs.

Further ECHO examination would reveal peak systolic and diastolic values desreasing
with inspiration, and impaired diastolic function.

Constrictive pericarditis is associated with Rheumatoid Arthritis. Constrictive pericar-


ditis can be a difficult clinical diagnosis. Read a case review of

A 64-year-old woman is evaluated in the emergency department 6 hours after the onset of
severe crushing chest pain associated with diaphoresis, nausea, and vomiting. Her medical
history is significant only for mild hyperlipidemia; her medications include atorvastatin and
aspirin.

Her blood pressure is 140/88 mm Hg, and her pulse rate is 88/min. The lungs are clear; she
has no murmurs; examination of the abdomen and extremities is normal. Electrocardiogram
shows a 3-mm ST-segment elevation in leads II, III, and aVF, with occasional premature
ventricular contractions. The hospital does not have cardiac catheterization facilities, and
the patient is therefore given fibrinolytic therapy and transferred to another hospital's inten-
sive care unit. In transit, the chest pain resolves. The patient has two episodes of 6- to 10-
beat ventricular tachycardia and stable hemodynamic parameters. Electrocardiogram now
shows a <0.5-mV ST-segment elevation.

In addition to heparin and aspirin, which of the following approaches is the most appropri-
ate next step?

A Coronary angiography
B Clopidogrel
C β-blocker
D Amiodarone
E Dobutamine stress echocardiography
This question has been answered. To clear answers, open the Answer Sheet.
Answer and Critique (Correct Answer = C)
Key Points

 In patients with ST-elevation myocardial infarction, successful fibrinolysis is sug-


gested by resolution of chest pain and ST-segment elevation and/or transient ven-
tricular arrhythmias early after reperfusion.
 In patients with ST-elevation myocardial infarction, reperfusion arrhythmias, typi-
cally manifested as a transient accelerated idioventricular arrhythmia, usually do not
require additional antiarrhythmic therapy.

This patient has features of successful reperfusion after acute inferior ST-elevation myocar-
dial infarction and may be treated medically until risk stratification is performed or recur-
rent ischemia or complications occur. Patients with depressed left ventricular systolic func-
tion by echocardiography are at high risk for ventricular tachyarrhythmias. Even the occur-
rence of asymptomatic nonsustained ventricular tachycardia within 48 hours of myocardial
infarction should not change usual management consisting of aspirin, β-blockers, angioten-
sin-converting-enzyme inhibitors, and statins.

Evidence of successful fibrinolysis involves resolution of both chest pain and ST-segment
elevation. The rapidity with which these resolve is directly related to early patency of the
affected artery. Reperfusion arrhythmias, typically manifested as a transient accelerated
idioventricular arrhythmia, usually do not require additional antiarrhythmic therapy.

Immediate coronary angiography is not indicated unless recurrent ischemia, persistent ST-
segment elevation, or hemodynamic instability including congestive heart failure occurs.
Clopidogrel may be added if indicated by stenting or significant additional evidence of ath-
erosclerotic vascular disease.
Dobutamine stress echocardiography to assess the heart for regions of myocardial viability
and inducible ischemia can be performed after the patient has been stabilized and treated
with initial medical management of myocardial infarction. If the patient can exercise, a lim-
ited exercise stress test may be the best option to assess cardiovascular risk. Predictors for
future adverse events in post–myocardial infarction patients include inability to exercise,
exercise-induced ST-segment depression, failure to achieve 5 metabolic equivalents during
treadmill testing, and failure to increase systolic blood pressure by 10 to 30 mm Hg during
exercis

A 26-year-old woman is admitted to the hospital for evaluation after having survived a car-
diac arrest. She had been resuscitated promptly using an automated external defibrillator.
The initial rhythm was ventricular fibrillation. She does not smoke or use illicit drugs, and
was feeling well before the event. Her medical history is unremarkable, and there is no fam-
ily history of cardiovascular disease.

The cardiac examination is pertinent for a grade 2/4 early systolic murmur along the left
sternal border. The echocardiogram demonstrates a septal wall thickness of 3.2 cm (normal
<1.1 cm).

What therapeutic intervention would improve her survival?

A Septal myomectomy
B β-blocker therapy
C Placement of an implantable cardioverter-defibrillator
D Avoidance of strenuous exercise
This question has been answered. To clear answers, open the Answer Sheet.
Answer and Critique (Correct Answer = C)
Key Point
Implantable cardioverter-defibrillator therapy reduces risk of sudden death in survivors of
cardiac arrest due to ventricular tachycardia or ventricular fibrillation without a reversible
cause.

This patient's echocardiographic findings indicate that she has hypertrophic cardiomyopa-
thy. Survivors of cardiac arrest due to ventricular tachycardia or ventricular fibrillation
without a reversible cause remain at risk for recurrent arrhythmia with a high mortality rate.
Implantable cardioverter-defibrillator therapy is the treatment of choice in this population,
and has been shown to be superior to antiarrhythmic drug therapy. In addition, nonrandom-
ized studies have shown that patients with hypertrophic cardiomyopathy at high risk of
sudden death benefit from an implantable cardioverter-defibrillator, even those patients
who are already on β-blocker therapy.

Myomectomy is mainly reserved for patients who are symptomatic from outflow obstruc-
tion (the case patient was feeling well prior to her arrest) but it is unclear whether relief of
outflow obstruction affects survival. β-Blockers have not been shown to improve survival
in patients with hypertrophic cardiomyopathy. Avoidance of strenuous exercise is recom-
mended to patients with hypertrophic cardiomyopathy, but it is not known whether this
helps survival.
A 72-year-old man is evaluated in the emergency department for the sudden onset of severe
sharp anterior chest pain radiating into the back. He is a former smoker with a long history
of type 2 diabetes mellitus, chronic renal insufficiency (creatinine 2.0 mg/dL [176.84
μmol/L]), sick sinus syndrome with a DDD pacemaker implanted in 1995, and hyperten-
sion. His medications include insulin, furosemide, ramipril, and aspirin.

On examination, the blood pressure is 185/85 mm Hg bilaterally, and the pulse rate is
90/min and regular. A grade 2/6 systolic murmur and a soft decrescendo diastolic murmur
are heard at the second right intercostal space. There are abdominal and bilateral femoral
bruits, with absent distal pulses.

Which of the following is the most appropriate initial imaging study?

A Non-contrast chest CT
B Chest MRI
C Transesophageal echocardiography
D Transthoracic echocardiography
This question has been answered. To clear answers, open the Answer Sheet.
Answer and Critique (Correct Answer = C)
Key Points

 Chest CT scan with contrast is indicated to detect acute aortic dissection.


 In patients at risk for radiocontrast nephropathy and contraindication to MRI,
transesophageal echocardiography is the test of choice for possible aortic dissection.

The most important predisposing risk factor for acute aortic dissection in older patients is
hypertension. In the International Registry of Acute Aortic Dissection (IRAD), 72% of pa-
tients had a history of hypertension, but only 34% of those younger than 40 years. Bicuspid
aortic valve was more common in younger patients. In patients with Marfan's syndrome,
50% of those younger than 40 years had a family history of aortic dissection compared with
2% in older patients. Pain is also quite common, with only 6% of patients in the IRAD reg-
istry having painless dissection. A history of diabetes mellitus, aortic aneurysm, or cardio-
vascular surgery was more common in patients with silent dissection, along with a slight
increase in age. Syncope occurs in a small minority of cases, with an increased risk of
tamponade and stroke, as well as a worse outcome.

The evaluation of patients with suspected thoracic aortic aneurysm or dissection includes
chest CT with contrast, contrast-enhanced aortic MR angiography, and transesophageal
echocardiography. Although chest CT without contrast may be acceptable for detecting an
aortic aneurysm, it has a low sensitivity for aortic dissection. Chest radiographs have a low
sensitivity and low specificity for aortic dissection. Although gadolinium contrast is not ne-
phrotoxic, a chest MRI or contrast-enhanced aortic MR angiography would be relatively
contraindicated in this patient because he has an older pacemaker. Transesophageal echo-
cardiography is the most appropriate imaging and can safely be performed in the emergen-
cy department for patients with suspected acute aortic dissection

A 38-year-old woman is evaluated in the emergency department for generalized itching, an


erythematous skin rash, and joint pain. She initially tried over-the-counter diphenhydramine
but her itching and rash did not improve. She was diagnosed with a sinus infection 2 weeks
ago that was treated with a course of amoxicillin. Her sinus drainage and cough have im-
proved. However, her joint pain remains and her temperature has been between 37.5 °C
(99.5 °F) and 37.8 °C (100 °F). She states that she has otherwise been healthy and takes no
additional medications.

On physical examination, temperature is 37.3 °C (99.2 °F), pulse rate is 88/min, and blood
pressure is 122/68 mm Hg. There is a diffuse erythematous macular papular skin rash in-
volving her trunk, arms, and upper thighs.

Laboratory Studies
Hemoglobin 12.5 g/dL (125 g/L)
Leukocyte 9800/µL (9.8 × 109/L) (10%
count eosinophils)
Platelet 325,000/µL (325 × 109/L)
count
Blood urea 36 mg/dL (12.86 mmol/L)
nitrogen
Creatinine 2.6 mg/dL (229.89 μmol/L)
Sodium 138 meq/L (138 mmol/L)
Potassium 4.4 meq/L (4.4 mmol/L)
Bicarbonate 26 meq/L (26 mmol/L)
Urinalysis pH 5, specific gravity 1.020,
2+ blood, trace protein, 4+
leukocyte esterase, 20–25 leu-
kocytes and several leukocyte
casts/hpf, 3–5 intact erythro-
cytes/hpf, Hansel stain shows
eosinophils

Which of the following is the most likely diagnosis in this patient?

A Thrombotic thrombocytopenic purpura


B Antineutrophil cytoplasmic autoantibody–associated vasculitis
C Acute tubular necrosis
D Acute interstitial nephritis
E Membranous glomerulopathy
This question has been answered. To clear answers, open the Answer Sheet.
Answer and Critique (Correct Answer = D)
Key Point
The classical triad of acute interstitial nephritis (fever, skin rash, and arthralgias) in the set-
ting of acute or subacute renal failure is present in only a minority of affected patients.
Inflammatory disorders of the kidney such as acute interstitial nephritis due to antibiotics or
nonsteroidal anti-inflammatory drugs often are associated with sterile pyuria as well as
eosinophiluria on Wright's or Hansel's stains. Other conditions associated with kidney in-
flammation such as atheroembolic disease, postinfectious glomerulonephritis, rapidly pro-
gressive glomerulonephritis, and pyelonephritis may have eosinophiluria. Therefore, the
clinical presentation is essential in making an accurate diagnosis in this setting. The classic
triad of acute interstitial nephritis (fever, skin rash, and arthralgias) is present in only a mi-
nority of affected patients. More commonly, patients with this condition have only a fever
or skin rash in the setting of a medication exposure.

Thrombotic thrombocytopenic purpura should be considered in patients with fever, skin


rash, and kidney failure. However, the absence of concomitant anemia, mental status
changes, and thrombocytopenia makes this diagnosis less likely. Antineutrophil cytoplas-
mic autoantibody–associated small-vessel vasculitis also should be considered in patients
with kidney failure and concomitant arthralgias, skin rash, and fever. However, this pa-
tient's lack of dysmorphic erythrocytes or erythrocyte casts makes this diagnosis unlikely.
In addition, her exposure to medication is more suggestive of acute interstitial nephritis.

Suspicion for acute renal failure due to acute tubular necrosis from pyelonephritis should be
raised in a patient with kidney failure, fever, and pyuria. However, the lack of other clinical
symptoms of sepsis, such as hypotension and tachycardia, makes this condition less likely.
Membranous glomerulopathy due to systemic lupus erythematosus may present with skin
rash, fever, arthralgias, and kidney failure, but this patient's lack of an active urine sediment
and proteinuria are uncommon in this condition and favor the diagnosis of acute interstitial
nephritis from antibiotic use

A 25-year-old woman who is a new patient is evaluated for elevated blood pressure. She is
14 weeks pregnant, and this is her first pregnancy. She has a family history of hypertension
and type 2 diabetes mellitus.

On physical examination, blood pressure is 150/90 mm Hg. BMI is 28. The remainder of
her examination is unremarkable.

Laboratory Studies
Blood urea ni- 16 mg/dL (5.71 mmol/L)
trogen
Creatinine 1.5 mg/dL (132.63 µmol/L)
Sodium 136 meq/L (136 mmol/L)
Potassium 3.8 meq/L (3.8 mmol/L)
Chloride 100 meq/L (100 mmol/L)
Bicarbonate 24 meq/L (24 mmol/L)
Urinalysis 2+ protein, no leukocytes
or erythrocytes

Which of the following is the most likely diagnosis?


A Preeclampsia
B Chronic essential hypertension
C Chronic glomerulopathy
D Gestational hypertension
This question has been answered. To clear answers, open the Answer Sheet.
Answer and Critique (Correct Answer = C)
Key Points

 Elevated blood pressure in early pregnancy is most likely caused by a chronic con-
dition.
 Glomerulonephritis, not preeclampsia, is the most likely diagnosis in patients with
elevated creatinine levels and proteinuria early in pregnancy.

This patient's elevated blood pressure, renal insufficiency, and proteinuria detected early in
pregnancy are most consistent with chronic glomerulopathy. Preeclampsia would not pre-
sent before 20 weeks gestation in the absence of a molar pregnancy. This patient's pro-
teinuria and elevated creatinine level indicate the presence of renal disease, which is not
consistent with chronic essential hypertension. In addition, signs of renal involvement, such
as proteinuria or mild azotemia, are unlikely in a young patient with essential hypertension.
Early hypertensive nephrosclerosis may present with these findings, but this condition is
highly unlikely in a 25-year-old patient.

Gestational hypertension is characterized by normal blood pressure measurements in early


pregnancy and hypertension that develops during the latter part of pregnancy. Unlike
preeclampsia, this condition is not associated with proteinuria, elevated creatinine levels, or
hyperuricemia. Gestational hypertension usually resolves after delivery; this presentation is
known as transient hypertension in pregnancy. However, essential hypertension is the most
likely diagnosis if affected patients do not become normotensive by 3 months postpartum;
the decrease in blood pressure associated with early pregnancy usually masks this condition
initially. Hypertension that presents in patients at <20 weeks gestation is most likely caused
by a chronic condition that preceded pregnancy.

A 32-year-old woman is hospitalized because of symptoms that began as numbness and


tingling in both feet and progressed over several days to include gait instability, hand
weakness, diplopia, and dyspnea. The symptoms began 11 days after a viral illness. On ad-
mission, she has sinus tachycardia, proximal and distal weakness in her upper and lower
extremities bilaterally, areflexia, and marked vibratory and position sense loss in the toes
and fingers. She cannot walk.

Which of the following is the most appropriate treatment?

A Prednisone
B Pyridostigmine
C Plasma exchange
D Intravenous methylprednisolone
This question has been answered. To clear answers, open the Answer Sheet.
Answer and Critique (Correct Answer = C)
Key Points

 Guillain–Barré syndrome is characterized by proximal and distal weakness, auto-


nomic symptoms, cranial nerve involvement, and respiratory failure.
 Treatment of Guillain – Barré syndrome with either intravenous immunoglobulin
or plasmapheresis is indicated in patients who cannot walk independently or who
have impaired respiratory function or rapidly progressive weakness.

This patient's signs and symptoms are most consistent with Guillain – Barré syndrome.
Guillain–Barré syndrome is an immune-mediated demyelinating polyneuropathy character-
ized by proximal and distal weakness, distal sensory loss, autonomic symptoms, cranial
nerve involvement, and respiratory failure in 25% of patients. Treatment consists of either
intravenous immunoglobulin therapy or plasmapheresis, which have been shown in clinical
studies to be equally effective. Plasmapheresis should be avoided in patients who have la-
bile blood pressures or infection. Intravenous immunoglobulin is contraindicated in patients
with renal insufficiency, congestive heart failure, or IgA deficiency. Both of these treat-
ments are expensive and have potential morbidity and should therefore be reserved for pa-
tients who are unable to walk independently, have impaired respiratory function, or have
rapidly progressive weakness.

Studies of oral corticosteroid therapy and intravenous methylprednisolone have shown no


significant clinical improvement in patients with Guillain–Barré syndrome. Pyridostigmine
is helpful for the symptomatic treatment of myasthenia gravis, a condition that may cause
similar symptoms of weakness, diplopia, and dyspnea but would not be associated with the
sensory symptoms or signs seen in this patient.

A 22-year-old man is evaluated in the emergency department 8 hours after the sudden onset
of moderate neck pain followed by vertigo, ataxia, slurred speech, and difficulty swallow-
ing. His medical history is unremarkable and he is not taking any medications. Physical ex-
amination shows left ptosis, anisocoria with the left pupil smaller than the right, nystagmus,
left-sided dysmetria, and decreased pain and temperature sensation on the left side of the
face and right side of the body. CT scan of the brain is normal.

Which of the following is the most appropriate next step in the evaluation of this patient?

A Repeat noncontrast CT scan of the brain in 24 hours


B Carotid ultrasound
C MRI and magnetic resonance angiography of the brain and neck
D Lumbar puncture
This question has been answered. To clear answers, open the Answer Sheet.
Answer and Critique (Correct Answer = C)
Key Points

 Vertebral artery dissection typically presents with neck or head pain, Horner's syn-
drome, dysarthria, dysphagia, decreased pain and temperature sensation, dysmetria,
ataxia, and vertigo.
 Magnetic resonance angiography is a sensitive diagnostic test for vertebral artery
dissection as a cause of stroke.

This patient has an ischemic stroke (cerebral infarction). The symptoms and signs involve
multiple lower cranial nerves (dysphagia, dysarthria), crossed sensory deficits, and cerebel-
lar ataxia, which suggest a left lateral medullary localization, possibly also involving the
left cerebellum. The sudden onset of symptoms suggests that stroke is the cause. The nor-
mal CT rules out a parenchymal intracerebral hemorrhage, which would be unlikely in the
medulla. Blood is supplied to this area by the posterior inferior cerebellar artery, a major
branch of the vertebral artery. In a previously healthy young person, the less common caus-
es of stroke must be considered, such as vertebral artery dissection, which often occurs
spontaneously without trauma or typical vascular risk factors. Typical symptoms of verte-
bral dissection include neck or posterior head pain, Horner's syndrome (ptosis and miosis),
dysarthria, dysphagia, decreased pain and temperature sensation of the face and contrala-
teral body, dysmetria, ataxia, and vertigo. Magnetic resonance angiography is an excellent
tool in diagnosing dissection. Noncontrast CT scan in 24 hours will only reveal the evolv-
ing stroke, not its cause. Carotid ultrasound studies do not reliably characterize abnormali-
ties in the vertebral artery other than reversal of flow. Lumbar puncture is used to evaluate
suspected subarachnoid hemorrhage in a patient who has a severe headache with a normal
CT scan, but such localized medullary symptoms would be atypical for subarachnoid hem-
orrhage.

٢٠١٢ ‫ﻣﺤﻤﺪ ﻋﺒﺪاﻟﺮﺣﯿﻢ‬.‫ﻣﻊ ﺗﺤﯿﺎﺗﻲ د‬


Best of five question recall
MD exam July 2017
By : Batol Gurashi
1- 50 years old male presented to emergency room with severe central chest pain ECG
done in the ER revealed ST segment elevation in lead 2, 3 and AVF he was admitted to
the CCU given thrombolysis and become stable. Three days later he developed severe
SOB O/E he is dyspneic RR 30 b/min PR 100 b/min BP 70/50 with raised JVP chest
examination there is crackle all over, cardiovascular examination normal heart sound no
murmers.
What could be the cause of deterioration?
1- pump failure
2- rupture papillary muscles
3- VSD
4- cardiac tamponade
5- mitral reguirgitation
1- 42 years female diabetic patient for 15 years of average control presented complain of
upper limb pain for 6 weeks some time weakness in her manual work examination of her
upper limb revealed normal power but exaggerated reflexes she has positive tinel test more
in the left what is your diagnosis
1-mononeuritis multiplex
2-myelopathy
3-compression neuropathy
4-cervical radiculopathy
5-vasculitic neuropathy
2- 40 years obese lady, she took a medicine to lose weight presented with SOB O/E she is
morbidly obese dyspniec at rest chest examination is clear cardiovascular normal except for
loud second heart sound ABG done showed PO2 is 8 KPa and PCO2 is 7 KPa what could
be the diagnosis
1-primary pulmonary hypertension
2-secondary pulmonary hypertension
3-obesity hypoventilation
3- There is bacteria called wolbachia endobacteria discovered to have an effect of
development of loa loa , which of the following drug is used to treat this bacteria so as to
eradicate loaliasis?
1-co amoxiclav
2-doxycycline
4- 35 years pregnant female in the 34 weeks of her pregnancy presented with SOB for 2 days
she is previously well O/E she is dyspniec at rest PR 100b/min BP 100/70 raised JVP chest
examination there is bibasal crackles cardiovascular examination there is galloping she has
lower limb oedema she immediately started IV diuretics, in addition to diuretics which of
the following will be required for treating this patient

1-hydarlazine

2-ramipril

3-digoxin

4-bisoprolol

6-which of the following may need an adjuvant steroid in treatment

1-strongyloides stercolaris

2-loa loa

3- onchocerca volvolus

4-wuchereria bancrofti

7-which of the following is treated with single dose of ivermectin

1-loa loa

2-onchocerca volvolus

3-stongyloides stercolaris

4-ascaris lumbricoides

8- which of the following has some action against schistosomula

1-praziquantel

2-oxamniquine

3-artemether

4-metrifonate

9- 30 years old male with history of severe muscular disease he is bed ridden presented with
fever and cough O/E he is febrile temp 39 ,oxygen saturation is 92 chest examination there is
crackles what investigation you will immediately require

1-ABG

2-CXR

3-CRP

10-Which of the following medication is absolutely contraindicated in bronchial asthma

1-adenosine

2-verapamil
3-bisoprolol

11- 45 years male presented with his wife as she described him as being strange for three days
he developed odd behavior and he was wandering around the house without aim he has just
have flu like illness one week ago O/E he is confused not febrile no neck rigidity intact cranial
nerve normal chest cardiovascular and abdomen what is the drug of choice in his case

1-cefriaxone

2-cefotaxime

3-acyclovir

12-30 years old female nurse unfortunately she sustained needle stick injury from patient with
hepatitis B she presented I month later with fever, vomiting and jaundice LFT done ALT 700
AST 600 ALP 130 bilirubin 10 , which one of the following will be positive in this patient

1hepatitis B surface antigen and anti core immunoglobulin G

2- hepatitis B surface antigen and anti core immunoglobulin M


3- hepatitis B surface antigen and hepatitis e antigen
4- hepatitis B surface antigen and hepatitis e antibodies

13-we say this is multidrug resistant tuberclosis when the patient has resistance to

1- INH,rifampicin,pyrazinamide,ethambutol
2- INH and rifampicin
3- INH,rifampicin,pyrazinamide

14-46 years old laborer with flu illness 2 weeks ago presented with inability to stand

O/E flaccid paralysis reduced reflexes planter equivocal and sensation is normal

Investigations TSH less than 0.01 , what is the investigation of choice?

1-serum K

2-MRI spine

3-LP

4-EMG

15-which of the following is expected regarding a 68 years male type 2 DM diagnose with RTA
type 4

1- aminoaciduria
2- fludicortizone treatment is effective
3- increase GFR is expected
4- increase urinary bicarbonate
5- normal renal handling of K and H

16- 3o years old male working in pet shop presented with dry cough and severe SOB for 3 days
then he developed severe diarrhea and abdominal pain which of the following could be the
causative organism

1- legionella
2- H5N1

17-this is the iron study of the patient with anemia his serum iron is ….. (low) TIBC ….. (low)
serum ferritin ……..( high) what is likely cause of his anaemia

1-iron deficiency anaemia

2-sideroplastic anaemia

3-anaemia of chronic disease

4-hemolytic anaemia

18-35 years old female known case of antiphopholipid syndrome presented complain of severe
chest pain her ECG showed ST segment depression in V1 to V3 and her troponin is positive what
is the diagnostic investigation

1-coronary angiography

2-CTPA

3-echocardiography

19-65 years old male presented with chest pain ECG done in ER is shown below ( anterolateral
STEMI) troponin is positive he has history of stroke 3 months ago what will be the best
treatment

1-thrombolysis

2-emeregency Catheter

3-Heparin

20- Patient with COPD he has FEV1 of 33 according to the gold criteria for severity of COPD this
patient has

1-mild

2-moderate

3-severe

4-very severe
21-which of the following can be caused by plasmodium vivax

1-cerebral malaria

2-severe anaemia

3-acute renal failure

4-hypoglycemia

22- 66 years old male presented with fever and cough O/E he is conscious but febrile RR 30

BP 70/50 chest examination revealed signs of consolidation his urea is 11 mmol what is his CURB
score?

1-3

2-4

3-5

23- 30 years old male presented with fever and productive cough which is preceded by cold
sore around his mouth O/E he is febrile chest examination there is signs of consolidation in the
right lower zone what is the most likely organism ?

1- Staph aureus
2- Kliebsiella
3- Strept. Pneumoniae
4- Pseudomonas aurginosa

24-65 years old patient presented with painful right knee . he has history of congestive cardiac
failure to which he use to take hydrochlorothiazide , lisinopril and bisoprolol and spironolactone
O/E he is in pain has raised JVP chest has bilateral basal crackles and lower limb oedema
investigation uric acid 11 what is the best management regarding his knee problem ?

1- IM Diclofenac
2- IV colichicine
3- Intraarticular steroid

25- 35 years old female has history of sudden loss of vision twice she also developed left sided
weakness which resolved completely, which of the following will maximally help her remission

1-steroid

2- interferon alpha

3-alemtuzumab

4-glatimer
5-natalizumab

26- 55 years diabetic patient accompanied by his wife who said that he use to forget the things
surrounding him and keep asking same question many time during the day. His symptoms last
for the last six months no history of trauma, what could be the cause?

1-early demenitia

2-transient global amnesia

27-30 years old healthy patient who get trauma in his chest while playing football he is
asymptomatic chest x ray done in ER revealed pnemothorax with rim of air less than 2 Cm what
is your management

1-aspiration

2-chest tube

3-Reassure and discharge home review him in clinic later

28-25 years old female came complain of palpitation sweating and tremor which relieved by
eating food it occurred four times before O/E normal except for BMI 26

Investigation RBS 55 mg/dl insulin (normal) what will be your next plan

1-glucose monitoring at home

2- 72 hours fasting test

3-CT abdomen

29-54 years old male presented with right leg swelling for one day he has no significant history

Investigation proved right lower limb DVT . CBC Hb 13 gm/dl WCC 9000 plts 1,000,000 RFT
normal what is appropriate investigation

1-bone marrow trephine biopsy

2-protien electrophoresis

3-JAK2

30-15 years old boy presented with left knee swelling examination revealed knee effusion which
is proven to be blood when aspirated investigation done: CBC (normal), APTT (high) ,

PT (normal), bleeding time (normal), factor 8 (normal) what is the diagnosis ?

1-haempophilia A

2-haemophilia B

3-Vonwillibrand disease
31-56 years old patient presented to emergency room complain of haematemesis he has end
stage renal disease on haemodialysis . four unit of blood is prepared to be given and endoscopy
will be done , which of the following will be of prognostic value if given with blood

1-platelets transfusion

2-FFP

3-Vit K

4-octeriotide

5-cryopreciptate

32-know HIV patient presented with visual impairment fudus examination revealed exudate
alongside retinal vessel what is appropriate management?

1-ceftriaxone

2-ganciclovir

3-flucytocine

33-50 years patient who is known to have decompensated chronic liver disease on
spironolactone and laculsoe 30 ml three times a day presented with confusion and he has
positive flapping tremor what is your next action

1- Increase dose of lactulose


2- Rifaximine

34-what is true regarding leflunamide

1-relative contraindicated in pregnancy

2-irreversible alopecia

35- 34 years old diabetic patient presented with generalized body swelling 24 hours urinary
protein is 6 gms what is your management

1- ACE inhibitor
2- Prednislone
3- Renal biopsy

36-32 years old female presented for health checkup she is just taking Oral contraceptive pills
investigation CBC Hb 12 gm /dl WCC 5000 Platelets 21,000. peripheral blood picture showed
clumps of platelets what is your next action

1- Start prednislone
2- Give platelete
3- Bone marrow aspirate
4- Platelets antibody level
5- Repeat the CBC after putting blood in heprinized tube
37- 50 years old female who is taking chemotherapy for her breast malignancy presented with
generalized body swelling she has protienuria renal biopsy done it showed glomerualar
subepithelial deposition of C3 and Ig G what is the renal diagnosis

1- Membranous GN
2- Minimal change GN
3- Diffuse proliferative GN
4- Focal glomerulosclerosis

38-40 years diabetic male presented with haemoptysis CXR done revealed multiple cavities with
halo sign he has positive galactomannan test what is the diagnosis?

1- Aspergilloma
2- Invasive aspergillosis
3- Exterensic allergic alveolitis

39- 50 years old male presented with recurrent abdominal pain after meals and diarrhea bulky
stool for the last six months he also has significant loss of weight , he is alcoholic O/E nothing
significant investigation CBC Hb 13gm/dl MCV 105 WCC 5,000 plts 250,000 normal RFT normal
LFTs how you will confirm the diagnosis?

1-colonoscopy

2-CT abdomen

3-upper GI endoscopy

40- 30 years old male he has history of recurrent UTI presented with loin pain and haematuria
US abdomen done it showed multiple renal stone bilaterally what condition may this man
complain of

1- Medullary sponge kidney


2- Medullary cystic disease
3- Polycystic kidney disease
4- Horseshoe kidney

41- 42 years old male investigated for previous history of DVT and PE CBC Hb 9 gm/dl

WCC 2000 plts 100,000 how you will confirm the diagnosis

1- Acid lysis test


2- Blood immunophenotyping
3- Bone marrow biopsy

42- 24 years pregnant lady underwent checkup investigation her TFT is shown: ( normal FT3
normal FT4, low TSH (0.1) what will be your action

1- Start antithyroid treatment


2- Request antithyrotopin hormone
3- Reassurance
43- 41 years old lady incidentally found to have adrenal mass while investigated for other issue

She is asymptomatic , how to exclude subclinical cushing?

1- 24 hours urinary cortizol


2- Low dose dexametasone suppression test
3- Salivary gland cortizol level
4- Dexamethasone every 6 hours and measure cortizol level

44- 19 years college student who is absent from the class for three days found by his colleague in
his room collapsed on examining him he has tremor and his pupil is dilated sluggish reaction his
BP 140/90 PR 100 b/min what would be the cause of collapse?

1-ethanol intoxication

2-ectasy

3-CO poisoning

45- 53 years know HIV presented with forgetfulness his mininmental examination is 10 out of 30
his CT brain is normal what is your treatment?

1- Acyclovir
2- Start him HAART
3- Flucytosine

46- 43 years old known HIV who recently started HAART and treated treated for pneumocystis
carnii infection with pentamidine, is investigated for hyperkalemia He is found to have BP 90/60
RFT normal urea and creatinine, his K is 6 mmol Na 130mmol what is the likely cause?

1- addison disease
2- pentamidine side effect
3- HAART side effect

47- 80 years old female who had history of complex surgery for upper limb fracture presented
with right wrist pain and swelling she underwent diagnostic aspiration from her joint what do
you think it will be positive in this fluid

1- congo red dye


2- weakly positive birefringence
3- negative birefringence

48- 42 years female presented with loss of lipido O/E she has breast atrophy. Her last menstrual
period was 18 years ago when she delivered her last daughter who couldn’t lactate What will be
the cause ?

1- prolactinoma
2- pituitary apoplexy
3- autoimmune ovarian failure
49- 30 years old male newly discovered diabetic what will be positive?

1- Anti insulin antibodies


2- Glutamic acid decarboxylase

50- 65 years old male who underwent small bowel resection presented with loose stool his
investigation revealed CBC Hb 10 gm/dl MCV 110 WCC 6,000 PLTS 350,000 LOW B12
normal folate what is your diagnosis?

1- crohns disease
2- bacterial overgrowth
3- pernicious anaemia

51- 25 years old lady who is asymptomatic discovered to have mitral prolapse she will undergo
upper GI endoscopy for dyspepsia, what about antibiotic prophylaxis?

1- No need because it is minor valvular lesion


2- No need because it is minor procedure

52-54 years old presented 6 weeks after renal transplantation with fever and generalized
lymphadenopathy what is the most likely organism?

1- EBV
2- CMV

53-29 year’s old male presented with nausea and vomiting for 3 days O/E he has dry mucous
membrane investigation RFT urea 130 S.cr 3.5 k 5.6 urinary sodium 20 urinary creatinine 3000

What is the cause of his renal impairment ?

1- Acute tubular necrosis


2- Renal azotemia due to vomiting

54- which of the following its entry through the skin?

1- Ascaris lumbricoides
2- Loa loa
3- Strogyloides stercolaris

55- 47 years old with kidney transplant, he is on immunosuppressive therapy on tarcolimus


cyclosporine and steroid he presented with tonsillitis which of the following is contraindicated?

1- Ciprofloxacin
2- Erythromycin
3- Coamoxiclav

56- 28 years old lady known case of mythenia gravis on prednisolone 10 mg and azathioprine 50
mg presented with increasing difficulty of swallowing for three days and it reached the
maximum what is your next action?

1- Increase dose of steroid


2- Increase the dose of azathioprine
3- IV IG

57- which of the following complication of typhoid occur during the third week?

1- Myocarditis
2- Rose spot

58- 30 years female presented with dry eyes O/E she has little saliva and enlarged
submandibular glands which of the following will be positive

1- Anti SSA/Ro
2- Anti dsDNA
3- RF

59- 27 years old female presented with motion sickness she was given promethasine but it has
little effect what will be an alternative medications?

1- Hyosine
2- Ondasetron

60- what is true regarding new oral anticoagulant

1- Increase risk of bleeding


2- Difficult reversal of its effect
3- Contraindicated in valvular lesion

61- 50 years old male presented with progressive limb weakness for 3 months O/E fully
conscious and has intact cranial nerves examination of limbs there is wasting , power grade 2
with hypotonia and hyporeflexia planter reflex is mute bilaterally sensation is intact how u will
confirm the diagnosis

1- EMG
2- NCS
3- MRI spinal cord
4- Muscle biopsy
62- 21 years old female presented with high grade fever and rash for 3 days she has clear
medical background O/E she is very ill temp 40 BP 90/60 she has bleeding from her
hypertrophied gums and ecchymosis in her skin chest CVS abdomen normal examination
investigation revealed Hb 8 gm/dl wcc 2000 Plts 80000 fibrin degradation product is high and
fibrin( low) , how would you will reach the diagnosis

1- Peripheral blood smear


2- Bone marrow examination
3- Protien electrophoresis
63- 40 years old male has history of chronic sinusitis and recurrent upper respiratory tract
problem O/E nothing apart from saddle nose, investigation : Urea 200 mg/dl Scr 6 mg /dl urine
analysis contain RBCs what is the likely diagnosis?

1- Goodpasture syndrome
2- Wegeners granulomatosis
3- Churg strauss syndrome
64- 30 years old female presented with worsening renal function she has history of upper
respiratory problem examination nothing significant investigation RFT urea 180 S. cr 5 urine
contain RBCs and proteins p ANCA (antimyeloperioxidase) is positive what is likely diagnosis?

1- Wegeners granulomatosis
2- Churgstrauss Syndrome
3- Microscopic polyangitis
4- Goodpasture syndrome
5- PAN
65- what is true regarding hyperreactive malaria syndrome

1- Common in male
2- Increase in immunoglobulin G
3- Treated with quinine for years
66- 25 years old pregnant lady presented with vomiting and severe palpitations she has history
of weight loss for 6 months O/E she is febrile with dry mucous membrane PR 155 b/min BP
90/60 what is your immediate action?

1- Iv fluids and antiemetics


2- Iv dexamethasone
3- Iv fluids and propyothiouracil
67- 34 years old lady presented with and she had decrease level of consciousness and agitation
she had history of weight gain and constipation for 6 months examination she is obese confused
and irritable temp 35.5 PR 62 b/min chest cardiovascular and abdomen examinations are
normal what is your immediate management?

1- IV Dexamethasone

2- IV thyroxine
3- IV dextrose
68- 70 years old lady presented with fatigue and she is unable to raise her arm to comb her hair
investigation done S Ca low, phosphate low , Alp high what is your management ?

1- Ca supplements
2- Vit D and Ca supplements
3- Steroid
4- Hormone replacement therapy
69- 40 years old lady who is known case of chronic viral hepatitis presented with purpuric rash
her investigation showed RF positive and low c4 what is the diagnosis

1- PAN
2- Cryoglobulinemia
70- Gut associated lymphoid tissue treatment in patient with history of dyspepsia

1- Helicobacter eradication
2- Chemotherapy
3- Radiotherapy
4- Surgery
71- patient with fatigability more pronounced in the evening underwent investigations CXR
showed mediastinal mass what investigation you will request

1- Anticholinesterase antibody
2- Ct chest
3- NCs
72- pregnant lady known case of rheumatoid arthritis presented with painful swelling in her
fingers joint what treatment you will start

1- Sulphasalazine
2- Methotrexate
3- Leflunamide
73- patient has hypopigmented skin lesion on treatment for it presented with redness and
swelling in the site of the skin lesion what is the cause

1- Leprae reaction type 1


2- Leprae reaction type 2
74- Patient has hypopigmented skin lesion on treatment presented with pain and tenderness
over the site of lesion and nerves he was given steroid but it has little benefit what is the
additional drug you will add
1- Clofazimine
2- NSAIDs
75- Patient known to have coarctation of aorta who was treated in his childhood presented
with fatigue and dyspnea examination Bp 130/50 pulse large volume has systolic murmer
what will be the cause
1- Bicuspid valve with AR
2- Bicuspid valve with AS and AR
3- Recurrence of coarctation
76- 17 years lady presented with syncopal attack for the last 3 months it last for seconds
while standing it preceeded by dizziness and disorientation her sister state that the last
attack continued for 2 minutes ended with complete recovery what is your investigation
to know the nature of the attack
1- EEG
2- 24 hrs holter monitoring
3- CT brain
4- Vestibular testing
77- Patient with schizophrenia presented with high grade fever and rigidity O/E he is febrile
temp 40 and has rigidity all over his body what is your immediate action?
1- Diazepam
2- Dantrolene
78- Why pyridoxine is given with INH ( mechanism of action )
1- INH inhibit conversion of methionine to cystathionine ??????????
2- ?????????????????????????
79- Patient with antiphospholipid syndrome developed DVT and was put on warfarin with
target INR 2 to 3 developed another DVT what is your plan?
1- Lifelong warfarin with target INR 2-3
2- Lifelong warfarin with target 3-4
80- 15 years girl presented with haeamturia and abdominal pain she has purpuric rash over
the extensor surafce of her lower limbs what is diagnosis?
1- cryoglobulinemia
2- Henoch schonlien purpura
3- PAN
81- 60 years old male presented with palpitations he is not diabetic nor hypertensive he has
no history of stroke nor TIA O/E he is not dyspiec PR 82 b/min, irregular irregular chest
cardiovascular and abdomen examination is normal what is drug of choice
1- warfarin
2- Aspirin
82- 30 years female presented with fatigability and epistaxis O/E she is pale no
lymphadenopathy no organomegally investigations CBC Hb 8.6 gm/dl Wcc 5,000 Plts
80,000 coombs test positive what is diagnosis?
1- Autoimmune haemolytis anaemia
2- Evans syndrome
3- SLE
83- 30 years old patient presented to emergency as he is involved in road traffic accident he is
asymptomatic however his CXR showed that his left hemidiaphragm is elvated how you
will confirm the diagnosis?
1- Pulmonary function test
2- ABG
3- Sniff test
4- Pulse oximetry
5- CT scan for his chest
84- Patient with symptoms and signs of fibromyalgia what is the treatment?
85- Patient with symptoms of anaemia and feature of autoimmune disease (Autoimmune
haemolytic anaemia) what next investigation?
Antiglobulin test

‫مع تمنياتي للجميع بالتوفيق‬

You might also like